You are on page 1of 481
ELEMENTS OF THE DIFFERENTIAL AND INTEGRAL CALCULUS ® (REVISED EDITION) | BY WILLIAM ANTHONY GRANVILLE, Pu.D., LL.D. PRESIDENT OF GETTYSBURG COLLEGE WITH THE EDITORIAL COUPERATION OF PERCEY F. SMITH, Pu.D. PROFESSOR OF MATHEMATICS IN-THE SHEFFIELD SCIENTIFIC SCHOOL YALE UNIVERSITY GINN AND COMPANY BOSTON + NEW YORK + CHICAGO - LONDON ATLANTA + DALLAS + COLUMBUS - SAN FRANCISCO corynn r, 1904, 1911, BY WinitAM ANTHONY GRANVILLE AND Percy F. SMITH ALL RIGHTS RESERVED ont Tbe Atheneum Press GINN AND COMPANY - PRO- PRIETORS - BOSTON U.S.A. PREFACE That teachers and students of the Calewlus have shown such a gen- erous appreciation of Granville’s “Elements of the Differential and Integral Calculus” has been very gratifying to the author. In the last few years considerable progress has been made in the teaching of the elements of the Calculus, and in this revised edition of Granville’s “Calculus” the latest and best methods are exhibited, — methods that: have stood the test of actual classroom work. Those features of the first edition which contributed so much to its usefulness and popu- larity have been retained. The introductory matter has been cut down somewhat in order to get down to the real business of the Calculus sooner, As this is designed essentially for a drill book, the pedagogic principle that each result should be made intuitionally as well as analytically evident to the student has been kept constantly in mind. The object is not to teach the student to rely on his intuition, but, in some cases, to use this faculty in advance of analytical investigation. Graphical illustration has been drawn on very liberally. ‘This Calculus is based on the method of limits and is divided into two main parts, — Differential Calculus and Integral Calculus. As special features, attention may be called to the effort to make per- fectly clear the nature and extent of each new theorem, the large number of carefully graded exercises, and the summarizing into working rules of the methods of solving problems. In the Integral Calculus the notion of integration over a plane area has been much enlarged upon, and integration as the limit of a summation is con- stantly emphasized. The existence of the limit ¢ has been assumed and its approximate value calculated from its graph. A large num- ber of new examples have been added, both with and without answers. At the end of almost every chapter will be found a col- lection of miscellaneous examples. Among the new topics added are approximate integration, trapezoidal rule, parabolic rule, orthogonal : vi PREFACE trajectories, centers of area and volume, pressure of liquids, work done, etc. Simple practical problems have been added throughout; problems that illustrate the theory and at the same time are of interest to the student. These problems do not presuppose an ex- tended knowledge in any particular branch of science, but are based on knowledge that all students of the Calculus are supposed to have in common. ‘The author has tried to write a textbook that is thoroughly modern and teachable, and the capacity and needs of the student pursuing a first course in the Calculus have been kept constantly in mind. The book contains more material than is necessary for the usual course of one hundred lessons given in our colleges and engineering schools ; but this gives teachers an opportunity to choose such subjects as best suit the needs of their classes. It is believed that the volume con- tains all topics from which a selection naturally would be made in preparing students either for elementary work in applied science or for more advanced work in pure mathematics. WILLIAM A. GRANVILLE Gurrvsnone Connnce Gettysburg, Pa. CONTENTS DIFFERENTIAL CALCULUS CHAPTER I COLLECTION OF FORMULAS SECTION 1. Formulas from Algebra, Trigonometry, and Analytic Geometry 2. Greek alphabet 8. Rules for signs in the four quadrants 4. Natural values of the trigonometric functions 5. Tables of logarithms CHAPTER IL VARIABLES AND FUNCTIONS 6. Variables and constants : . . : . . + . 7. Interval of a variable. : . . : : : . . 8. Continuous variation 9. Functions 10. Independent and dependent variables 11. Notation of functions 12. Values of the independent variable for which a function is defined CHAPTER IIT THEORY OF LIMITS 13. Limitofavariable =... 14. Division by zero excluded 15. Infinitesimals . . . . * . . . 16. The concept of infinity _ 17. Limiting value of afunction. =... we 18. Continuous and discontinuous functions 19. Continuity and discontinuity of functions illustrated by their graphs 20. Fundamental theorems on limits . aoe 21. Special limiting values . 22. The limit of as 28. The number e 5 24. Expressions assuming the form =... 5s vii race aR eee warraaa abl 12 13 13 = ney 16 18 20 21 22 23, ay viii CONTENTS CHAPTER IV DIFFERENTIATION SECTION 25. Introduction . 26, Increments 27. Comparison of increments 98. Derivative of a function of one variable 29. Symbols for derivatives . 30. Differentiable functions . 31. General rule for differentiation 82. Applications of the derivative to Geometry CHAPTER V PAGE 25 25 26 27 28 29 29 BL RULES FOR DIFFERENTIATING STANDARD ELEMENTARY FORMS 33. Importance of General Rule . 84. Differentiation of a constant . 85. Differentiation of a variable with respect to itself. 36. Differentiation of a sum 87. Differentiation of the product of a constant and a function 38. Differentiation of the product of two functions 39. Differentiation of the product of any finite number of fumetions 40. Differentiation of a function with a constant exponent . 41. Differentiation of a quotient . 42. Differentiation of a function of a function 43. Differentiation of inverse functions 44. Differentiation of a logarithm 5 45. Differentiation of the simple exponential function . 46. Differentiation of the general exponential function 47. Logarithmic differentiation 48. Differentiation of sin» . 49. Differentiation of cos» . 50. Differentiation of tanv . 51. Differentiation of cotv . 2. Differentiation of seev . . Differentiation of esev . Differentiation of versv. 5. . Differentiation of are sin» . Differentiation of are cos » . Differentiation of are tan o . Differentiation of are cot » . Differentiation of arc see » . Differentiation of are ese » 34 36 37 387 37 38 38 39 40 44 45 46 48 49 50 54 5D 56 56 56 BT 57 61 62 62 63 63 64 | CONTENTS ix \ SECTION PAGE | G1. Differentiation of areversv 2 see OB | 62. Implicit functions . S . i. = eS . oe 68. Differentiation of implicit functions. 9. 0... ss OD CHAPTER VI SIMPLE APPLICATIONS OF THE DERIVATIVE 64. Direction of a curve. 73 65. Equations of tangent and normal, lengths of subtangent and subnormal. Rectangular cobrdinates. 5. sete TB 66. Parametric equations of acurve . 79 i 67. Angle between the radius vector drawn on point Go ours oa as | tangent to the curve at that point 7 . rn: | 68. Lengths of polar subtangent and polar subnormal =... 86 i 69. Solution of equations having multiple roots. oe 70. Applications of the derivative in mechanics. Velocity. . | | 90 Z. Componcutvelocitics © = 72. Acceleration a h—hlrar—Ss=s=SsaC 78. Component accelerations =». sw wee 8 CHAPTER VIL SUCCESSIVE DIFFERENTIATION \ 74, Definition of successive derivatives 2... wes | 75. Notation. err——ssss i 76. The nth derivative | ot TT. Leibnita’s formula for the nth derivative of a product. =. =. 98 78. Successive differentiation of implicit functions . =. =... 100 CHAPTER VIIT MAXIMA AND MINIMA. POINTS OF INFLE! TION. CURVE TRACING 79. Introduction —. a... i 80. Increasing and decreasing functions . 106 81. Tests for determining when a function is increasing and when de- creasing =. oo le 82. Maximum and minimum values of function | » 109 i 88. First method for examining a function for maximum and minimum values. 0. = ll | 84. Second method for examining a function for maximum and minimum values. 12 ..LDrt~”C”—OC—COdC;SOOSCORSCSC«SCiSCdsCdsSCSsS¥SCéséSKsé flection. Be 185 i SGcCuvetuacme Ce : CONTENTS CHAPTER IX DIFFERENTIALS SECTION 87. 88. 89. 90. 91. 92. 93. 94. 95. 96. 97. 98. 99. 100. 101. 102. 103. 104. 105. 106. 107. Introduction Definitions Infinitesimals : Derivative of the aro in restangular cobrdinates Derivative of the are in polar codrdinates . Formulas for finding the differentials of functions Successive differentials . . . . CHAPTER X RATES ‘The derivative considered as the ratio of two rates CHAPTER XI CHANGE OF VARIABLE Interchange of dependent and independent variables . Change of the dependent variable Change of the independent variable . Simultaneous change of both independent and dependent variables CHAPTER XII CURVATURE, RADIUS OF CURVATURE Curvature . 5 Curvature of a circle . Curvature at a point . Formilas for curvature Radius of curvature . : : : - Circle of curvature. 5 a _ a CHAPTER XIII ‘THEOREM OF MEAN VALUE. INDETERMINATE FORMS Rolle’s Theorem oo inecrheona cP Moak vale 8 oN ce The Extended Theorem of Mean Value... PAGE 131 181 182 134 135 187 139 141 148 149 150 152 155, 155 156 156 159 161 164 165 166 CONTENTS SECTION 108. Maxima and minima treated a 7 109. Indeterminate forms . : 110. Evaluation of a function taking ... 111. Evaluation of the indeterminate form : oo 112. Evaluation of the indeterminate form... we 113. Evaluation of the indeterminate form 0-0 114. Evaluation of the indeterminate form « — 0 7 115. Evaluation of the indeterminate forms 0°,1°,0® . . CHAPTER XIV CIRCLE OF CURVATURE. CENTER OF CURVATURE 116. Circle of curvature. Center of curvature 117. Second method for finding center of curvature . 118. Center of curvature, the limiting position of the intersection of nor- mals at neighboring points . 119. Evolutes . 120. Properties of the evolute : 121. Involutes and their mechanical construction CHAPTER XV PARTIAL DIFFERENTIATION 122. Continuous functions of two or more independent variables 128. Partial derivatives G6 124, Partial derivatives interpreted geometrically |. 125. Total derivatives 126. Total differentials so 197. Differentiation of implicit functions. =. se 128. Successive partial derivatives 2. 0... 129. Order of differentiation immaterial CHAPTER XVI ENVELOPES 130. Family of curves. Variable parameter... 181. Envelope of a family of curves depending on one parameter : The evolute'of a given curve considered as the envelope of its normals . Two parameters connected by one equation of condition xi Page 167 170 170 171 174 174 175 176 178 180 181 182 186 187 190 191 192 194 197 198 202 208 205 205 208 209 xii CONTENTS CHAPTER XVII SECTION ee PAGE, eae tutroducom le 135. Infinite series. ee, OB 186. Existence of a limit . eg 187. Fundamental test for eonvergence | 5 sss RB 138. Comparison test for convergence =... ws 189. Cauchy's ratio test for convergence... www 8. 40 Alternating cores rl Cl 141. Absolute convergence =. ss we eee 200 dea Powerserieg 2 ts—ri—“(<—er”ssts=aC=s=CzdCl CHAPTER XVIII EXPANSION OF FUNCTIONS 143. Introduction eo 14. Taylor's Theorem and Taylor's Series =. | sss. 298 145. Maclaurin’s Theorem and Maclaurin’s Series . =... . 280 146. Computation by series. so eee 147. Approximate formulas derived from series. Interpolation. | | 987 148. Taylor’s Theorem for functions of two or more variables... =. 240 149. Maxima and minima of functions of two independent variables. 243 CHAPTER XIX ASYMPTOTES, SINGULAR POINTS 150. Reetilinear asymptotes ke 151. Asymptotes found by method of limiting intercepts. |. 240 152. Method of determining asymptotes to algebraic curves... 250 153. Asymptotes in polar codrdinates =. sw wwe 154. Singular points... 5 255 155. Determination of the tangent to an algebraic curve at a given point Dy inepection os 156. Nodes. we ee 888 157. Cusps we ee 158. Conjugate or isolated points «=» | | ss est 260 159. Transcendental singularities . 2... +. ey 260 CHAPTER XX APPLICATIONS TO GEOMETRY OF SPACE 160. ‘Tangent line and normal plane to a skew curve whose equations are Given inperemeticform S20 16l Tangent plenstaeourtece CONTENTS seotiox 162. Normal line to a surface : 168. Another form of the equations of the tangent line to a skew enrve 164. Another form of the equation of the normal plane to a skew curve CHAPTER XXI CURVES FOR REFERENCE INTEGRAL CALCULUS CHAPTER XXII INTEGRATION. RUL FOR INTEGRATING STANDARD ELE) FORMS 165. Integration oo. 166. Constant of integration, Indefinite integral 167. Rules for integrating standard elementary forms 168. Trigonometrie differentials 169. Integration of expressions containing Va? — 2? or VE + a bya trigo- nometric substitution . CHAPTER XXIII CONSTANT OF INTEGRATION 170. Determination of the constant of integration by means of initial con- ditions 171. Geometrical signification of the constant of integration 172. Physical signification of the constant of integration . CHAPTER XXIV THE DEFINITE INTEGRAL 178. Differential of an area 174. The definite integral . 175. Calculation of a definite ine 176. Calculation of areas . : 177. Geometrical representation of an antegral - 178. Mean value of $(2) . 179. Interchange of limits 180. Decomposition of the interval 181. The definite integral a function of its limits 182. Infinite limits 188. When y = $(2) is discontinuous xiii PAGE 266 268, 269 ENTARY, 279 281 282 298 304 307 307 309 314 34 316 318 319 320 320 320 321 321 822 xiv sEori0N 184. 185. 186. 187. 188. 189. 190. 191. 192. 198. 194. 195. 196. 197. 198. 199. 200. 201. 202. CONTENTS CHAPTER XXV INTEGRATION OF RATIONAL FRACTIONS Introduction 53 Col Case II Case IIT Case IV CHAPTER XXVI INTEGRATION BY SUBSTITUTION OF A NEW VARIABLE. RATIONALIZATION Introduction Differentials containing fruetional powers of z only Differentials containing fractional powers of a + bx only Change in limits corresponding to change in variable Differentials containing no radical except Va + bx + 2? Differentials containing no radical except Va + be — 2? Binomial differentials : : Conditions of integrability of binomial differentials . ‘Transformation of trigonometric differentials Miscellaneous substitutions CHAPTER XXVII INTEGRATION BY PARTS. REDUCTION FORMULAS Formula for integration by parts . Reduction formulas for binomial differentials Reduction formulas for trigonometric differentials To find fe sin nede and fe cos nede CHAPTER XXVIII INTEGRATION A PROCESS OF SUMMATION }. Introduction . . ‘The fundamental theorem of Integrel Celculus . . Analytical proof of the Fundamental Theorem . . Areas of plane curves. Rectangular codrdinates . Area when curve is given in parametric form . Areas of plane curves. Polar codrdinates . . Length of a curve poo . Lengths of plane curves. Rectangular codrdinates . Lengths of plane curves. Polar codrdinates PAGE 825 825 327 329 331 335, 335 336 336 338 338 340 341 843, 345 347 350 356 359 361 361 364 365, 368 370 372 373, 875 7 CONTENTS SECTION 212. 213. 214. 215. 216. 217. 218. 219. 220. 221. 222. 224. 225. 226. 227. 228, 229. 230. 231. 232. Volumes of solids of revolution . Areas of surfaces of revolution . Miscellaneous applications. =... we CHAPTER XXIX SUCCESSIVE AND PARTIAL INTEGRATION Successive integration Partial integration oe Definite double integrai. Geometric interpretation Value of a definite double integral over a region Plane area as a definite double integral, Rectangular codrdinates Plane area as a definite double integral. Polar codrdinates ‘Moment of area. Center of area Moment of inerti: Plane a areas: Polar moment of inertia, Rectangular eodrdinates Polar moment of ine General method for finding the areas of surfaces Volumes found by triple integration . Polar coérdinates CHAPTER XXX ORDINARY DIFFERENTIAL EQUATIONS Differential equations. Order and degree . Solutions of differential equations Verifications of solutions ; Differential equations of the first order and of the first degree . Differential equations of the nth order and of the first degree CHAPTER XXXI xv AGE 3877 381 385, 393, 395, 396 400 402 406 408 408 410 410 411 413 417 421 422 423 424 432 INTEGRAPH. APPROXIMATE INTEGRATION, TABLE OF INTEGRALS 238. 234. 235. 236. 237. 238. 239. 240. 241. ‘Mechanical integration Integral curves . ‘The integraph Polar planimeter A Area swept over by a line . Approximate integration Trapezoidal rule 5 Simpson’s rule (parabolic rule) . Integrals for reference INDEX . . . 443 443, 445 446 446 448 448 449 451 461 DIFFERENTIAL CALCULUS CHAPTER I COLLECTION OF FORMULAS 1, Formulas for reference. For the convenience of the student we give the following list of elementary formulas from Algebra, Geome- try, Trigonometry, and Analytic Geometry. 1, Binomial Theorem (n being a positive integer) : (a4 De = an § nanrb 4 MEX D an -aye 4 MOTOR gp ROAD ABD MATF®) ger ttor a gee, a 808 aes 2 ni=[n=1-2-8-4...(n—1)n : a 3. In the quadratic equation az? + be +¢ = 0, when b? — 4ac > 0, the roots are real and unequal ; when 0 — 4ac = 0, the roots are real and equal ; when 8? — 4ac <0, the roots are imaginary. 4, When a quadratic equation is reduced to the form 2° + px + =0, p= sum of roots with sign changed, and q = product of roots, 5. In an arithmetical series, at(@—1)d; n ae Zt =52utm—1al. 6. Ina geometrical series, rl—a_a(m—1) 1=am-1 = r-1 or—1 1: log. ab = log a + log. 10. log Va = 2 log a. 18. log 2 =— log a. 8. log # = loga—logb. 11. log 1 = 0. 14. Circumference of cirele=2mr. 9. log a" = nlog a. 12. logaa 15. Area of circle = mr’, * In formulas 14-25, r denotes radius, a altitude, B area of base, and s slant height. a to DIFFERENTIAL CALCULUS 16. Volume of prism = Ba. 17. Volume of pyramid = 3 Ba. 18. Volume of right circular cylinder = ma, 19. Lateral surface of right circular cylinder = 20. Total surface of right circular eylinder = 2r(r + a). 21, Volume of right circular cone = } ma, 22, Lateral surface of right circular cone = rs, 23, Total surface of right circular cone = wr(r + 8). 24. Volume of sphere = $ ar’. 25, Surface of sphere = 4 mr’, ra. 1 1 1 26. sinz = ——; cost = ——; tang =—_. wor seca oot ine cos 27. tang 3 cota = 2 cost sing 1 28. sin?z + cos*x =1; 14 tanta = see®a; 1+ cot?x = cscta, 29. sine = cos (Z — s 31. sin(@@ + y) =sinz cosy + cosasiny. 32, sin (e — y) = sin x cosy — cose siny. cons = sin (Z = 5 a 33. cos(t + y) = cose cosy ¥ sinzsiny. tanz = oot (= ). 2 34. tan(z + y) = me titony | : : T—tanetany 30, sin (wz) = sina; cos (# — 2) =— cos; 35, tan(e— y= 2B2— tan tan (w— 2) =— tang. T+ tanztany 36, sin2e = 2Qsinz cos; cos2e= costa — sintz; tan22—= Re, 1— tan?x 37. sing = 2sin¥ cos’; cosa = cost — sin? tans as 38. costa = 4 + }cos2z; sin?e = }— } cos2e, 39. 14 cosa = 2 cost; 1— cosa = asin? 40. sin 5 = eaf—O*) cost = 44 pees tang = 41. FEO AE 42. sin — sin y = 2 cos] ( + y)sin} (a — ). 43. cosx + cosy = 2 c0s}(x + y) cos} (e— y). 44. cosz — cosy =— 2sin} (x + y)sin} (e—y). i — cose 1+ cosa Gi Law of Sines. 46, a? =U + c? — 2be cos A; Law of Cosines, 41. d=V(@,— 2," + WH, — Vo} distance between points (2, 1 and (tq) ¥s)+ An + Buy +0 48. d= te; £VA +B distance from line Az + By + C = 0 to (t,, %)- COLLECTION OF FORMULAS 3 49, 2-0 tt yo Mth 2 2 50. e=a)+2', y= yt Y's transforming to new origin (x, ¥). BL. 2 = 2 cosd— sind, the angle 6 with old. 52. «=p cosé, y =p sind; transforming from rectangular to polar cobrdinates, ; codrdinates of middle point. =a’ sind + f cos; transforming to new axes making 53. p=Vae+y, O=are tan z ; transforming from polar to rectangular codrdinates. 54, Different forms of equation of a straight line: fa) = =e = ‘two-point form; () £441, intercept form; (©) y— y= me — 2), slope-point form; (@) v= me + b, slope-intercept form ; (e) 200s @ + y sin a= p, normal form ; () Ax + By + C= 0, general form, 55. tand = itm? angle between two lines whose slopes are m, and my, ‘m, =m, when lines are parallel, and my=— x when lines are perpendicular. 56. (@ — a)? + (y — 6)? = °, equation of circle with center (a, 8) and radius r, 2. Greek alphabet. Letters Names Letters Names’ Letters Names Aa Alpha dk Tota Pp Rho BB Beta K « Kappa = os Sigma T y Gamma A 2X Lambda Tr Tau A & Delta Mp Mu ev. Upsilon. Ee Epsilon. Nv Nu ® ¢~ Phi ZE Zeta BE Xi KX xy Chi H Eta © 0 — Omicron Vy Psi @ 6 Theta iz bi Q o Omega 3. Rules for signs of the trigonometric functions. Quadrant | Sin Cos ‘Tan Cot See se First + + + + Second | . . = = ao . ‘i Thi - - + + = a DIFFERENTIAL CALCULUS 4, Natural values of the trigonometric functions. Angle in Angle in Smead pet ae Cos tan cot See se 0 ° 0 1 0 » i © 7 90° 1 0 * 0 © al 7 180° 0 ae 0 o a | o oe 270° — 0 - 0 ~ fe 2 Qa 360° 0 a 0 oO . o ‘Angle in | Angle in : - : ceca! pee) cos ‘tan cot See xe 0 oe 0 7 0 o 2 2 x 1 v3 v3 vi 2Vv3 B a = = 3 — 2 6 2 2 3 3 m= va va = 45° v2 v2 1 1 2 2 4 2 2 @ 2 7 vB 1 v3 2v3 e 60° ~ 5 8 aly eney 8 2 2 ~ 3 : 3 = 2g 90° 7 0 co oO o 1 Anglo in | Angle in : - Radians Degrees: bee Goi ee Sed 0000 ° .0000 | 1.0000 | .0000 2 90° | 1.6708 0175, i 0175 +9998 0175 57,290 Bm 1,5533 0349 2 0849 | .9004 | 0349 | 28.636 | 88° | 1.5859 0524 a 0523 | .9086 | 0524 | 19.081 | 87° | 1.5184 0698 4 0698 9976 0699 14,300 86° 1.5010 0873 & 0872 | 9062 | .0875 | 11.480 | 85° | 1.4885 «1745 10° 1736 | 9848 | 1763 | 5.671 | 80° | 1.8968 2618 15° 2588 9659 2679 ‘8.782 15 1.3090 3491 20° 8420, 9897 3640 2.747 70° 1.2217 4368 26° 4226 | .9063 | 4608 | 2.145 | 65° | 1.1345, 5236 30° 5000, 8660 174 1.732, 60° 1.0472 6109 36° 5736 | .8192 | .7002 | 1.428 | 55° 9509 6981 40° 6428 | .7660 | .9301 | 1.192 | 50° 8727 «7854 45° 7071 7071 1.0000 1,000, 45° 7854, : = ‘Angie in | Angle in os sin Cot aon oe | ace COLLECTION OF FORMULAS 5. Logarithms of numbers and trigonometric functions. Taste or Manvissas or THE Common Locaritums or Numbers No. O _ 2 3 4 5 6 7 8 9 1 |] 0000 | 0414 | o792 | 1139 | 1461 || 1761 | 2041 2553 | 2788 2 |] 3010 | 8222 | 8424 | 3617 | 3802 || 3979 | 4150 4472 | 4024 3 |] 4771 | 4914 | 5051 | 5185 | 5816 || 5441 | 5563 5798 | 5911 4 || 6021 | 6128 | 232 | 6335 | 6485 |] 6582 | 628 6812 | 6902 5 |} 990 | 7076 | 7160 | 7243 | 7324 || 7404 | 7482 7634 | 7709 6 || 7782 | 7858 | 7924 | 7993 | 8062 || 8120 | 8195 9325 | 8388 7 || 8451 | 8518 | 8573 | 8638 | 8692 || 8751 | 8808 8921 | 8976 8 || 9031 | 9085 | 9138 | 9191 | 9243 || 9204 | 9345 9445 | 9494 9 || 9542 | 9590 | 9088 | 9685 | 9731 || 9777 | 9823 gg12 | 9956 10 || 0000 “0043 | “0086 | "0128 | 0170 || 0212 | “0253 03384 | 087d 11 |) nit | 0458 | “0492 | “0581 |~0569 || 0607 | 0645 0719 | 0755 12 |] 0792 | 0828 | 0864 | 0899 | 0984 || 0969 | 1004 1072 | 1106 13 || 1139 | 1178 | 1206 | 1239 | 1271 || 1808 | 1885 | 1367 | 1399 | 1430 14 || 1461 | 1492 | 1528 | 1553 | 1584 |} 1614 | 1644 | 1673 | 1703 | 1732 15 || 1761 | 1790 | 1818 | 1847 | 1875 || 1903 | 1981 | 1959 | 1987 | 2014 16 |} 2041 | 2068 | 2095 | 2122 | 2148 || 2175 | 2201 | 2227 | 2253 | 2270 17 || 2804 | 2830 | 2355 | 2880 | 2405 |} 2430 | 2455 | 2480 | 2504 | 2529 18 |] 2558 | 2577 | 2601 | 2625 | 2648 || 2672 | 2695 | 2718 | 2742 | 2765 19 || 2788 | 2810 | 2888 | 2856 | 2878 || 2000 | 2928 | 2945 | 2067 | 2080 Taste or Logarrrams oF THE TRIGONOMETRIC FuN ONS. Angle in | Angle in Angiein [Angletn| ogsin | -togeos | togtan | togeot 0000 oe 0.000 oe eee 90° wos’ |e 9.0000 | s.2i9 | 1.7581 | 89° 0349, - 9.9997 5431 1.4569 ‘88° 10524 | 3° 7104 | 1.2806 | 87° soeos |e seus | 1.1554 | 86° 0873 |e 8.9120 | 1.0580 | 85° 745 | 10 9.2468 | 0.7587 | 80° -2618 | 15° 9.4281 0.5719 TS ssior | 20° 9.5611 | 0.4380 | 70° 4363 25° 9.6687 0.3813 65° 5286 | 30° o.764 | 0.93886 | 60° | 1, 6109 | 35° 9.8152 | 0.1548 | 55° | 0.0509 6981 40? 9.9238 50° 0.8727 -Ta54 | 45° 9.8495 | 0.0000 | 45° | 0.7854 | vegece | tpsin | me.ot | sort [Arete | ante CHAPTER II VARIABLES AND FUNCTIONS 6. Variables and constants, A variable is a quantity to which an unlimited number of values can be assigned. Variables are denoted by the later letters of the alphabet. Thus, in the equation of a straight line, 244-1, ats x and y may be considered as the variable codrdinates of a point: moving along the line. A quantity whose value remains unchanged is called a constant. Numerical or absolute constants retain the same values in all prob- lems, as 2, 5, V7, 7, ete. Arbitrary constants, or parameters, are constants to which any one of an unlimited set of numerical values may be assigned, and they are supposed to have these assigned values throughout the inves- tigation. They are usually denoted by the earlier letters of the alphabet. Thus, for every pair of values arbitrarily assigned to a and 8, the equation e+ $1 represents some particular straight line. 7. Interval of a variable. Very often we confine ourselves to a portion only of the number system. For example, we may restrict our variable so that it shall take on only such values as lie between a and 0, where a and } may be included, or either or both excluded. We shall employ the symbol [a, 6], a being less than 4, to represent the numbers a, }, and all the numbers between them, unless otherwise stated. This symbol [a, 0] is read the interval from a to b. 8. Continuous variation. A variable « is said to vary continuously through an interval [a, 6], when « starts with the value a and increases until it takes on the value 4 in such a manner as to assume the value 6 VARIABLES AND FUNCTIONS 7 of every number between a and 6 in the order of their magnitudes. This may be illustrated geometrically as follows: 2 x b 6 Ta ce B ‘The origin being at O, lay off on the straight line the points 4 and B corresponding to the numbers a and 6. Also let the point P corre- spond to a particular value of the variable 2. Evidently the interval [a, 8] is represented by the segment 4B. Now as z varies continuously from a to 6 inclusive, ie. through the interval [a, 6], the point P gen- erates the segment 4B. 9. Functions. When two variables are so related that the value of the first variable depends on the value of the second variable, then the first variable is said to be a function of the second variable. Nearly all scientific problems deal with quantities and relations of this sort, and in the experiences of everyday life we are con- tinually meeting conditions illustrating the dependence of one quan- tity on another. For instance, the weight a man is able to lift depends on his strength, other things being equal. Similarly, the distance a boy can run may be considered as depending on the time. Or, we may say that the area of a square is a function of the length of a side, and the volume of a sphere is a function of its diameter. 10. Independent and dependent variables. The second variable, to which values may be assigned at pleasure within limits depending on the particular problem, is called the independent variable, or argument ; and the first variable, whose value is determined as soon as the value of the independent variable is fixed, is called the dependent variable, or funetion. Frequently, when we are considering two related variables, it is in our power to fix upon whichever we please as the independent variable ; but having once made the choice, no change of independent variable is allowed without certain precautions and transformations. One quantity (the dependent variable) may be a function of two or more other quantities (the independent variables, or arguments). For example, the cost of cloth is a function of both the quality and quantity ; the area of a triangle is a function of the base and altitude; the volume of a rectangular parallelepiped is a function of its three dimensions, 8 DIFFERENTIAL CALCULUS. 11. Notation of functions. The symbol f(z) is used to denote a function of x, and is read f of x. In order to distinguish between different functions, the prefixed letter is changed, as F(x), $(#), f'(a), ete. During any investigation the same functional symbol always indi- cates the same law of dependence of the function upon the variable. In the simpler cases this law takes the form of a series of analytical operations upon that variable. Hence, in such a case, the same func- tional symbol will indicate the same operations or series of operations, even though applied to different quantities. Thus, if f@=2— 90414, then Also FO+1)=64)- 9641) +14 = 7546, fO)=0—9-0414=14, P-D=C1- 9H + 14 = 34, f3)=8—9-8414=—4, fD=T—9-T+14 = 0, ete. Similarly, $(2, y) denotes a function of z and y, and is read $ of and y. lf o@ya=sin@t+y), then (a, b)=sin(a +), and $(§-0)=sin 5 =1, Again, if Fe ye) =204+8y-122, then F(m, —m, m)= 2m —8m—12 m =—18 m, and FQ, 2%, 1) =2°843-2—12-1=0. Evidently this system of notation may be extended indefinitely. 12. Values of the independent variable for which a function is defined. Consider the functions w@—2xr+5, sing, are tang of the independent variable 2 Denoting the dependent variable in each case by y, we may write y=e—2Qat5, y=sing, y=are tana VARIABLES AND FUNCTIONS 9 In each case y (the value of the function) is known, or, as we say, defined, for all values of 2. This is not by any means true of all functions, as the following examples illustrating the more common exceptions will show. Oy i Here the value of y (i.e. the function) is defined for all values of x except 2=, When 2=6 the divisor becomes zero and the value of y cannot be computed from (1).* Any value might be assigned to the function for this value of the argument. Q y=va. In this case the function is defined only for positive values of 2. Negative values, of x give imaginary values for y, and these must be excluded here, where we are confining ourselves to real numbers only. (8) y=log,e. a>0 Here y is defined only for positive values of z. For negative values of x this function does not exist (see § 19). a (4) y=aresina, y =are cosa. Since sines and cosines cannot become greater than +1 nor less than —1, it follows that the above functions are defined for all values of ranging from —1 to + 1 inclusive, but for no other values. EXAMPLES 1. Given f(2) = 2* — 102? + 812 — 80; show that F(0) =— 380, SW) =y — 10? + 81y —30, FQ) =0, F(a) = a — 10a + 81a — 30, FQ=fH), S (yz) = P28 — 102? + 81 yz — 80, FU)>S(— 8), S@— 2) = 29 — 1622 + 832 — 140, S(-1)=- 6F(6). » . It f(a) = 29 — 8x + 2, find £(0), £1), F(— 1), F(— 4), FAY)- 8 1022 + 81a — 30, and (x) = x4 — 55a? — 2102 — 216, show that (—2), F@)=4(— 8), FG) = o(— 4), FO) + (0) + 246 =0. . If F(a) = 2, find F(0), F(— 8), FQ), F(-2). ad op . Given F(z) = 2 (e — 1) ( + 6) (2 — 3) (@ + 9); show that F()= FQ) =F(-0)=FQ)=F(-) =0. * See § 14, p12. 10 = - 10. 11. 12. DIFFERENTIAL CALCULUS =1 my : TE Fm) = Fz show that F(mry) =F (Mg) _ Mm — My L+F(m)F (mq) 1+ mymy . If p(z) =a, show that ¢ (y)-@ (2) =o +2)- 1= 2, show that T¥e oe tom =o (224). Given ¢ (2) = log: . If f(g) = cos g, show that F(8)=S(— 9) =—-S(@ — 9) =F (FF 4). If F(6) = tan 8, show that Fe6) = 27 (2) 1=TFOF Given y (x) = 2 4 2™ 41; show that ¥0)=38, ¥O=1, ¥@=¥(-4%). It f@)= =e o find f(V3). Ans. —.0204, CHAPTER III THEORY OF LIMITS 13, Limit of a variable. If a variable v takes on successively a seriés of values that approach nearer and nearer to a constant value J in such a manner that |» — J|* becomes and remains less than any assigned arbi- trarily small positive quantity, then v is said to approach the limit 1, ov to converge to the limit l. Symbolically this is written limit v= 1, or, v=. The following familiar examples illustrate what is meant: (1) As the number of sides of a regular inscribed polygon is indefi- nitely increased, the limit of the area of the polygon is the area of the circle. In this case the variable is always less than its limit. (2) Similarly, the limit of the area of the circumscribed polygon is also the area of the circle, but now the variable is always greater than tts Limit. (3) Consider the series Hoa (4) Tease The sum of any even number (27) of the first terms of this series is 4 tt al al: 1 Syal—5+q—gt tg ot 2 1 (B) Su= 53 By 6, p.1 Similarly, the sum of any odd number (2”-+1) of the first terms of the series is 111 1061 Smu=l—gt+g—gts— gat ge oT gen Al ©) gay eo By 6, p.1 # To be read the numerical value of the difference between v and l. 12 DIFFERENTIAL CALCULUS Writing (B) and (@) in the forms 2 1 F- Seager Sen Gay limit (2 _ limit 1 we have : " G-s.)=,) reaeh end limit (s. 3)= limit 1 n=0 3) "asad an +1 3, Hence, by definition of the limit of a variable, it is seen that both: S,, and S;,4, are variables approaching 2 as a limit as the number of terms increases without limit. Summing up the first two, three, four, etc., terms of (4), the sums are found by (B) and (C) to be alternately less and greater than 3, illustrating the case when the variable, in this case the sum of the terms of (A), is alternately less and greater than its limit. In the examples shown the variable never reaches its limit. This is not by any means always the case, for from the definition of the témit of a variable it is clear that the essence of the definition is simply that the numerical value of the difference between the variable and its limit shall ultimately become and remain less than any positive number we may choose, however small. (4) As an example illustrating the fact that the variable may reach its limit, consider the following. Let,a series of regular polygons be inscribed in a circle, the number of sides increasing indefinitely. Choosing any one of these, construct the circumscribed polygon whose sides touch the circle at the vertices of the inscribed polygon. Let p, and B be the perimeters of the inscribed and circumscribed polygons of n sides, and C the circumference of the circle, and sup- pose the values of a variable x to be as follows: pe Cb Pee CO; hee el. Then, evidently, and the limit is reached by the variable, every third value of the variable being C. i 14. Division by zero excluded. 0 is indeterminate. For the quotient of two numbers is that number which multiplied by the divisor will give the dividend. But any number whatever multiplied by zero gives THEORY OF LIMITS 13 zero, and the quotient is indeterminate; that is, any number whatever may be considered as the quotient, a result which is of no value. a . oa: . ‘ & has no meaning, a being different from zero, for there exists no number such that if it be multiplied by zero, the product will equal a. Therefore division by zero is not an admissible operation. Care should be taken not to divide by zero inadvertently, ‘The following fallacy is an illustration. Assume that ‘Then evidently Subtracting 0%, Factoring, Dividing by a —b, But therefore or, ‘The result is absurd, and is caused by the fact that we di ed by a 0. 15. Infinitesimals. A variable » whose limit is zero is called an infinitesimal.” ‘This is written limit» = 0, or, v= 0, and means that the successive numerical values of v ultimately become and remain less than any positive number however small. Such a variable is said to become indefinitely small or to ultimately vanish. It limit v = 1, then limit (» — = that is, the difference between a variable and its limit is an infinitesimal. Conversely, if the difference between a variable and a constant is an infinitesimal, then the variable approaches the constant as a limit. 16. The concept of infinity (co). If a variable v ultimately becomes and remains greater than any assigned positive number however large, we say v increases without limit, and write limit v=+ 00, or, v=+o. If a variable v ultimately becomes and remains algebraically less than any assigned negative number, we say v decreases without limit, oud ate limit» =— 0, or, » ‘*Hence a constant, no matter how small it may be, is not an infinitesimal, 14 DIFFERENTIAL CALCULUS If a variable v ultimately becomes and remains in numerical value greater than any assigned positive number however large, we say v, in numerical value, increases without limit, or v becomes infinitely great,* and write limit v= 00, or, v= 00. Infinity (0) is not a number; it simply serves to characterize a particular mode of variation of a variable by virtue of which it increases or decreases without limit. 17. Limiting value of a function. Given a function f(z). If the independent variable z takes on any series of values such that limit «= and at the same time the dependent variable f(x) takes on a series of corresponding values such that limit f(z) =4, then as a single statement this is written lim 44) = and is read the limit of f(), as x approaches the limit a in any manner, is A, 18. Continuous and discontinuous functions. A function f(2) is said to be continuous for « =a if the limiting value of the function when approaches the limit a in any manner is the value assigned to the function for z=a. In symbols, if wat f@) =F@), then f(2) is continuous for x =a. The function is said to be discontinuous for x =a if this condition is not satisfied. For example, if Limit 72) = 0 the function is discontinuous for 2= a. ‘The attention of the student is now called to the following cases which occur frequently. *On account of the notation used and for the sake of uniformity, the expression v 4+ is sometimes read » approaches the limit plus infinity. Similarly, v += is read v approaches the limit minus infinity, and + is read v, in numerical value, approaches the limit infinity. ‘While the above notation is convenient to use in this connection, the student must not forget that infinity is not a limit in the sense in which we defined a limit on p. 11, for infinity is not a number at all. THEORY OF LIMITS 15 Case I. As an example illustrating a simple case of a function con- tinuous for a particular value of the variable, consider the function x @=5 For ¢=1, f(z) =f(1)=8. Moreover, if x approaches the limit 1 in any manner, the function f(e) approaches 8 as a limit. Hence the function is continuous for 2 =1. Case II. The definition of a continuous function assumes that the function is already defined for 2=a. If this is not the case, how- ever, it is sometimes possible to assign such a value to the function for z=a that the condition of continuity shall be satisfied. The following theorem covers these cases. Theorem. Lf f(x) ts not defined for x= a, and ' wr FC) =B, then f(2) will be continuous for x =a, if B is assumed as the value of f(@) for x=a, Thus the function v4 z—2 is not defined for 2 = 2 (since then there would be division by zero). But for every other value of 2, limit and pet D=4; limit 2°— 4 therefore ime Although the function is not defined for z= 2, if we arbitrarily assign it the value 4 for z= 2, it then becomes continuous for this value. A function f(2) is said to be continuous in an interval when it is continuous for all values of x in this interval.* ‘In this book we shall deal only with functions which are in general continuous, that is, continuous for all values of , with the possible exception of certain isolated values, our results in general being understood as valid only for such values of x for which the function in question is actually continuous. Unless special attention is called thereto, we shall as a rule pay no attention to the possibilities of such exceptional values of for which the function is discontinuous. ‘The definition of a continuous funetion f(z) is sometimes roughly (but imperfectly) summed up in the statement that @ sinall change in x shall produce « small change in f(z). We shall not consider funetions having an infinite number of oscillations in a limited region. r : 16 DIFFERENTIAL CALCULUS 19. Continuity and discontinuity of functions illustrated by their graphs. (1) Consider the function 2’, and let “4 yar. If we assume values for 2 and calculate the corresponding values of y, we can plot a series of points. Drawing a smooth line free-hand [ through these points. a good representation of the gen- cral behavior of the unction may be obtained. ‘This picture or image of the function is called its graph. It is evidently the locus of all points satisfying equation (A). Such a series or assemblage of points is also called acurve, Evidently we may assume values of x so near together as to bring the values of y (and therefore the points of the curve) as near together as we please. In other words, there are no breaks in the curve, and the function 2” is continuous for all values of 2. (2) The graph of the continuous function sinz is plotted by draw- ing the locus of 0 x . |O x y=sina. It is seen that no break in the curve occurs anywhere. (3) The continuous function e is of very frequent occurrence in the Calculus. If we plot its graph from y=e, (¢=2.718--) ¥ we get a smooth curve as shown. From this it is clearly seen that, 1 (a) when x= 0, Jmlt y=) =1; a (b) when 2>0, y(=c*) is positive and increases as we pass towards the right from the origin; (ce) when 2<0, y(=@) is still positive and decreases as we pass towards the left from the origin. 7 (4) The function log, is closely related to the last one discussed. In fact, if we plot its graph | x from oie it will be seen that its graph has the same rela- tion to OX and OY as the graph of e* has to OY and OX. THEORY OF LIMITS 17 Here we see the following facts pictured : (a) For #=1, log.« = log,1= 0. (b) For 2 >1, log, is positive and increases as « increases, (c) For 1>2>0, log, is negative and increases in numerical value as x diminishes, that is, !™* log 2 =— 0. (A) For x=, log.z is not defined ; hence the entire graph lies to the right of OY. hg (5) Consider the function 4, and set = If the graph of this function be plotted, it will be seen that as x approaches the value zero from the left (negatively), the points of the curve ultimately drop down an infinitely great distance, and as 2 approaches the value zero from the right, the curve extends upward infinitely far. The curve then does not form a continuous branch from one side to the other of the axis of ¥, showing graphically that the function is discontinuous for z = 0, but continuous for all other values of 2. (6) From the graph of _ 2 7 it is seen that the function Qa 1-2 is discontinuous for the two values 241, but continuous for all other values of 2. (7) The graph of yestane shows that the function tana is dis- continuous for infinitely many values of the independent variable z, namely, z=", where n denotes any odd positive or negative integer, (8) The function are tan & Bb DIFFERENTIAL CALCULUS has infinitely many values for a given value of 2, the graph of equation y= are tana consisting of infinitely many branches. If, however, we confine our- selves to any single branch, the function is continuous. For instance, if we say that y shall be the are of smallest numeri- cal value whose tangent is 2, that is, y shall take 2 limited to the branch passing through the origin, and the condition for continuity is satisfied. (9) Similarly, on only values between —7 and 7, then we are are tan, 2 is found to be a many-valued function. Confining ourselves to one branch of the graph of ye tan , we see that as 2 approaches zero from the left, y approaches the limit — Z, and as approaches zero from the right, y approaches the 2 PP ight, y apy limit + z Hence the function is discon- tinuous when 2=0. Its value for 2=0 can be assigned at pleasure. Functions exist which are discontinuous for every value of the independent vari- able within a certain range. In the ordinary applications of the Cal- culus, however, we deal with functions which are discontinuous (if at all) only for certain isolated values of the independent variable s such functions are therefore in general continuous, and are the only ones considered in this book. 20, Fundamental theorems on limits. In problems involving limits the use of one or more of the following theorems is usually implied. It is assumed that the limit of each variable exists and is finite. Theorem I. The limit of the algebraic sum of a finite number of vari- ables is equal to the like algebraic sum of the limits of the several variables. Theorem II. The limit of the product of a finite number of variables ts equal to the product of the limits of the several variables. Theorem III. The limit of the quotient of two variables is equal to the quotient of the limits of the separate variables, provided the limit of the denominator is not zero. THEORY OF LIMITS 1 Before proving these theorems it is necessary to establish the fol lowing properties of infinitesimals. 1) The sum of a finite number of infinitesimals ts an infinitesimal. ‘To prove this we must show that the numerical value of this sum can be made Jess than any small positive quantity (as e) that may be igned (§ 15). That this is possible is evident, for, the limit of each infinitesimal being zero, each one can be made numerically less than £ (nm being the number of infinitesimals), and therefore their sum can n be made numerically less than e (2) The product of a constant ¢ and an infinitesimal is an infinitesimal. For the numerical value of the product can always be made less than any small positive quantity (ase) by making the numerical value of the infinitesimal Jess than £. e (3) The product of any finite number of infinitesimals és an infinitesimal. For the numerical yalue of the product may be made less than any small positive quantity that can be assigned. If the given product contains » factors, then since each infinitesimal may be assumed less than the xth root of ¢, the product can be made less than e itself. (4 Ifo ts a variable which approaches a limit l different from zero, then the quotient of an infinitesimal by v is also an infinitesimal. For if limit » = J, and & is any number numerically less than J, then, by defini- tion of a limit, v will ultimately become and remain numerically greater than & Hence the quotient £, where ¢ is an infinitesimal, will ulti- 0 mately become and remain numerically less than 4, and is therefore by (2) an infinitesimal. Proof of Theorem I. Let vy ry Uy +++ be the variables, and Uy 2 ly «=> their respective limits. We may then write ly where ey € limit)? Adding ++ are infinitesimals (ie. variables having zero for a (A) OF yt BA DH GHGt ht =Car ett). 20 DIFFERENTIAL CALCULUS Since the right-hand member is an infinitesimal by (1), p. 19, we have, from the converse theorem on p. 18, limit 2, + 4,44: =4ththt--5 or, limit (v, + v, + ¥,+ -+-) = limit v, + limit v, + limit o,+---, which was to be proved. Proof of Theorem II. Let v, and v, be the variables, 2, and 2, their respective limits, and ¢, and ¢, infinitesimals; then wy=lte, . and Malt ey Multiplying, 2,= + &) at &) = Ut het bet ee or, (B) v,04— Uly= Le, + ye, + €6y+ Since the right-hand member is an infinitesimal by (1) and (2), p. 19, we have, as before, limit (0,»,) = U1, = limit », - limit », which was to be proved. Proof of Theorem IIE. Using the same notation as before, % 4+) (t8—2) » Lte L' Mite ly or, ©) % 4 lee % lL LG+s) Here again the right-hand member is an infinitesimal by (4), p. 19, if 1, #0; hence io 1, __ limit v, limit (-*)=7'=j55° », i, limit », which was to be proved. It is evident that if any of the variables be replaced by constants, our reasoning still holds, and the above theorems are true. 21. Special limiting values. The following examples are ofspecial importance in the study of the Calculus. In the following examples a>O and ¢#0 THEORY OF LIMITS 21 Written in the form of limits. Abbreviated form often used. wm limit & 23 i oS @® c-o= 0, 3 2, @ 2 = 0, e 4 eam 6) when a <1; a-*=+0, (6) when a <1; ate =0, ~ when a>1; a7=0. (8) when a>1; Geo. 9 log,2=+ 00, when a <1; log,0 =+ Ba Ba (lo) _ log,2=— oo, when a<1; log, (+ 0)=—a (11) Hm loge =— 00, when a>1; log,0 =— oo. (12) , Nt log, =+ 0, when a>1; — log,(+.0)=-+o0. ‘The expressions in the second column are not to be considered as expressing numerical equalities (co not being a number); they are merely symbolical equations implying the relations indicated in the first column, and should be so understood. limit sin x _ x=0 Let 0 be the center of a circle whose radius is unity. Let are AM = are AM' =a, and let UT and M'L be tangents drawn to the cirele at Mand M'. From Geometry, MPM! < MAM' < MIM’; a : 2 si 2 or Qsine ) a & 0 Taking the logarithm of both sides to the base e, we get log y =» log a, or, v aot dy Toga y? and from (C), § 48, relating to inverse functions, we get @ or, cs Ties (4) tog aa’ Since v is a function of « and it is required to differentiate a” with respect to x, we must use formula (A), § 42, for differentiating a function of a function, namely, dy _dy do dx dv dz Substituting the value of y from (A), we get a . =loga-a d dv i oS (a) = log a- av 2. FO = toga aS When a=e, ies and IX becomes dv 1X =(e)=e=. @ Jc Daer The derivative of a constant with a variable exponent is equal to the product of the natural logarithm of the constant, the constant with the variable exponent, and the derivative of the exponent. RULES FOR DIFFERENTIATING 49 46. Differentiation of the general exponential function. Let yous Taking the logarithm of both sides to the base e, log.y = v log.u, or, y=erey, Differentiating by formula Ixa, ym 4 log u) =w (22+ logu). x _ Zw = 00 B 4 tog uw. The derivative of a function with a variable exponent is equal to the sum of the two results obtained by first differentiating by VI, regarding the exponent as constant; and again differentiating by IX, regarding the function as constant. Let » =n, any constant; then X reduces to Cau a“ Qt) = nv ae But this is the form differentiated in § 40; therefore VI holds true for any value of n. Iususreanive Examene 1, Differentiate y = log (2? + a). a < (a? +a) dy _ a, Solution. ao Fra by VIIa =at+a) = 2 Ans, eta Inpusrrative Examrre 2. Differentiate y = log VI— aa. d tt ~at + Solution, 2 by VIIa ae at at a=) F(- 20) by VI a-ayt 2 Ans. * wcan here assume only positive values. 50 DIFFERENTIAL CALCULUS In.usrrative Exampve 8. Differentiate y = a=, Solution, YW —roga-02 4 gx) by IX = 6xloga-at, Ans. Intusrrarive Exampe 4, Differentiate y = be?+2*. ay _ Solution. lution, a a b— (ee? +2) Jet) by IV betes (et + at) by Xu = 2beet+s", Ans. Intusrative Examere 5, Differentiate y = x, Solution, u = eet é (0) + 24 loge: é e by X = ent + we loge e 7 exe( o loge). Ans. 47. Logarithmic differentiation. Instead of applying VIII and VIIa at once in differentiating logarithmic functions, we may sometimes simplify the work by first making use of one of the formulas 7-10 on p. 1. Thus above Illustrative Example 2 may be solved as follows: Ittusrrative Exampre 1, Differentiate y = log V1— 2. Solution, By using 10, p. 1, we may write this in a form free from radicals as follows : y= }log(1— 2), a fay ‘Then wie = by VIIa 1 —2¢ - + Ans 2 1-2 feat Inusrrative Exampre 2, Differentiate y = log Solution, Simplifying by means of 10 and 8, p. 1, y= 4 [log (1 + 2%) — log (1 — 2%). a a uy fae ay weet? gt?) by Vitra, ee de 2,142? T—a ’ x 22 = =e. ai i¢e@*i-e-i-aw “™ RULES FOR DIFFERENTIATING 51 In differentiating an exponential function, especially a variable with a variable exponent, the best plan is first to take the logarithm of the function and then differentiate. Thus Illustrative Example 5, p- 50, is solved more elegantly as follows: sar, Solution, ‘Taking the logarithm of both sides, Intusrrative Exampre 8. Differentiate logy = et loge. By 9, p.1 Now differentiate both sides with respect to z. wy S =e 2 (log) + loge t oy by VIII and V aot 4 loge-er, sed + loge. e, dy 1 Wo ge iy(L a « ees) =en(E+ Ing). Am Iniusrrative Exampre 4, Differentiate y = (42? — 7)2+-V#—6, Solution. ‘Taking the logarithm of both sides, logy = (2 +-V2" = 5)log (422 — 7). Differentiating both sides with respect to 2, cae a 5) 8% (422 — z= yatta 5) ge qt los de Ve Y _ (a2 — y+ vans [8A +VE = 8) , loeP= 1] Ans, a te ans In the case of a function consisting of a number of factors it i times convenient to take the logarithm before differentiating. Ituustrative Exampre 5. Differentiate Solution, Taking the logarithm of both sides, logy = 3 [log (@ — 1) + log (x — 2) — log (z — 8) — log (x — 4)]. Differentiating both sides with respect to 2, a 2-3 4] 227-102411 @-Ne@-HE-He—4" 227-102—11 . @-pte—a!@-ai@—4ai or, Ans. 52 DIFFERENTIAL CALCULUS EXAMPLES Differentiate the following : = loge + a). 2. y= log (ax +b). 4. y = log(a? +2). 5. y= log (3 — 22 + 5). 6. y = loga (2a +2). 7. y=aloge, 8. f(z) = loge’. 9. f(z) = log*z. Hinr. logte = (log2)®, Use first VI, »=loge, n=3; and then VIIa. 10. $@) = log +2. ra =—24 1. f(@) = loge + VIF a). F(t) = : vit¢e 12, © cox = aes, a7, Leitso = aaa, dz dx 13, © arte a4 etess, 1s, 4 ato = 1aree toga. dz ao 6 14, 2 ase = 8087 loga, 19, Lo =2slogh-b*. dx ds a 15. “tog (8 — 28) = 20. aos ¢ ) @ lt 16. “to = al. ay my we 22, ya Tt t2e, Y= 2log7- (a + 1) 7 +24, 23, y= et-2, 2eloge-c# ==, 24. y = log : ae ie +e 25. Stea-2] =e(1-22—-2%, d (#1 Qer a ae et . teeny = ner (ar + 2). °6 alan) @+m ne RULES FOR DIFFERENTIATING 53 Yo = @2"~1 (n+ zloga) y =a (loge +1). 1 2 (1~loga) | 32, yaar. : : z 33, y = alex, of =loga?. alosz—1, 34. f(y) =logy- ev. rwe=e (sy +3 : _ logs vg) _ 1— Slogs 35, f(0) = MES. Sy = ashe, oe 36. f(z) =log (log). IO) = say" vig) _ 40g? (log 2) ', F(z) = log* (lc le F’ _—_ 31. F(@) = log" (og 2). te) = See . (t) = log (logt a). (oo 38, 6 (2) = log (lost) +O = aes Hlogaftt¥, z _ 89. ¥(y) = log 7 vy) i-# po echteee Vet +2 Hint. First rationalize the denominator. 1 40. f(0) = Io I@= 41. y = ae, dy ya, WW _ (1 4 loga) 2%. 42. y= e*. a (1+ log 2) a: ct dy ‘e\= c 43. y=. &_ (2¥ (iogS —1). Sue a @) (0 z ) n\n ae 2) =(-)- —=n(- 14 log’ sv-() eon G(s 45. w= 0. == ve (Ene), dv v a\ ad _ [ay =(-). =—=(- —logt—1). 46. 2 () & () (log a — logt 1). 41. yam". $= ast n-1 (loge +1). : 1 48, y=0%. $= te (Ioge + lout +2)- dy syloga 54 DIFFERENTIAL CALCULUS 50, Differentiate the following functions : @) ge logz. © Z elogz, () Eos (+H (b) ce 1. () ge 8, Q Ee, (a? + 52). 0) zg oe @ gee vies. @) ce Fayette, o£ © Letra @+1? dy @ +I (Sat 14245) bly “Ermer ae @ +2) @ +3) Him. Take logarithm of both sides before differentiating in this and the following examples. 8. y= w= dy __ @=1)8 (722 + 802 — 97) (e—28@—9)F de 12(2—2)f@@—8)¥ 63. y=a2Vi—a(l+2). wu 24+a—528 (+ 2%) | dy Sa 55. y= 28 (a + 82)* (a— 22)%, ba, y= a-at 5a (a + 82)? (a— 22) (a? + 2ar—122%), z 48. Differentiation of sin v. Let y=sin v By: General Rule, p. 29, considering v as the independent variable, we have Fist Srer. y + Ay =sin(v +Av). Seconp Step. Ay =sin(v + Av) —sin o* Av\ Av =2 fons t = 2 (o+ 8) int If we take the, third and fourth steps without transforming the right-hand member, there results : A Ay _sin (v + Av)—sin » | Third step. cd 7. ‘Av ‘av Fourth step. te jr Which is indeterminate (see footnote, p. 46). tLet Anvtav A= vedo and Bao Bao Adding, IFB=Tt Subtracting, 4-B= Av 1 Av 1 Therefore 4(4+B)=04 82. dua- mya erefore F(A +B) = v4 pd Be S Substituting these values of 4, B, }(4+B),}(4—B) in terms ofv and Av in the formula, from Trigonometry (42, p. 2), sin A—sin B=2cos 4(4 4B) sin 4 (4-B), we get sin (»-+ Ar) sin v= 2 608 (v+ 42) sin S2- RULES FOR DIFFERENTIATING 55 A s sin Se Taro Srer. SY cos (+2) x=} 2 Fourtn Srep. dy =cos vw dy rimie (202 sine ml ( 2 Since v is a function of 2 and it is required to differentiate sin v with respect to a, we must use formula (A), § 42, for differentiating a function of a function, namely, mt by $20.2 and Bthee(oot)-or] dy _ yy we, ae du de Substituting value wy from Fourth Step, we get dy_ dv a =cosv a I oe Lin v= cos 0. The statement of the corresponding rules will now be left to the student. 49. Differentiation of cos v. Let y = cose By 29, p. 2, this may be written in (5-*) Differentiating by formula XI, 4 — on (§—») 4 (5-2) ao 2) alo emf) 2 a, =—sine®. te [sinc cos (Ze) asin, y 9.0.2] mm “ Zoos ») =—sin of, 56 DIFFERENTIAL CALCULUS 50. Differentiation of tan v. Let. y=tany. By 27, p. 2, this may be written sine "cosy Differentiating by formula VII, a cos 0 5- (sin ») — sin» 5 S (eos ») | da cos*y dvs dv costy 2 . os” or ry Fain v qe - cos*v dv dx a, dv =a ees d dv XU .. (tan v) = sec?v—- a z ax 51. Differentiation of cot v. Let y=cotv. By 26, p. 2, this may be written tan Differentiating by formula VII, i 4 (tan v) az tanty sec?y — =——— =~ osc S. tan?e oh d dv xm ws = (cot v) =—csctv—- V gloat n) = — eset 52. Differentiation of sec v. Let y=secr. By 26, p. 2, this may be written cosy RULES FOR DIFFERENTIATING 57 Differentiating by formula VI, ad = (cos 0 4 ee dx cos*v sino @ agents cos*e _ 1 sinv dv cos v cos v dx = secv tan o®. dx da dv a. = (secv) = secv tanv—- wv ae ee) 53. Differentiation of csc v. Let y=osen By 26, p. 2, this may be written a - Y= Sno Differentiating by formula Vit, _ 4 Ging a Gis dcx sin*v dv cos o dv =— escv cotv——- da d a w. = (escv) = — cscv cotv—-- a Frat) a 54, Differentiation of vers v. Let. y = vers v. By Trigonometry this may be written y=1—cose 58 DIFFERENTIAL CALCULUS Differentiating, an ae +. 4 (versy) = sinv® Xvi =. ag = ae In the derivation of our formulas so far it has been necessary to apply the General Rule, p. 29 (ie. the four steps), only for the following: a du, dv dw ' WL Qe o—wy= T+ 2-2. Algebraic sum, a dy, du Vv Fw) =uZ4 0H. Product ot _ 2 a [uw _ "ae “ae . vil Z()= FI cuotient a a de . vir Fog.) = loge =. Logarithm, a. a . xI EZ Gino) = cos 02. Sine. dy _ ay de, XXV Bowe Function of a funetion. XXVI ao Inverse functions. de te dy Not only do all the other formulas we have deduced depend on these, but all we shall deduce hereafter depend on them as well. Hence it follows that the derivation of the fundamental formulas for differentiation involves the calculation of only two limits of any difficulty, viz., limit sin» v=0 0 by § 22, p. 21 and Jimit 44 9)" By § 23, p. 22 RULES FOR DIFFERENTIATING EXAMPLES Differentiate the following : 1, y=sinaz®, ay oc Fe 7 C8 ae F(a) (v= az] = 2 az cos az. 2 y=tanvi—a. = sect Vina a_ at or [v= vi-z.] = see? VI—#- (1a) (1) sec? VI— 2vi-« 3. y= costs, ‘This may also be written y= cosa)’, dy a ve 2 3 (cos)? — (cosa) [v= cosa and n=3.] Scostz(—sinz) — 8sinz costa, 4. y=sinnesinnz, dy _ sin me (sina) + sinne (sin na) (w= sinnz and v= sint2] 59 by XI by XIII by VI by XII = sinne-n(singye-12 (sina) + sine cosne © (nt) by VI and XI =nsinne-sint-12 cost + nsinz cos nt nsint—1z (sin ne cose + cos nz sin) nsin"—tzsin(n + 1). 5. y =secan, Ans. {Y= a secas tan av. 6. y = tan (ax +b). 2 asec (ax +0). 1. 8 = cos8az. Ba sasindax. | 8. 8 = cot(2f +8). s —Atesc?(2t +8). 9. f(y) =sin2y cosy. 14. r= acos26. S'(y) = 2cos2y cosy — sin 2y siny. F(t) =— 10 cot 5x esc? 52. F/(6) = tan2g. 03g. in?t (8 cost — sin*t). 1 \ | I i 60 DIFFERENTIAL CALCULUS a a oa o 15, Zsinte = sin2e, 23. Z cont = Erin‘. d ; Cee 2 a. 16. £ costa? =— 6 costat sinat, aa, Fain 5 = — F008 5s 2 2 og at. 2 es0% = F ot 2 25, Lesinz = esinz cos. a 18, 2 aVeosds =— 28028, 26, 4 sin(logay = £80984), ds cos 28 dz © oe = a _ sec® log2) 19, FG a(l — e088) = 8. 27. 7, tan (log) ==—TES. a ao 68 . © (log cos) =— tan. . © asin? 2 = asin? 2 cos’ 20. 7, (log cos) =— tana, 28, asin? 5 = asin? cose aa. © (og tana) : 29, © sin (cos a) =— sin 008(c08 @) a dtr ~ sine * da . Co dtane-1_ |, 22. (og sin?z) = 2 cots. 30. nang = Sine + cosa as az ~ cose = ae a 9% y= Jogtan( +5) de cose 83. f(z) = sin (# + a) cos (x — a). I’ (@) = cos 2a. 34, y= alana, natenns sec? na log a. y = @m= (cosa — sin?2). (cot + log sinz). 35. y = ese sing. 36. = et log sing, 87. Differentiate the following functions: (@) Z sin 5a? @ é ese (log). e) g ea beort, (b) Ecos a — ta). ) awsae, @ g jeoste (O) cian. (n) Loos! (loga). (m) Foor d. @ Z cot Vaz. i) Stant vi (a) BVI cor. @ Z seced=, Oe log (sin? az). (0) E tog VI= Binks 38. é (anesinay ar-1etinz (n 4 2 082). 39. ce cosmz) = e# (1 cos me — msin mz). _ 14 0088 oo aan Oe coad TO=— Green di 41. Ig) = Plese= 89), F'($) = ent sing. 42. f(s) = (scots)?. F'(8) = 2s cots (cots — s cscs). RULES FOR DIFFERENTIATING 61 43, r=} tan®O— tand +8. g- ant 6. = ong, wy ine (Sat ). 44, y = ang, = ans (SRE + loge cos 45, y = (sin2)= y' = (inz)= [log sinz + zcotz). 46. y = (sina)nz, * y= (sin z)"02(1 + sec? log sine) a 47. Prove — rove 5 cos =- sino®, using the General Rule. 48, Prove Zcotv =- eset by replacing cot » by = 55. Differentiation of arc sin v. Let, y =are sin v;* then v=siny. Differentiating with respect to y by M1, oe ay 83 dy_ 1 h yi it , pe Al therefore ueoney By (C), p. 46 But since v is a function of 2, this may be substituted in (A), p. 45 giving 1 ad Vi-vde [x yaVizsinty =VI= a, the positive sign of the radical betng | singe cosy is positive for all values of y between — 3 and inclusive, a ‘ XVI a. (are sin v) vi _ ) vi-v * Tt should be remembered that this fnnetion is defined only for values of » between —1 and +1 inclusive and that y (the function) is many-valued, there being infinitely many ares whose sines all equal v. ‘Thus, in the figure (the locus of y=are sin v), when ¥= OM, y= MP1, MPa, MPs,-~, MQuy, MQoy-++- In the above discussion, in order to make the funetion single-valued, only values of y between — and 7 inclusive (points on are QOP) are considered ; a i ees 62 DIFFERENTIAL CALCULUS 56. Differentiation of arc cos v. Let y =are cos 0; * then v= cosy. Differentiating with respect to y by XI, dv . erty therefore # ay By (€), p. 46 But since v is a function of 2, this may be substituted in the formula (A), p. 45 giving XIX = (are cos v) = dx 1-¥ 57. Differentiation of arc tan v, Let y=are tan v;t then v=tany. Differentiating with respect to y by XIV, ® sect ys dy ys dy a therefore S¥ ag By (€), p. 46 * This function is defined only for values of » between ~1 and +1 inclu- sive, and is many-valued. In the figure (the locus of y= arc cos), when v=OM, y= MP3, MPs,+**, MQq, M Qa, In order to make the function single-valued, only values of y between © and ® inclusive are considered; that is, the smallest positive are whose cosine is 2. Hence we confine ourselves to are QP of the graph. 4 This function is defined for all values of », and is many- valued, as is clearly shown by its graph. Tn order to make it, single-valued, only values of y between ~ 5 sidered; that is, the are of smallest numerical value whose tangent is v (branch 408). © and 5 are con- RULES FOR DIFFERENTIATING 63 But since v is a function of 2, this may be substituted in the formula di dy dv Z 2.2, (A), p. 45 giving yt dz secy dz 1d 1+ dr [seoty =14 tanty=14 v4] dv d dx Sa tan v) =——- wx gear tan) = 58. Differentiation of arc cot v.* Following the method of the last section, we get dv : d a XX (are cot v) = ———. a » 14+v 59. Differentiation of arc sec v. Let y =are sec v3t then vasecy. * ‘This funetion is defined for all values of », and is many-valued,-as is seen from its graph (Fig. a). In order to make it single-valued, only values of y between 0 and m are considered; that is, the smallest positive are whose cotangent is v. Hence we confine our selves to branch 4B. Fie.a Fie.d } This function is defined for all values of » except those lying between ~1 and +1, and is seen to be many-valued. ‘To make the function single-valued, y is taken as the are of smallest. numerical value whose secant is v. ‘This means that if v is positive, we confine ourselves to points on are AB (Fig. b), y taking on values between 0 and 5 (Omay be included) ; and if v is negative, we confine ourselves to points on are DC, y taking on values between ~ and - 5 (Cm may be included). 64 DIFFERENTIAL CALCULUS Differentiating with respect to y by XV, Fiasco y tan ys dy 1 therefore By (C), p. 46 dy secytany But since v is a function of 2, this may be substituted in the formula dy_dy d SY YS, ‘A), p. 45 de dv de oe ivi, ao giving de secy tany dx =a oVe—l de eye ad any = Seecty sed ha plas ian or a radieal being taken, since tan y is positive for all values of y dotweon Oand and between — wand —F including and —7. 2 beets oe (are sec v) = bad "de vv a1 60. Differentiation of arc csc v.* Let y =are csc v3 then v=cscy. Differentiating with respect to y by XVI and following the method of the last section, we get gle d AMIE = — (are csc v) = - ———- a 2 vv 1 = __* This function is defined for all values of v except those lying between — 1 and +1, and is seen to be many-valued. To make the function single-valued, y is taken as the are of small- ‘est numerical value whose cosecant is v. ‘This means that if » is positive, we confine ourselves to points on the are AB (Fig. a), y taking on values between 0 and = G may be included); and if v is negative, we confine ourselves to points on the are CD, y taking on values between ~ 7 and — 3 (- 7 may be included) RULES FOR DIFFERENTIATING 65 61. Differentiation of arc vers v. Let y = are vers v; * then v=vers y. Differentiating with respect to y by XVII, therefore By (€), p. 46 But since v is a function of 2, this may be substituted in the formula ay _ aya, ee dz dv de (A), p- 45 vin, oe | de siny de d 5 ido, V2y— de [en ee the plus sign of the ral] being taken, since sin y is positive for all values of y between 0 and © inclusive, dv xxv 2 (are vers v) = ade - vav—-# EXAMPLES Differentiate the following : 1. y= arc tanaz*, Solution. by XX ae T+ Cae (v= art] 2ax “T+ at 2. y=aresin(@x— 428), Solution. # by XVIII [waz 420) - 8-122? 8 “Vi-te+aat—ioe Ving * Defined only for values of » between 0 and 2 inclusive, and is many- valued. To make the funetion continuous, y is taken as the smallest positive are whose versed sine is v; that is, y lies between 0 and x inclusive. Hence ‘we confine ourselves to are OP of the graph (Fig. a). 66 DIFFERENTIAL CALCULUS wt. 3 y=aresee— 2 Solution. a oe [eH ei (F l — a Vai a —22 5. — are cot (2? — 5) = ————_.. a Ce aie; po cian ee ee dz 1-2 142 a 1. © are cosee a a 2 8. —are vers 22? = o a ea 14. f(a) = 2 Vo? — @ + a? are sin 15. f(e) = Ve —# + aaresin®. 16, z= rare vers! — V2ry— 17. 6 = are sin(r—1). 18. ¢= are tan7 +4, 1—ar 1 19. s = are see 1—# d ina) = (ware sin 2) = are sing + a. Hltano are tan 8) = sec? are tan @ + “8% 22. $ Dog (are cos t)] = 23. f(y) = are cos (logy). 24. f(8) = aresin by XXII a i meres 12, “are tan ae 13, Laresin2 2 v2 I(t) =2Ve—2. se =(452) a+, e dy ao ar ay ar as tand 146 S'()= AVI + c8cd. RULES FOR DIFFERENTIATING 67 25. f(¢) = are tan. eo S’()= }. rotan 26. p = ewetsna, ale dg +? 27. w= aretan® 2". a do eee 28, 8 = are cos oe dat +e oo yarns (bane nes) e } Vow 30, y = “are tana. va or[ ag terete tana (lt tees) | 31. y = aresin (Sinz). y=1. 4sinz 4 32, y = are tan ————_. (= . : B+ Boose Y= 548 cose . 33, y = are cot! + log, |® os © nea a—at 34, y = log (43, J arctan. a 35. y = VI—@ are sine — yan 36. Differentiate the following functions : 4 are sin 22? 2p aresin’ (a) Faresin 2a, (f) Fe aresing. da da a 2 garctanct, (b) Faretan ate, ae . © Zare seoe. (ny om g?-are tan gt. @ Zaare cosa. @ Garesind’. (@) 2 at are cotaz. @) SaretanvIe ® az” * aw . zaresing ® a sinVI-#. () 4 are tan (log a2). ae a 8 (0m) gy (at + S)are see. Qa 8 (ny = arc cot. (0) Z VI=@aresint. Formulas (A), p. 45, for differentiating a function of a function, and (C), p. 46, for differentiating inverse functions, have been added to the list of formulas at the beginning of this chapter,as XXV and XXVI respectively. ay In the next eight examples, first find u and substitute the results in ° * * to fina &. de by differentiation and then by XXV + As was pointed out on p. 44, it might be possible to eliminate v between the two given expressions so as to find y directly as a function of z, but in most cases the above method is to be preferred. 68 DIFFERENTIAL CALCULUS In general our results should be expressed explicitly in terms of the independent ! variable; that is, a in terms of 2, $ in terms of y, 2 in terms of 8, etc. 81, y=2—4,0= 82241. WW < 4p; @ — 6a, substituting in XXV, av a 7 38. y= tan2v, v= aretan(2¢—1), ; substituting in XXV, +1 dy Qsoct2v _» tan?2Qv4+1 _ 2et— 2241 @ 2-241 22-2241 2@—ay? et | [sinco v= ato tan 22—b, tan y= 22-1, an 20m 22=L, | 39. y= 309-4045, v= 22*— 5, Me raa8 — sot, | iz 20 dy 40. y= 10s ove Y= Fy—8 Qa— ax @— | A dy 41. y = log (a? — v2), v= asing. qen7 Btn. I dy e | 42, y =arctan(a+v), v= 6 jad ae | : | a 1+ are? 43, r= e884 et, 8 = log(t— 2). fo4e—oe st. In the following examples first find $ by differentiation and then substitute in wot by XXVI to fina 2, dy 44,c=yVIty. a 2vity_ 45, 2= Vivo. oy. 2Vi toy a siny 46. 5= Teelosv oa oe 41. 2 = alogt t¥e—¥ venr, ay __ Vea ¥ 48. ¢ = rare vers! — Vary =, 49, Show that the geometrical significance of XXVI is that the tangent makes complementary angles with the two codrdinate axes. RULES FOR DIFFERENTIATING 69 62. Implicit functions. When a relation between z and y is given by means of an equation not solved for y, then y is called an implicit function of x For example, the equation 2—4dy=0 defines y as an implicit function of 2. Evidently z is also defined by means of this equation as an implicit function of y. Similarly, e+y+2—ae=0 defines any one of the three variables as an implicit function of the “other two. It is sometimes possible to solve the equation defining an implicit function for one of the variables and thus change it into an explicit function. For instance, the above two implicit functions may be solved for y, giving 2 Jag and y=tVe—2—2; the first showing y as an explicit function of 2, and the second as an explicit function of z and z. In agiven case, however, such a solution may be either impossible or too complicated for convenient use. ‘The two implicit functions used in this article for illustration may be respectively denoted by ¢(n, y= 0 and Fay )=0. 63. Differentiation of implicit functions. When y is defined as an implicit function of x by means of an equation in the form 1c) S@N=% it was explained in the last section how it might be inconvenient to solve for y in terms of x; that is, to find yas an explicit function of « so that the formulas we have deduced in this chapter may be applied directly. Such, for instance, would be the case for the equation (B) aa’ + 22'y —y'x—10 = 0. We then follow the rule: Differentiate, regarding y as a function of x, and put the result equal to zero.* That is, © Ere n=0. process will be justified in §7. Only corresponding values of # and y which given equation inay be substituted in the derivative. 70 DIFFERENTIAL CALCULUS d, Let us apply this rule in finding on from (B). dx SF (att 22ty— yfr—10)=0; by (€) J @ty+ Lary-Lyn—-Lany=o; 5 dy dy ax? + 229% —y Tay +200 + 6aty— yf Tay 0; Ba =T ayy Uo yf — 6 a2 — 6 2°ys dy _y'—6ax'—~6a°y Beer The student should observe that in general the result will contain both « and y. EXAMPLES Differentiate the following by the above rule : Ly =4pe. 2, » 2 x 10. 12, 13, 14, aeyar. Dat + aty? = att, By +2ar=0, ata yt oat. Baytaat. n> van +6 . yt — Day += 0. . a8 + > — Bary = 0. may, » p? = a? cos 20, pt cosd = a? sin86, cos (uv) = ev. 8 = cos (0+ 4). de Pa at dy _y—aylogy dz x? — zy loge dp __atsin28 ae : dp _ 8a*cos36 + psing do 2p cosd . du _ c+ usin (w) dp ~ = vsin(uv) ao sin (0+ ¢) de ~T+sin@+ RULES FOR DIFFERENTIATING 71 15, Find 2 from the following equations: (a) 2? = ay. (f) yt+yt+4c=0 (k) tanz + y> (b) a? 4 4y? = 16, (g) yet —y = 5. () cosy + 8a? (0) 2? — aty? = a, (h) 2? — 24 = 98, (m) zeoty+y @ x=a+a. (i) ay + 4y= (n) y? = loge. () ®@-y=16. (i) Y =sin2a, (0) +278 =0, 16. A race track has the form of the circle 2? + y# = 2600, The directions OX and OY are east and north respectively, and the unit is 1 rod. If a runner starts east at the extreme north point, in what direction will he be going (a) when 25-V2 rods east of OY? Ans, Southeast or southwest, (b) when 25 -V2 rods north of OX? Southeast or northeast. (c) when 30 rods west of OF? E, 86° 52/ 12” N. or W. 8652/12” N. (a) when 40 rods south of OX? (e) when 10 rods east of OF ? 17. An automobile course is elliptic in form, the major axis being 6 miles long and running east and west, while the minor axis is 2 miles long. If a car starts north at the extreme east point of the course, in what direction will the car be going (a) when 2 miles west of the starting point? (b) when 3 mile north of the starting point? MISCELLANEOUS EXAMPLES Differentiate the following functions : 1, aresin-VI Ans. 42° 2. xer, oF (22% +1). 3, log sin = Scot 8 22 4, are cos”. a y roe a @-ai 6 loge * T+ log (1+ log 2)?" 7. log see (1 — 22). —2tan(1— 22). 8, wet 3z, ne-82(2— 82), 9. tos.4/ cost, eset. 1+ cost 10, arc sin -V¥ (1 — cos). . a 28 2 11. are tan — —_————. ‘s (= 58) Vs? 1 12. @2—-1),[ 2. 44a of 2. lta 38042) Vite yg, Maresing , (e+ 2)V1— at, @aresina. 72 u 16. WW. 18. 19. 20. 21. 2% 25. 26. 27. 28. 29. 30. s s DIFFERENTIAL CALCULUS sed + tet. are tan } (@* 4-32), a2 ( ine, @+ahee—nt al eeett—82), are tan V1— a, 2 cosz gana, log sin? 44. = log sin ax. sin? 9 cos g. a 2V0= com mia onectane tm Vi+¢at tan? a — log sec?sr, Blog (2 cose + 3sine) + 2a 13 a are cot + log 31. 32. 33. 84, 35. 36. 37. 38. 39. 40, 41. 42, 43. 44, 45. 46. 47. «+ (log tan 3 — 22), anaes t (+2029 242) @+2)@—8z) . are tan (log 82). . VO= anny, log V (a — bam, log fA +1. y-l enresse20, (esa 1+4z cose e log sinz, are sin arc tan a. atime, cot (log az). 1 (1— 82%) e*, vi-w log. ‘ Vite CHAPTER VI SIMPLE APPLICATIONS OF THE DERIVATIVE 64. Direction of a curve. It was shown in § 82, p. 31, that if y=f@) is the equation of a curve (see figure), then #.. tan = slope of line tangent to the curve at any point P. The direction of a curve at any point is defined to be the same as the direction of the line tangent to the curve at that point. From this it follows at once that ay an tant = slope of the curve at any point P. At a particular point whose coérdinates are known we write (z = slope of the curve (or tangent) at point (x, y,). c= v= At points such as .D, F, H, where the curve (or tangent) is parallel to the avis of X, oy 7 =0°; therefore Fae At points such as A, B, G, where the curve (or tangent) is per- pendicular to the axis of X, dy 7 = 90°; therefore 2 =o. dx 8 74. DIFFERENTIAL CALCULUS. At points such as Z, where the curve is rising,* 7 =an acute angle; therefore # =a positive number. The curve (or tangent) has a positive slope to the left of B, between D and F, and to the right of @ At points such as C, where the curve és falling,* a + =an obtuse angle; therefore S =a negative number. ‘The curve (or tangent) has a negative slope between B and D, and between F and G. 2 Tnuusrrative Exampre 1, Given the curve y = . —a42 (see figure). (a) Find + when # =1. (b) Find 7 when 2 = 3, (©) Find the points where the curve is parallel to OX. (a) Find the points where + = 45°, (e) Find the points where the curve is parallel to the line 20— By =6 (line AB), Solution. Differentiating, au = 2 — 2 = slope at any point, (a) tan -|% —2=-1; therefore r= 135°, Ans, (b) tanr -[2 —6=8; therefore 7 = are tan3, Ans. () T=0% tant= & =0; therefore 2? 22= 0, Solving this equation, we find that 0 or 2, giving points C and D where the curve (or tangent) is parallel to OX. ty Solving, we get « =14-V2, a) 45%, tant 1; therefore 2? — 22 giving two points where the slope of the eurye (or tangent) is unity. (e) Slope of line = 3; therefore 2* — 2% = 3. Solving, we get 2 =1-4. points Z and F where curve (or tangent) is parallel to line AB. giving Since a curve at any point has the same direction as its tangent at that point, the angle between two curves at a common point will be the angle between their tangents at that point. Inwusrrariy. Vind te angle of intersection of the circles (A) e+y—4e51, (B) e+y—2y * When moving from left to right on curve. 75 SIMPLE APPLICATIONS OF THE DERIVATIV Solution. Solving simultaneously, we find the points of intersection to be (8, 2) and te fy 2-2 rom (4). By § 63, p. 69 yy —*_ from (B). By § 68, p. 69 — — } = slope of tangent to (A) at (3, 2). — 8 = slope of tangent to (B) at (8, 2). ‘The formula for finding the angle between two lines whose slopes are m, and m, is aa 55, p. 3 Substituting, tan @ = oe =1; therefore 9 = 45°. Ans. ‘This is also the angle of intersection at the point (1, — 2). tang = EXAMPLES ‘The corresponding figure should be drawn in each of the following example: 1. Find the slope of y = the origin, Ans. 1= tant. i+a 2. What angle does the tangent to the curve 2%y? = a¥(e + y) at the origin make with the axis of X? Ans. 7 = 135°, 3. What is the direction in which the point generating the graph of y= 38a?—x tends to move at the instant when z=12? Ans. Parallel toa line whose slope is 5. 4, Show that & (or slope) is constant for a straight line. in 5. Find the points where the eurve y= 2° — 32° — 9x + 5 is parallel to the axis of x. Ans, 2 =8,2=—1. 6. At what point on y? = 22° is the slope equal to 3? Ans. (2, 4). 7. At what points on the circle a? + y? = 1? is the slope of the tangent line equal a . ane, (45, 22). 8. Where will a point moving on the parabola y = 2*— 7. +8 be moving paral- lel to the line y = 5242? Ans. (6, — 8). 9. Find the points where a particle moving on the civele a® + y2 = 169 moves per- pendicular to the line 52 + 12y = 60. Ans, (+12, 5). 10. Show that all the curves of the system y = log kr have the sume slope ; i.e. the slope is independent of k, 11. The path of the projectile from a mortar cannon lies on the parabola y= 2x —2a?; the unit is 1 mile, OX being horizontal and OY vertical, and the origin being the point of projection, Find the direction of motion of the projectile (a) at instant of projection ; (b) when it strikes a vertical cliff 1) miles distant. (c) Where will the path make an inclination of 45° with the horizontal ? (a) Where will the projectile travel horizontally ? Ans. (a) are tan 2; (b) 185°; (¢) Gi. #5 (@) GD. 6 DIFFERENTIAL CALCULUS 12, If the cannon in the preceding example was situated on a hillside of inclination 45°, at what angle would a shot fired up strike the hillside ? Ans, 45°. 18. At what angles does a road following the line 8y — 2 — 8 = O intersect a rail- way track following the parabola y? = 82. ‘Ans. arctan}, and arc tan}. 14. Find the angle of intersection between the parabola y? = 6x and the circle ayy =i6. Ans, are tan§ V8. 2 yt 15. Show that the hyperbola 2° —y? = 5 and the ellipse + e =1 intersect at right angles. = 16. Show that the circle 2? 4 y? = 8 ax and the cissoid y* = (a) are perpendicular at the origin ; (b) intersect at an angle of 45° at two other points. 2B 2a—z 17. Find the angle of intersection of the parabola 2?=4ay and the witch 8a* Bae Ans, arctan 8 = 71° 88.9, 18. Show that the tangents to the folium of Descartes 2° + y° = 3 azy at the points where it meets the parabola y* = az are parallel to the axis of Y. 19. At how many points will a particle moving on the curve y = «8 — 22% 42—4 be moving parallel to the axis of X? What are the points ? Ans. Two at (1, — 4) and (, — 484). 20. Find the angle at which the parabolas y = 84? — 1 and y = 2? 42 intersect. Ans. are tan fy. 21, Find the relation between the coefficients of the conics a,2* + by? =1 and az? + byy? = 1 when they intersect at right angles. iota a a by 65. Equations of tangent and normal, lengths of subtangent and subnormal. Rectangular codrdinates. The equation of a straight line passing through the point (z, y,) and having the slope, m is y-y=m@e—2,). 54, (0), p.8 If this line is tangent to the curve AB at the point P(x, y,), then from § 64, p. 73, m=tan Tt Hence at point of contact R(a, y,) the equation of the tangent line TR is i wy, () y-W=Te- 4H): dx, * By this notation is meant that we should first find a, then in the result substitute a for x and y; for y. ‘The student is warned against interpreting the symbol mm to mean the 1 derivative of 1 with respect to 7, for that has no meaning whatever, since #, and y; are doth constants. SIMPLE APPLICATIONS OF THE DERIVATIVE TT The normal being perpendicular to tangent, its slope is 1_ dy, ae By 55, p.3 And since it also passes through the point of contact B(x, y,), we have for the equation of the normal RN dr, @) y-Ww=—- (4-H). dy, That portion of the tangent which is intercepted between the point of contact and OX is called the length of the tangent (= TR), and its projection on the axis of X is called the length of the subtangent (=7M). Similarly, we have the length of the normal (= BN) and the length of the subnormal (= MN). i" a In the triangle 7RM, tan r= 2 TM ; therefore _ tant @) rM* y, 21 = tength of subtangent. dy, In the triangle MBN, tan r= ay, therefore LR? (4) MNt =MP tan t=y, # = length of subnormal. 4 The length of tangent (= 7P) and the length of normal (= RN) may then be found directly from the figure, each being the hypotenuse of a right triangle having the two legs known. Thus een TP, =V TM + UR = NCE) +, : dA? 6) =y, (3) +1=length of tangent. i 1s dz, ay. 6 = Ht +(F) = length of normal. fy The student is advised to get the lengths of the tangent and of the normal directly from the figure rather than by using (5) and (6). When the length of subtangent or subnormal at a point on a curve is determined, the tangent and normal may be easily constructed. * Uf subtangent extends to the right of T, we consider it positive; if to the left, negative. + If subnormal extends to the right of a, we consider it positive; if to the left, negative. 78 DIFFERENTIAL CALCULUS. EXAMPLES 1, Find the equations of tangent and normal, lengths of subtangent, subnormal 3 tangent, an/l normal at the point (a, ) on the eissoid 9? = = = ane 3 Solution. cae : es de ~y@a—ap dy, [dy Baa . Hence w= [z sop aaa a 2 = HORE of tangent. ¥ ies Substituting in (1) gives y= 2a — a, equation of tangent. ie Substituting in (2) gives x 2y +2 =a, equation of normal, opr Substituting in (8) gives T= 5 = length of subtangent. Substituting in (4) gives MN =2a _ Also PT = V(THY + MPF = Vite +a length of subnormal. ae = length of tangent, and PN =V(MN} + (MPP = Vie + 2 5 = length of normal. 2. Find equations of tangent and normal to the ellipse 2? + 2y* — 22y — 2 =O at the points where x = 1. Ans, At (1,0), 2y=e—l,y+2e=2 At(1,1),2y=e41y42e=3. 8. Find equations of tangent and normal, lengths of subtangent. and subnormal at the point (2,, y,) on the circle a? + y? = 12. Ans. 22+ AY =", ty — yt =0, — 4, Show that the subtangent to the parabola y? = 4 pz is bisected at the vertex, and that-the subnormal is constant and equal to 2p. 5. Find the equation of the tangent at (2,, 7,) to the ellipse = @ 3 6. Find equations of tangent and normal to the witeh y= we the point wherez = 2a, ae Ans, 24 2y=4a,y 22-84. 7. Prove that at = point on, the eatenary y ge 4e74 the lengths of sub- normal and normal are? (ea. Sie) ae a 8, Find equations of tangent and normal, lengths of subtangent and swbnormal, to cach of the following curves at the points indicated : (a) y= a2 at (1, 4). (©) y= 9—22at (— 3,0). () y= 4eat (9,— 6). (f) @ = Gy where z (c) 2 + Sy? = 14 where y = 1. (g) a —ay + 22—-9=0, (8,2). (@) 224 y= Hat (—3,—4). (h) 22? y= 14 at B, — 2). “In Exs. 8 and 5 the student should notice that if we drop the subscripts in equations of tangents, they reduce to the equations of the curves themselves. SIMPLE APPLICATIONS OF THE DERIVATIVE 79 9, Prove that the length of subtangent to y = a* is constant and equal to ; og a 10. Get the equation of tangent to the parabola y? = 20x which makes an angle of 45° with the axis of X. Ans, y=ut5. Hin. First find point of contact by method of Illustrative Example 1, (4), p. 74. 11, Find equations of tangents to the cirele 2? + y? = 62 which are parallel to the line 22 + 8y =6. Ans, 22+ 3y + 26=0. 12. Find equations of tangents to the hyperbola 42?— 9y? + 86 = 0 which are perpendicular to the line 2y + 5a = 10. Ans, 20 —5y4+8=0, 18. Show that in the equilateral hyperbola 2y =a? the area of the triangle formed by a tangent and the codrdinate axes is constant and equal to a’. 14, Find equations of tangents and normals to the curve y?=2a?— a at the points where z = 1. Ans. At(1,1),2y=241,y +22 =3. At(1,—1),2y —ly-22=— 15. Show that the sum of the intercepts of the tangent to the parabola dt ytoat on the codrdinate axes is constant and equal toa. 16. Find the equation of tangent to the curve 2? (x + y) = a? (x — y) at the origin. Ans. y =a. 17. Show that for the hypocycloid 28 + y$ = al that portion of the tangent included between the cobrdinate axes is constant and equal to a, 18. Show that the curve y = ae* has a constant subtangent. 66. Parametric equations of a curve. Let the equation of a curve be (A) Fay)=0. If « is given as a function of a third variable, ¢ say, called a param- eter, then by virtue of (A) y is also a function of ¢, and the same fune- tional relation (4) between x and y may generally be expressed by means of equations in the form : : z=fO» oy y= 9s each value of ¢ giving a value of z and a value of y. Equations (B) are called parametric equations of the curve. If we eliminate ¢ between equations (B), it is evident that the relation (4) must result. For example, take equation of circle Pyar, ory=Vr Let a=rcost; then y=rsint, and we have r=rcost, Cc fi {yr rSine as parametric equations of the circle in the figure, ¢ being the parameter. VS 80 DIFFERENTIAL CALCULUS If we eliminate ¢ between equations (C) by squaring and add- ing the results, we have 2+ y= 7 (cos*t + sin®t)= 7", the rectangular equation of the circle. It is evident that if ¢ varies from 0 to 27, the point P (2, y) will describe a complete circumference. In $71 we shall discuss the motion of a point P, which motion is defined by equations such as z=fO, y=oO- We call these the parametric equations of the path, the time ¢ being the parameter. Thus in Ex. 2, p. 93, we see that T= 0, COSa-t, y=—hot +, sina-t are really the parametric equations of the trajectory of a projectile, the time ¢ being the parameter. ‘The elimination of ¢ gives the rectan- gular equation of the trajectory ge =a tana ——*_.. y Due cosa Since from (2B) y is given as a function of ¢, and ¢ as a function of a, we have 2 dy dy dt aa ae ae by Xx by XXVE that is, dy ® a Hence, if the parametric equations of a curve are given, we can find equations of tangent and normal, lengths of subtangent and subnor- mal at a given point on the curve, by first finding the value of 2 at that point from (D) and then substituting in formulas (1), (2), (3), (4) of the last section, SIMPLE APPLICATIONS OF THE DERIVATIVE 81 Intusrrative Examere 1. Find equations of tangent and normal, lengths of sabtangent and subnormal to the ellipse P (x =acos¢, ©) Ly =bsing* at the point where ¢ = Z 7 j ae : Solution, The parameter b apt tn, z = beos¢. : dy __ deose Substituting in (D), . = slope at any point. ubstituting in (D), SH =— ZEB — slope at any poin Substituting ¢ == in the given equations (E), we eet (Fe Za) as the point of v2 V2, ss contact. Hence Substituting in (1), p. 76, ” or, br + ay =V2ab, equation of tangent, > Substituting in (2), p. 77, y— 3(e- @ ) = v2} v2, or, ‘V2 (az — by) = a? — b°, equation of normal. Substituting in (8) and (4), p. 77, d ( ») a — 2) =— = tength of subnormal, : v2\ a av2 ab (- *) — = length of subtangent. ~ va) 5 ; *As in the figure draw the major and minor auxiliary circles of the ellipse. Through two points B and C on the same radius draw lines parallel to the axes of codrdinates. ‘These lines will intersect in a point P (x, y) on the y| ellipse, because 2=0A= OB cosp~acosd and y= AP = OD= OCsing=bsing, on, cos and = sing. a Now squaring and adding, we get y cos? + sin? 6=1, the rectangular equation of the ellipse. ¢ is sometimes called the eccentric angle’of the ellipse at the point P. 82 DIFFERENTIAL CALCULUS Iuwsrrarive Exampie 2. Given equation of the eycloid * in parametric form z=a(—sind), t = a(1— cos), 4 being the variable parameter; find lengths of subtangent, subnormal, tangent, and normal at the point where =f. Solution. ae a(1—cos6), uy = asin. Substituting in (D), p. 80, dy _ _8iN6__ stope at any point. dz 1—cos@ ~ Since @ =, the point of contact is & -4, 4), and ze Substituting in (8), (4), (6), (6) of the last section, we get length of subtangent length of subnormal = length of tangent, v2, length of normal EXAMPLES Find equations of tangent and normal, lengths of subtangent and subnormal to each of the following curves at the point indicated : Tangent Normat Subt. Subn. ‘Lea t2y L e—dy41=0, 8242y-9=0, 2% 4 Ba=,yaBs t= 122—y—16=0, t4+12y—98=0, 3, 96. 3 2=By=8; 82—2y—-1=0, 2248y—-5=0, % 4. g=2e,y ert; t z+2y—4=0, 22—y—-3=0, -h 5. @=sint, y= cos2t; t=. 2y44e—8=0, 4y—20— * The path described by a point on the circumference of a cirele which rolls without sliding on a fixed straight line is called the eycloid. Let the radius of the rolling cirele be a, P the generating point, and Af the point of contact with the fixed line OX, which is called the base. If are PM equals O3fin length, then P will touch at 0 if the circle is rolled to the left. ‘We have, denoting angle POM by 6, M-NM= a6-a sin 0=«(9-sin 6), PN= MC- AC=a-aeos = a(1~c0s 6), the parametric equations of the cycloid, the angle @ through which the rolling circle turns Deing the parameter. OD=27a is called the base of one arch of the eycloid, and the point V is called the vertex. Eliminating 6, we get the rectangular equation eaasieea(=2) VF. SIMPLE APPLICATIONS OF THE DERIVATIVE 83 Ly=e; lL. «= cost, y= sin2t; 1. e=8t,y=6t—-#; t=0. 12. 2 =B8e-4 y= Bet; tH 0. =@jy=t;t=2 13, 2 =sint, y =2cost; t 9 e=0,y=2;t=—-1. 14, = dost, y = Bsint; =F. 10. ¢=2-t,y=80; t=1. 15. c= log(t + 2), y=t; t= 2. In the following curves find lengths of (a) subtangent, (b) subnormal, (c) tangent, (@) normal, at any point : Oa { a(cost + tsind), : y = a(sint — t cost). Ans. (a) ycott, (b) y tant, (c) _ @ Pa 4a cost, =4dasint, 17. The hypocyeloid (astroid) { Yy Ans. (a) —ycott, (b) —ytant, (¢) 2, @ 4. (@) ~voott, 0) — tant, (0) 2 (a) 2G (2=rcost, 18. The circle : yersint. =a(2 t— 21) 19. The cardioid te a y =a(2sint—sin20, Bt 148" ). The folium 20. The folium a U Tee a= feos, 21, The hyperbolic spiral y= Ssint t 67. Angle between the radius vector drawn to a point on a curve and the tangent to the curve at that point. Let the equation of the curve in polar codrdinates be p = (8). Let P be any fixed point (p, 8) on the curve. ENO If 6, which we assume as the independent vari- Ip able, takes on an increment Ad, then p will A, take on a corresponding increment Ap. Denote by @ the point (p+Ap, @-+A0). Draw PR perpendicular to 0@. Then 0Q=p+Ap, PR =psin AO, and OR = pcos Ad. Also, PR_ PR psin AO tan POR =F5=Go— Oh p+ Ap—pcosAd 84 DIFFERENTIAL CALCULUS Denote by the angle between the radius vector OP and the tangent PT. If we now let A@ approach the limit zero, then (a) the point Q will approach indefinitely near P; (b) the secant PQ will approach the tangent PT as a limiting posi- tion; and (e) the angle PQR will approach yp as a limit. Hence limit ___psin A@ ti = eee an Y= 49-0 ppap—p cos Ab EY _ limit __psin Ag fu & X [since trom a0, p. 29 eos sep cosa) =2paine 82. psin Ad _ limit Ae ~Ad=0 2psineA?® 2 Ap ae +20 {Dividing both numerator and denominator by 86) _sin AO — limit ary Ag=0 AG. nS psin a + me limit /,, 40) _ limit /sin AQ) _ aoe (sm =0, Vl ocd Ao simi imi : and gp ggg aE bY $.2% pl, we have 2 (A) tan = £. - From the triangle OP’ we get @ rong SIMPLE APPLICATIONS OF THE DERIVATIVE 85, Having found 7, we may then find tan, the slope of the tangent to the curve at P. Or since, from (B), tan 7 = tan (0 +) we may calculate tan y from (A) and substitute in the formula tan@ + tan (C) slope of tangent = tan 7 = Iutusrrative Exanrre 1. Find y and in the eardioid p = a(1—cos6), Also find the slope at @ = 3 : Solution. A = asin@, Substituting in (A) gives = (1 — cos) ons 6 pal = > = tan8. i tany = = ane FH xy By sop. 2, and 37, p.2 7] 2asin 3 cos a8 ve a Since tany=tan5.y=S- Ans, Substituting in (B), r= 6 +3 tan = tan = Ans. To find the angle of intersection ¢ of two curves C and C! whose equations are given in polar codrdinates, we may proceed as follows : angle TPT" = angle OP7'— angle OPT, c or, g=y'—y Hence (D) tang = tn y" = tan y 1+ tany' tay’ where tan yp! and tan y are calculated by (A) from the two curves and evaluated for the point of intersection. Itustrative Exampie 2. Find the angle of x intersection of the curves p = asin26, p = acos20. Solution. Solving the two equations simultaneously, we get at the point of inter section tan26=1, 20 = 45°, 6 = 223°. From the first curve, using (4), tan y/ = } tan 26 = 3, for @ = 224°. ‘From the second curve, tany =— J cot28 Substituting in (D), t 3, for @ = 224°, = are tan $. Ans, 86 DIFFERENTIAL CALCULUS 68. Lengths of polar subtangent and polar subnormal. Draw a line NT through the origin perpendicular to the radius vector of the 7 point P on the curve. If PZ is the tangent and PN the normal to the curve at P, then Lp OF = length of polar subtangent, ol x and ON = length of polar subnormal of the curve at P. A Or In the triangle OP7, tan = —- Therefore Mp Pp (7) OT =ptany=p* 2 = length of polar subtangent.* ip In the triangle OPN, tany =... Therefore 8 on’ p__4p (8) ON=——=+4= . (8) an 6 length of polar subnormal. The length of the polar tangent (= PT’) and the length of the polar normal (= PN) may be found from the figure, each being the hypot- enuse of a right triangle. Intusrrative Examrrx 8. Find lengths of polar subtangent and subnormal to the lemniseate p? = a? cos 26. Solution. Differentiating the equation of the curve as an implicit function with respect to 8, 2p = 2a2sin 20, = dp __atsin2a : a6 o Substituting in (7) and (8), we get length of polar subtangent =~?» _, os . Pain 20 = length of polar snbnormal =— 2226, ° If we wish to express the results in terms of 8, find p in terms of 8 from the given equation and substitute. Thus, in the above, p= +4 aVcos20; therefore length of polar subtangent = + a cot 26-Vcos 20. * When 0 increases with p, 7 fs positive and ¥ is an acute angle, as in the above figure. ‘Then the subtangent 0 T'is positive and is measured to the right of an observer placed at Oand looking along OP. When 7 is negative, the subtangent is negative and is measured to the left of the observer. SIMPLE APPLICATIONS OF THE DERIVATIVE 87 EXAMPLES I. In the circle p = r sind, find y and 7 in terms of 8. Ans. Y=0,7=28. 2. In the parabola p= a sect, show that r+ y=. 3. In the curve p? = a? cos 26, show that 2y = 7 + 46. 4, Show that y is constant in the logarithmic spiral p = e#®, Since the tangent makes a constant angle with the radius vector, this curve is also called the equi- angular spiral. 5. Given the eurve p 4y. asin, prove that 7 &. Show that tan ¥=@ in the spiral of Archimedes p= a9. Find values of y when = 27 and 47. Ans. y = 80° 57’ and 85°27’. 7. Find the angle between the straight line p cos@=2a and the circle p= Sasind. Ans. are tan}. 8, Show that the parabolas p= a sects and p = bese? g intersect at right angles, 9. Find the angle of intersection of p= asing and p= asin 26. Ans. At origin 0°; at two other points are tan8-V3. 10. Find the slopes of the following curves at the points designated : © (a) p= a(1—cosé). 6 Ans, —1. (b) p=asect9. p=2a. ES () p=asin4g, origin. 0, 1,0, —1. (@) p?=atsin 49. origin. 0, Le, —1. (e) p=asin36. origin, 0, V8, — V3. () p= acos8d. origin, (g) p= «@cos26. (h) p=asin29. @) p=asin3. () p=ad (&) p9=a. p=e. 11. Prove that the spiral of Archimedes p = a8, and the reciprocal spiral p=“, intersect at right angles. a 12, Find the angle between the parabola p= a sec? : and the straight line psind=2a. Ans, 45°, 18. Show that the two cardioids p= a(1+ cos) and p=a(1— cos) cut each other perpendicularly. 14. Find lengths of subtangent, subnormal, tangent, and normal of the spiral of : a ; Archimedes p= af. Ans, suit, <2, tan, VET subn, =a, nor. Va? + p*. ‘The student should note the fact that the subnormal is constant. 88 DIFFERENTIAL CALCULUS 15. Get lengths of subtangent, subnormal, tangent, and normal in the logarithmic spiral p= a, Ans. subt.=—°—, tan=pafi¢—, Toga Tog? a subn.=p loga, nor. = p V+ log?a. ‘When a =e, we notice that subt. = subn., and tan. = nor. 16. Find the angles between the curves p = a(1 + cos6), p = b(1— cos6). a Ans. OandZ. 17. Show that the reefprocal spiral p = $ has a constant subtangent. 2 18. Show that the equilateral hyperbolas p® sin 24 = a, p® cos 26 = J? intersect at right angles. 69. Solution of equations having multiple roots. Any root which occurs more than once in an equation is called a multiple root. Thus 38, 3, 3, —2 are the roots of (A) at —Tad +92" + 272-54 = hence 8 is a multiple root occurring three times. Evidently (4) may also be written in the form (e—3)°@+2)=0. Let f(z) denote an integral rational function of x having a multiple root a, and suppose it occurs m times. Then we may write B® F@O=@—O"9@), where $(z) is the product of the factors corresponding to all the roots of f(z) differing from «. Differentiating (B), i. SP@=@- OF @) + b@) mean, Ce) SO=@-O""[E-DP@)+4b@)m]- Therefore f"(z) contains the factor (e—a) repeated m—1 times and no more; that is, the highest common factor (H.C.F.) of f(«) and f(z) has m—1 roots equal to a. In case f(z) has a second multiple root 8 occurring r times, it is evident that the H.C.F, would also contain the factor (e —8)'-}, and so on for any number of different multiple roots, each occurring once more in f(#) than in the H.C.F. We may then state a rule for finding the multiple roots of an equation (2) =0 as follows: First Step. Find f(x). Sroonn Sree. Find the H.C.P. of f(#) and f'(2). Turrp Srer. Find the roots of the H.C.F. Each different root of the B:CLE. will occur once more in f(a) than it dovs in the H.C.F. SIMPLE APPLICATIONS OF THE DERIVATIVE 89 If it turns out that the H.C.F. does not involve 2, then f(2) has no multiple roots and the above process is of no assistance in the solution of the equation, but it may be of interest to know that the equation has no equal, i.e. multiple, roots. Iunustrative Examrie 1, Solve the equation 2? — 82% + 182—6=0. Solution. Place S(t) =a — Ba? 4 132-6, First step. Sa) = 822 — 162418 Second step. HCF,=2-1. Third step. z-1=0. «. Sinee 1 occurs once as a root in the H.C.F., it will oceur twice in the given equa- tion ; that is, (e — 1)? will occur there as a factor. Dividing 2° — 82? + 182 — 6 by (« —1)? gives the only remaining factor (z — 6), yielding the root 6. The roots of our equation are then 1, 1, 6. Drawing the graph of the function, we see that at the double root z= 1 the graph touches OX but does not cross it.* EXAMPLES Solve the first ten equations by the method of this section: 1, a — Ta? 4162-12 Ans. 2,2, 3. 2. at 62%— 82-8 -1,-1,-1,3. 3. at — Ta? + Oa? + 27x — 54 = 0, 8, 8,3, —2. 4, at 52° — 9a? 4 812—108=0. 8, 8,3, —4. 5. xt + 623 + 2? — 242 4+16=0, 1,1,-4, 6. at 949 + 284? — 34 —36=0. 8,8, —1,4. 7. ot — 62% + 1027-8 =0. 2,2, 14-V3. 8, oS at 5a tat48244=0. —1,-1,-1,2,2. 9, 25 — 152° +102? + 60z—72=0. 29 6 8 10, a = Bat — 529 + 182? + 242 +10 = 0. =1,-1,-1,84V—-1L Show that the following four equations have no inultiple (equal) roots: LL, 2 + 92% + 22-48 =0. 12, at — 15a? — 102 + 24 = 0. 18. at — 323 — 6a? + 14a +1: 14. 7 -— a= 15, Show that the condition that the equation + 3qrtr shall have a double root is 44° + 7? = 0. 0 16. Show that the condition that the equation 4 Spz? +r shall have a double root is r(4p% +r) = 0. Since the first derivative vanishes for every multiple root, it follows that the axis of X is tangent to the graph at all points eorre- sponding to multiple roots. If a multiple root occurs an even number of times, the graph will not eross the axis of X at such a point (see figure); if it occurs an odd number of times, the graph will cross. 90 DIFFERENTIAL CALCULUS 70. Applications of the derivative in mechanics. Velocity. Recti- linear motion. Consider the motion of a point P on the straight line AB. Let s be the distance meas- 8 As. . — ps5 ured from some fixed point as A to any position of P, and let ¢ be the corresponding elapsed time. To each value of ¢ corresponds a position of P and therefore a distance (or space) s. Hence s will be a function of ¢, and we may write s=f(t). Now let ¢ take on an increment At; then s takes on an increment As,* and As . (A) Arm the average velocity of P during the time interval At. If P moves with uniform motion, the above ratio will have the same value for every interval of time and is the velocity at any instant. For the general case of any kind of motion, uniform or not, we define the velocity (time rate of change of s) at any instant as the limit of the ratio “ as At approaches the limit zero; that is, — limit As, ™ At=OAt or, ds 9) v=S. 9) a The velocity is the derivative of the distance (= space) with respect to the time. To show that this agrees with the conception we already have of velocity, let us find the velocity of a falling body at the end of two seconds. By experiment it has been found that a body falling freely from rest in a vacuum near the earth’s surface follows approximately the law (BY s=16.18, where s=space fallen in feet, ¢=time in seconds. Apply the Gen eral Rule, p. 29, to (B). * As being the space or distance passed over in the time A¢, SIMPLE APPLICATIONS OF THE DERIVATIVE 91 Fist Svar. s+As—16.1(¢-+At)?=16.10432.2t-At+16.1(A0% Skconn Star, Av = 32.20-At-+16.1 (A? Tarep Sree. * =32.2-4+16.1At= average velocity throughout the time interval At. Placing t= 2, (©) a = 64.4 £161 At =average velocity throughout the time interval At after two seconds of falling. Our notion of velocity tells us ab once that (C) does not give us the actual velocity at the end of two seconds; for even if we take At very small, say zh; or alg of a second, (C) still gives only the average velocity during the corresponding small interval of'time. But what we do mean by the velocity at the end of two seconds is the limit of the average velocity when At diminishes towards zero; that is, the velocity at the end of two seconds is from (C), 64.4 ft. per second. ‘Thus even the everyday notion of velocity which we get from experi- ence involves the idea of a limit, or in our notation _ limit /As\_ ; =O (a3) = 64.4 ft. per second. The above example illustrates well the notion of a limiting value. ‘The student should be impressed with the idea that a limiting value is a definite, fixed value, not something that is only approximated. Observe that it does not make any difference how small 16.1 At may be taken; it is only the limiting value of 64.4 + 16.1 At, when At diminishes towards zero, that is of importance, and that value is exactly 64.4. 71. Component velocities. Curvilinear motion. The codrdinates x and y of a point P moving in the XY-plane are also functions of the time, and the motion may be defined by means of two equations, : z=fOQ, y=bO-" These are the parametric equations of the path (see § 66, p. 79). * The equation of the path in rectangular codrdinates may be found by eliminating ¢ between these equations. 92 DIFFERENTIAL CALCULUS ‘The horizontal component 2, of v* is the velocity along OX of the projection af of P, and is therefore the time rate of change of x. Hence, from (9), p. 90, when s is replaced by 2, we get ax 10) y= 2, (10) eae In the same way we get the vertical com- ponent, or time rate of change of y, x 11) yas. (1) v= Representing the velocity and its components by veetors, we have at once from the figure ‘ Pawty, or, > ds ‘dea? 12) vel= 0 a7 NG) +( giving the magnitude of the velocity at any instant. If + be the angle which the direction of the velocity makes with the axis of X, we have from the figure, using (9), (10), (11), dy ax dy vy dt vee dt v, _ at 13 i wot; aif; Se da sine = 4 aor ; ae a de at dt dt 72. Acceleration. Rectilinear motion. In general, v will be a function of ¢, and we may write vay). Now let ¢ take on an increment At, then v takes on an increment Av, and Av Ma the average acceleration of P during the time interval At. We define the acceleration a at any instant as the limit of the ratio x as At approaches the limit zero; that is, a — limit (Av), At=0\At dv a=-. dt The acceleration is the derivative of the velocity with respect to the time. or, (id) * The direction of » is along the tangent to the path. SIMPLE APPLICATIONS OF THE DERIVATIVE 93 73. Component accelerations. Curvilinear motion. In treatises on Mechanics it is shown that in curvilinear motion the acceleration is not, like the velocity, directed along the tangent, but toward the coneaye side of the path of motion. It may be resolved into a tan- gential component, a, and a normal component, a,, where a, vy ai a, =s° dt R (2 is the radius of curvature. See § 103.) The acceleration may also be resolved into components parallel to the axes of the path of motion. Following the same plan used in § 71 for finding component velocities, we define the component accelerations parallel to OX and OY, (18) ‘dv,\? ‘dv,\* 1 = om 4 ee ° (Gy+ (Ge which gives the magnitude of the acceleration at any instant. (14a) a, Also, EXAMPLES 1. By experiment it has been found that a body falling freely from rest in a vac uum near the earth's surface follows approximately the law 8 = 16.142, where s = space (height) in feet, ¢= time in seconds. Find the velocity and acceleration (a) at any instant; (b) at end of the first second ; (¢) at end of the fifth second. Solution. (4) s=16.12. (a) Differentiating, (B) ae 82.24, or, from (9), » = 82.2¢ft. per sec. Differentiating again, (C) ee 82.2, or, from (14), a = 82.2 ft, per (sec.)?, which tells us that the acceleration of a falling body is constant ; in other words, the velocity increases 82.2 ft, per see. every second it keeps on falling. (b) To find v and a at the end of the first second, substitute t= 1 in (B) and (C) ; v= 82.2 ft, per sec., @ = 32.2 ft, per (sec.)?. (o) Tofind v and a at the end of the fifth second, substitute t= 6 in (B) and (0) ; v= 161 ft. per sec., a = 82.2 ft. per (sec.)?. 2. Neglecting the resistance of the air, the equations of motion for a projectile are r= vcosg-t, y=v, sing-t— 16.12; yy where », = initial velocity, g = angle of projection with hori- zon, t= time of flight in seconds, 2 and y being measured in a feet. Find the velocity, acceleration, component velocities, iA and component accelerations (a) at any instant ; (b) at the end of the first second, having given v, = 100 ft, per sec., ¢ = 30°; (c) find direction of motion at the end of the first second, ax 94 DIFFERENTIAL CALCULUS Solution, From (10) and (11), ® cos; vy = v, sin g — 82.21. ‘Also, from (12), v= Vu? — 64.4 tm, sing + 1086.82. From (15) and (16), ar =0; ay=— 82.2; a=— 82.2. (¥) Substituting = 1, », = 100, g = 30° in these results, we get ng = 86.6 ft. per sec. a= 0. vy = 17.8 ft. per see. ay = — 82.2 ft. per (sec). » = 88.4 ft, per sec. a =— 82.2 ft. per (see.)*. © 7 =aretan™ = are tan "8 — 11° 367.6 = angle of direction of motion with the horizontal, °* a 3. Given the following equations of rectilinear motion, Find the distance, velocity, and acceleration at the instant indicated : Ans. 8=16,0=20,a=16. (b) 6 (c) 8=8— 41; (a) o=2t— {e) y=2t—-B; t= 0. (f) k= 204162; t=10. (g) #=2sint; (h) y= @cos () s=208" (j) 8328 — 36; (ky) e448 () y= oos2t; t= mt =bsin™ 5 ¢ (m) $= sin (n) = ae~2; t (0) sate ue: La ty 4 10 log Saat (p) 8 yt 4, If a projectile be given an initial velocity of 200 ft. per sec. in a direction inclined 45° with the horizontal, find {a) the velocity and direction of motion at the end of the third and sixth seconds; (b) the component velocities at the same instants. Conditions are the same as for Ex. 2. Ans, (a) When t= 8, 148.3 ft, per sec., T= 17°35’, when t= 6, v= 150.5 ft. per sec., t= 159° 53’; (b) when ¢=8, vp = 141.4 ft. per sec., vy = 44.8 ft. per sec. when t = 6, ve = 141.4 ft. per see., vy =— 51.8 ft, per sec, 5, The height (= s) in feet reached in t seconds by a body projected vertically upwards with a velocity of , ft. per see. is given by the formula $= ot — 16.18, SIMPLE APPLICATIONS OF THE DERIVATIVE 95 Find (a) velocity and acceleration at any instant; and, if v,= 800 ft. per sec., find velocity and acceleration (b) at end of 2 seconds; (c) at end of 15 seconds. Resist- ance of air is neglected. Ans, (a) 2= 0, — 82.21, a =— 82.2; (b) 0 = 235.6 ft. per sec. upwards, @ = 82.2 ft. per (sec.)? downwards; (c) v= 188 ft. per sec. downwards, 82.2 ft. per (sec.)? downwards. 6, A cannon ball is fired vertically upwards with a muzzle velocity of 644 ft. per sec, Find (a) its velocity at the end of 10 seconds; (b) for how long it will continue to rise. Conditions same as for Ex, 5, Ans, (a) 322 ft, per sec, upwards; (b) 20 seconds, 7. A train left a station and in ¢ hours was at a distance (space) of P+ 2h +8e : miles from the starting point. Find its acceleration (a) at the end of ¢ hours; (b) at the end of 2 hours, Ans, (a) a= 6t+4; (b) @=16 miles per (hour), 8. In ¢ hours a train had reached a point at the distance of } 4 — 44 4 16 miles from the starting’ point. (a) Find its velocity and acceleration. (b) When will the train stop to change the direction of its motion? (c) Describe the motion during the first 10 hours, Ans. (a) v= 8 —120 4 821, a= 80 ~ 241 + 82; (b) at end of fourth and eighth hours; (c) forward first 4 hours, backward the next 4 hours, forward again after 8 hours. a= 9. The space in feet described in ¢ seconds by a point is expressed by the formula $= 48t—168, Find the velocity and acceleration at the end of 1} seconds. Ans, 0=0,a= 82 ft. per (sec,)®. 10. Find the acceleration, having given ° Ans, « (a) v=acos8t; t=7 (e) v= 58; t= a=We, 11, At the end of t seconds a body has a velocity of 81 4 2t ft. per sec,; find its acceleration (a) in general ; (b) at the end of 4 seconds. Ans. (a) «= 6t+ 2 ft. per (sec,)?; (b) a = 26 ft. per (sec.)? 12, The vertical component of velocity of a point at the end of t seconds is vy = 8 — 20+ 6 ft, per sec, Find the vertical component of acceleration (a) at any instant; (b) at the end of 2 seconds, Ans, (a) @y = 6t—2; (b) 10 ft, per (see.)? 18. If a point moves in a fixed path so that sav, show that the acceleration is negative and proportional to the cube of the velocity. 96 DIFFERENTIAL CALCULUS 14, If the space described is given by aet + be-, show that the acceleration is always equal in magnitude to the space passed over. 15, If a point referred to rectangular codrdinates moves so that © acost +b, and y= asint +c, show that its velocity has a constant magnitude. 16, If the path of a moving point is the sine curve % = at, bsinat, show (a) that the «-component of the velocity is constant; (b) that the acceleration of the point at any instant is proportional to its distance from the axis of X. 17. Given the following equations of curvilinear motion, find at the given instant Tey My, P} Gz, y, ; position of point (codrdinates) ; direction of motion, Also find the equation of the path in rectangular codrdinates, @) e=Ay=t; t=2 (g) ©=2sint, y = Beost; t= m in Seyler aie (h) 2 =sint, y = cos2t; t=3. © 3t=38. i (@ e=2t y=043; t=0, (@) @=2t,y=8e; t=0. © Gj) 2 = 86, y =logt; t=1, ® 2 (k) z=t,y=12t-1; t CHAPTER VII SUCCESSIVE DIFFERENTIATION 74. Definition of successive derivatives. We have seen that the derivative of a function of 2 is in general also a function of 2. This new function may also be differentiable, in which case the derivative of the first derivative is called the second derivative of the original function. Similarly, the derivative of the second derivative is called the third derivative ; and so on to the nth derivative. Thus, if y=Br', W120 a (ay z(2)- 2 A[4 (ay) _ 79 alz (3)|- 722, ete. 75. Notation. The symbols for the successive derivatives are usually abbreviated as follows: d (ay _ ay dc\dz)~ de” If y=f(@), the successive derivatives are also denoted by POs Or FNCOr FOn oy FOC): Peo PP ry YS «, 24), Sra, Sr, Sr@, + Zr. oT 98 DIFFERENTIAL CALCULUS 76. The nth derivative. For certain functions a general expression involving % may be found for the nth derivative. The usual plan is to find a number of the first successive derivatives, as many as may be necessary to discover their law of formation, and then by induction write down the nth derivative. Inuusrrative Examene 1. Given y = e®, find eu. Solution, : ana 2% Tuwusmnative Examene 2, Given y= log, find 5. Solution. ‘ : ay Tuuwsrmative Examrie 8, Given y=sine, find 72, Solution © cosa = sin ¢ + 2) - ot = cos = 7), 2 oe og + z= cos (2 +2) = sin (2 + =), 2 pa Zain | ae a i (e ae %). Ans, 77. Leibnitz's Formula for the nth derivative of a product. This formula expresses the nth derivative of the product of two variables in terms of the variables themselves and their successive derivatives. SUCCESSIVE DIFFERENTIATION 99 If u and » are functions of z, we have, from V, Sw = Boe uk. Differentiating again with respect to 2, du dudv , dudv , d’v_ d*u du dv dv Wee egae ae ae gods aeman ean ae Simin, "nytt, , Pu de , odudr, gdudo , dude, de S dx* dx dx da? dx dx dx? dx dx® dz Mu gdudv, gdudy, dv “wpa aa eS However far this process may be continued, it will be seen that the numerical coefficients follow the same law as those of the Binomial Theorem, and the indices of the derivatives correspond to the expo- nents of the Binomial Theorem.* Reasoning then by mathematical Induction from the mth to the (m+1)th derivative of the product, we can prove Leibnite's Formula a au | Pou de. n(n—t) atu dy 17) — (uv) = —v pa el a rUrmr—~—sssS caneae tae du atv |’ a a ae Tae 3 Intusrranive Exanrie 1, Given y = é loge, find ey by Leibnitz’s Formula. Solution. Let &, and v= loge; dod th fat, hen EA et a--2 @_ 2 a Substituting in (17), we get PY « ertogn 4 3&3 5 2 @(i0 z+e ae 2). a eT ge OS 2, fou ate * To make this correspondence complete, u and v are considered as al and = 100 DIFFERENTIAL CALCULUS Inuusrrative Exampie 2, Given y = 2%, find oe by Leibnita’s Formula. Solution. Let u=2, and v=ew; du dv Dl then eh Ba @u_, @y moh gant Pu @y gan” <5 = wear, au ony an dan = Substituting in (17), we get ay = share™ 4 2nar- Laer + n(n — 1)ar~Feer = ar 2eez [22a + Qnaz + n(n— 1]. = 78. Successive differentiation of implicit functions. To illustrate the : process we shall find oy from the equation of the hyperbola Be aty'= a8. Differentiating with respect to 2, as in § 63, p. 69, dy 282 —2a’y Y= 0, eae or, dy _ bx )) on ay Differentiating again, remembering that y is a function of 2, dy yb? — bina ay ae ae ay Substituting for 4Y its vatue from (4), : aly — ale (=) 2 @y) RCL ay? Sey eae ay _ dat ay ay But from the given equation, b’z*— a°y*= al? dye dat SUCCESSIVE DIFFERENTIATION 101 EXAMPLES Differentiate the following : “Ly = 408 — 622 4 4047. f= 102-0, a wa) = 2 S@= PO) = Tae 3. fy=¥. FAQ) =[8. 4. y= a logz. m8. ¢ _n(nt ie By=s. y= Be 6. y=(@—B)er + dren +2, Sere, 8. f(t) = ax? + be +e. 9. f(c) = log(e +1). 10. f(t) = log (@ + €-*), a r=sinaé, = tang, 13, r= logsin g. 14. f() = e-tcost. 15. (0) =Vs0e 20. 16. p= (72 + @)aretan!. a =ar. = log (1 +2). 19. y = cosac. 20, y =2"-Noge. 21. y= : aie y= 4e[(e— Her +a]. rel @peyat y= sie +e FB rreao iO) =- aa re) =— : em S 4. = = atsin ad = atr, S = 6sectg — 4sec%p. 1" = Deot p esc? F'\() =—4e- "cost =— 4f(). FO) = BF} —F0)- —_e ag (+e) oy = (log ay"ar. oY _ (ay cee ‘Ge | _ _ 2 — wom (+ *2) dry (m= Cr =2(- 194 2 Gp apt Hint. Reduce fraction to form — 14 ed before differentiating. +2 22. If y = eesina, prove that os fy, 23, If y = acos(log2) + bsin (log2), prove that 2¢—¥ 4 2% 4 y 0, ae ae 102 DIFFERENTIAL CALCULUS Use Leibnitz's Formula in the next four examples: 24. y= 2%, fu = a*(loga)"~2{(@ loga +n)? — n] 25. y= ex. Fe exe +m. 26. f(a) = esine. FO) = (V2)reesin ( [ =). 21. (8) = c0s.a8 cosdo. foon(gy = a+ = DY 08 [@ +004 3 +45 Meale-nes2} 28, Show that the formulas for acceleration, (14), (15), p. 92, may be written ey. de 29, y= dace, | 30. bia? + ay? = a0? _ i aye Ble yar, i 32. ty =a 33, ax? + 2hey + by? =1. oe. (ia + te Ca Sate 34. yf? —22y = a — (y—2)8" aes was 35. sec g cos tan? — tantg | tanto = 2(5 +86? + 864) 36. 6 =tan(p +6). -Aeeee. 87. Find the second derivative in the following: (a) log(u +x) =u—v. (e) y+ 2° —3ary=0, (b) + u=erto. (f) Y —2may + 2*9-a=0, (c) s= ltl. (g) y=sin@ +). @ e+st—e=0. (a) ety = ay, CHAPTER VIII MAXIMA AND MINIMA. POINTS OF INFLECTION. CURVE TRACING 79. Introduction. A great many practical problems occur where we have to deal with functions of such a nature that they have a greatest (maximum) value or a least (minimum) value,* and it is very important to know what particular value of the variable gives such a value of the function, For instance, suppose that it is required to find the dimensions of the rectangle of greatest area that can be inscribed in a circle of radius 5 inches. Consider the circle in the following figure : Inseribe any rectangle, as BD. Let CD= 2; then DE =V100 —23, and the area of the rectangle is evidently @ A=xV100—2, ‘That a rectangle of maximum area must exist may be seen as follows: Let the base CD (=2) increase to 10 inches (the diameter); then the altitude D&Z=V100 —a? will decrease to zero and the area will become zero. Now let the base decrease to zero; then the altitude will increase to 10 inches and the area will again become zero. It is therefore intuitionally evident that there exists a greatest rectangle. By a careful study of the figure we might sus- pect that when the rectangle becomes a square its area would be the greatest, but this would at best be mere guess- work. A better way would evidently be to plot the graph of the function (1) and note its behavior. To aid us in drawing the graph of (1), we observe that (a) from the nature of the problem it is evident that 2 and 4 must both be positive; and (b) the values of x range from zero to 10 inclusive. * There may be more than one of each, as illustrated on p. 109. 108 104 DIFFERENTIAL CALCULUS Now construct a table of values and draw the graph. What do we learn from the graph? 19.6 28.6 36.6 43.0 48.0 49.7 48.0 39.6 0.0 Secwrcaawwne (a) If carefully drawn, we may find quite accurately the area of the rectangle corresponding to any value of x by measuring the length of the corresponding ordinate. ‘Thus, when z= 0M=8 inches, then A= MP = 28.6 square inches 5 and when r= ON =4} inches, then A=NQ= about 39.8 sq. in. (found by measurement). (b) There is one horizontal tangent (RS). The ordinate 7H from its point of contact 7’ is greater than any other ordinate. Hence this discovery: One of the inscribed rectangles has evidently a greater area than any of the others. In other words, we may infer from this that the function defined by (1) has a mazimum value, We cannot find this value (= HZ) exactly by measurement, but it is very easy to find, using Calculus methods. We observed that at 7’ the tangent was horizontal; hence the slope will be zero at that point (Illustrative Example 1, p. 74). To find the abscissa of 7 we then find the first derivative of (1), place it equal to zero, and solve for 2. Thus om A=2V100—2, dA _100—227 dz V100—2? 100-22" _ 4 MAXIMA AND MINIMA 105 Solving, x=5V2, Substituting back, we get D#=V100—2 =5V2. Hence the rectangle of maximum area inscribed in the circle is a square of area os A=CD x DE=5V2 x 5V2=50 square inches. The length of HT is therefore 50. ; Take another example. A wooden box is to be built to contain 108 cu. ft. It is to have an open top and a square base. What must be its dimensions in order that the amount of material required shall be a minimum; that is, what dimensions will make the cost the least ? Let =length of side of square base in feet, and _y=height of box. Since the volume of the box is given, how- |i ever, y may be found in terms of 2. Thus volume = 2*y =1083 .-. y=. We may now express the number (=I) of square feet of lumber required as a function of z as follows: area of base = 2? sq, ft., and area of four sides = 4ay = a2 sq. fl. Hence 432 (©) M=274+— z My { | sort mp t x | ar { ort 1 | 438 1 2 | 220 =| 1 3 | 158 sso; 3 [am 2 ri | ' 6 | 108 el 118 7 | a mr | fod ae of | id 1 tot ' ' 10 | 143 ele ' ia oft sa is a formula giving the number of square feet required in any such box having a capacity of 108 cu. ft. Draw a graph of (2). 106 DIFFERENTIAL CALCULUS. What do we learn from the graph? (a) If carefully drawn, we may measure the ordinate correspond- ing to any length (=) of the side of the square base and so deter- mine the number of square feet of lumber required. (b) There is one horizontal tangent (RS). ‘The ordinate from its point of contact 7 is less than any other ordinate. Hence this dis- covery: One of the boxes evidently takes less lumber than any of the others. In other words, we may infer that the function defined by (2) has @ minimum value. Let us find this point on the graph ex- actly, using our Calculus. Differentiating (2) to get the slope at any point, we have 432 ee At the lowest point 7’ the slope will be zero. Hence 432 ee that is, when z= 6 the least amount of lumber will be needed. Substituting in (2), we see that this is M=108 sq. ft. The fact that a least value of If exists is also shown by the follow- ing reasoning. Let the base increase from a very small square to a very large one. In the former case the height must be very great and therefore the amount of lumber required will be large. In the latter case, while the height is small, the base will take a great deal of lumber. Hence M varies from a large value, grows less, then increases again to another large value. It follows, then, that the graph must have a “lowest” point corresponding to the dimensions which require the least amount of lumber, and therefore would involve the least cost. We will now proceed to the treatment in detail of the subject of maxima and minima. 80. Increasing and decreasing functions.* A function is said to be increasing when it increases as the variable increases and decreases as the variable decreases. A function is said to be decreasing when it decreases as the variable increases and increases as the variable decreases. ‘The proofs given here depend chiefly on geometric intuition. The subject of Maxima, and Minima will be treated analytically in § 108, p. 167. MAXIMA AND MINIMA 107 The graph of a function indicates plainly whether it is increasing or decreasing. For instance, consider the function @* whose graph (Fig. a) is the locus of the equation yore. a>1 As we move along the curve from left to right the curve is rising + that is, as 2 increases the function (= y) always increases. ‘Therefore a* is an increasing function for all values of 2. Fre. a Fie. b On the other hand, consider the function (a—2)* whose graph (Fig. 2) is the locus of the equation y=(@—2). Now as we move along the curve from left to right the curve is ‘falling ; that is, as x increases, the function (= y) always decreases. Hence (a—z)* is a decreasing function for all values of 2. : That a function may be sometimes increas- ing and sometimes decreasing is shown by the graph (Fig. c) of y=2a®— Gat +1203. As we move along the curve from left to right the curve rises until we reach the point 4, then it falls from A to B, and to the right of B it is always rising. Hence Fie. (a) from z=— @ tox =1 the function is inereasing ; (b) from x to x= 2 the function is decreasing ; (c) from x= 2 to x=+ 0 the function is increasing. | | 108 DIFFERENTIAL CALCULUS The student should study the curve carefully in order to note the behavior of the function when z=1 and x= 2 Evidently A and B are turning points. At 4 the function ceases to increase and com- mences to decrease; at B, the reverse is true. At A and B the tan- gent (or curve) is evidently parallel to the axis of X, and therefore the slope is zero. 81. Tests for determining when a function is increasing and when decreasing. It is evident from Fig. ¢ that at a point, as C, where a funetion y=f@ is inereasing, the tangent in general makes an acute angle with the axis of X; hence d slope = tant = 22. = f(a) = a positive number. dx Similarly, at a point, as D, where a function is decreasing, the tan- gent in general makes an obtuse angle with the axis of X; therefore slope = tan 7 = Wt = #1(2) = a negative number.* a Jn order, then, that the function shall change from an increasing to a decreasing function, or vice versa, it is a necessary and sufficient condition that the first derivative shall change sign. But this can only happen for a continuous derivative by passing through the value zero. Thus in Fig. ¢, p. 107, as we pass along the curve the derivative (=slope) changes sign at A and B where it has the value zero. In general, then, we have at turning points (18) The derivative is continuous in nearly all our important applica- tions, but it is interesting to note the case when the derivative (= slope) changes sign by passing through «.f This would evidently *Conversely, for any given value of 2, iff’ (e) =4, then f (2) is increasing ; Uf’ (2) =~, then f (x) is decreasing. When “(2) = 0, we cannot decide without further investigation whether f(z) is increas. ing or decreasin, '{ By this is meant that its reciprocal passes through the value zero. MAXIMA AND MINIMA 109 happen at the points B, HZ, G in the following figure, where the tangents (and curve) are perpendicular to the axis of X. At such exceptional turning points YY _ 912) = 03 Pas @=03 or, what amounts to the same thing, A 0 I@) 82, Maximum and minimum values of a function. A maximum value of a function is one that is greater than any values immediately preceding or following. A minimum value of a function is one that is less than any values immediately preceding or following. Fie. d For example, in Fig. c, p. 107, it is clear that the function has a maximum value Jf4 (= y = 2) when «=1, and a minimum value NB (=y =) when x= 2. The student should observe that a maximum value is not neces- sarily the greatest possible value of a function nor a minimum value the least. For in Fig. e it is seen that the function (= y) has values to the right of B that are greater than the maximum JA, and values to the left of A that are less than the minimum VB. A function may have several maximum and minimum values. Suppose that the above figure represents the graph of a function f@)- ‘At B, D, G, I, K the function is a maximum, and at C,H, H, Ja minimum. That some particular minimum value of a function may be greater than some particular maximum value is shown in the figure, the minimum values at C and H being greater than the maximum value at K. 110 DIFFERENTIAL CALCULUS At the ordinary turning points C, D, #, 1 J, K the tangent (or curve) is parallel to OX; therefore slope = & =fl@)=0. ‘At the exceptional turning points B, B, G the tangent (or curve) is perpendicular to OX, giving slope = = (2) =20. One of these two conditions is then necessary in order that the function shall have a maximum ora minimum value. But such a con- dition is not sufficient; for at # the slope is zero and at it is infinite, and yet the function has neither a maximum nor a minimum value at either point. It is necessary for us to know, in addition, how the function behaves in the neighborhood of each point. Thus at the points of maximum value, B,D, G, I, K, the function changes from an increasing to a decreasing function, and at the points of minimum value, C, B, H, J, the function changes from a decreasing to an increasing func- tion. Tt therefore follows from § 81 that at mazimum points slope = # = f"(a) must change from + to—, and at minimum points slope = % = f'(a) must change from — to + when we move along the curve from left to right. ‘At such points as A and F where the slope is zero or infinite, but which are neither mazimum nor minimum points, slope = # = f"(#) does not change sign. We may then state the conditions in general for maximum and minimum values of f (2) for certain values of the variable as follows: (19) f(x) is a maximum if f'(x) = 0, and f!(x) changes from + to—. 20) f(x) is a minimum if f'(~) = 0, and f'(x) changes from — to+. ; ‘The values of the variable at the turning points of a function are called critical values ; thus z=1 and z= 2 are the critical values of MAXIMA AND MINIMA 111 the variable for the function whose graph is shown in Fig. , p. 107. The critical values at turing points where the tangent is parallel to OX are evidently found by placing the first derivative equal to zero and solving for real values of z, just as under § 64, p. 73." To determine the sign of the first derivative at points near a par- ticular turning point, substitute in it, first, a value of the variable just a little less than the corresponding critical value, and then one a little greater.t If the first gives + (as at L, Fig. d, p. 109) and the second — (as at Jf), then the function (= y) has a maximum value in that interval (as at I). If the first gives — (as at P) and the second + (as at 1), then the function (= y) has a minimum value in that interval (as at C). If the sign is the same in both cases (as at Q@ and R), then the function (= y) has neither a maximum nor a minimum value in that interval (as at 7). We shall now summarize our results into a compact working rule. 83. First method for examining a function for maximum and mini- mum values. Working rule. Fimst Srer, Find the first derivative of the function. Ssconp Srer. Set the first derivative equal to zero$ and solve the resulting equation for real roots in order to find the critical values of the variable. Tump Srep. Write the derivative in factor forms. if it is algebraic, write it in linear form. Fourtu Strep. Considering one critical value at a time, test the first derivative, first for a value a trifle less and then for a value a trifle greater than the critical value. If the sign of the derivative is first + and then—, the function has a maximum value for that particular critical value of the variable ; but if the reverse is true, then it has a minimum value. If the sign does not change, the function has neither. * Similarly, if we wish to examine a function at exceptional turning points where the tan- gent is perpendicular to OX, we set the reciprocal of the first derivative equal to zero and solve to find critical values. In this connection the term “little less,” or "trifle less,”” means any value between the next smaller root (critical value) and the one under consideration; and the term “little greater,” or “trifle greater,” means any value between the root under consideration and the next larger one. }A similar discussion will evidently hold for the exceptional turning points B, Z, and 4 respectively. § When the first derivative becomes infinite for a certain value of the independent vari- able, then the function should be examined for such a critical value of the variable, for it may give maximum or minimum values, as at B, E, or A (Fig. d, p. 109). See footnote on p. 108, 112 DIFFERENTIAL CALCULUS In the problem worked out on p. 104 we showed by means of the graph of the function A=xV100—2? that the rectangle of maximum area inscribed in a circle of radius 5 inches contained 50 square inches. ‘This may now be proved ana- lytically as follows by applying the above rule. Solution. F(@) =2Vi00—a. First step. I(ey= is Second step. a oer 0, =a a=5Vv2, which is the critical value. Only the positive sign of the radical is taken, since, from the nature of the problem, the negative sign has no meaning. 2(5-V2—2) (5V2 +2), Third step. res -#) v2 +. mune ° VV (10 — 2) (10 + 2) Fourth step. When 2 <5V2, ti(e) = ZH) = V(4)(+) When «>5-V2, w= 2(-)(+) VOHG) Since the sign of the first derivative changes from + to — at « = 5 V2, the function has a maximum value 1(5-V2) = 6V3-5-V2 = 50, Ans. 84. Second method for examining a function for maximum and mini- mum values. From (19), p. 110, it is clear that in the vicinity of a maximum value of f(2), in passing along the graph from left to right, S'(®) changes from + to 0 to —. Hence f'(2) is a decreasing function, and by § 81 we know that its derivative, ie. the second deriy- ative [=/"(2)] of the function itself, is negative or zero. Similarly, we have, from (20), p. 110, that in the vicinity of a minimum value of f(x) F'(a) changes from — to 0 to +. Hence f’(x) is an increasing function and by § 81 it follows that f"(@) is positive or zero. MAXIMA ‘AND MINIMA 113 The student should observe that f(x) is positive not only-at mini. mum points (as at 4) but also at points such as P. For, as a point passes through P in moving from left to right, d ee slope = tant = a = f'(z) ts an inereasing function. he At such a point the curve is said to be concave upwards. Similarly, f"(2) is negative not only at maximum points (as at B) but also at points such as Q. For, as a point passes through Q, slope = tant = % =f"(2) ta a decreasing function, At such a point the curve is said to be concave downwards.* We may then state the sufficient conditions for maximum and mini- mum values of f(z) for certain values of the variable as follows: 21) f(x) is a maximum if f'(x) = 0 and f"(x) = a negative number. 22) (2) isa minimum if f'(x) =0 and f"(x) =a positive number. Following is the corresponding working rule. Finsr Srev. Find the first derivative of the function. Sxconp Strep. Set the first derivative equal to zero and solve the result- ing equation for real roots in order to find the critical values of the variable. Turep Step, Find the second derivative, Fourtn Srep. Substitute each critical value for the variable in the second derivative. If the result is negative, then the function is a maximum Sor that critical value; if the result is positive, the function is a minimum. When f(x) = 0, or does not exist, the above process fails, although there may even then be a maximum or a minimum; in that case the first method given in the last section still holds, being fundamental. Usually this second method does apply, and when the process of find- ing the second derivative is not too long or tedious, it is generally the shortest method. Let us now apply the above rule to test analytically the function Maa +32 x found in the example worked out on p. 105. * At a point where the curve is concave upwards we sometimes say that the curve has a positive bending, and where it is concave downwards a negative bending. 114 DIFFERENTIAL CALCULUS Solution. feyaate 2. First step. _ Second step. = 6, critical value. Third step. Se) = 24 e. Fourth step. (0) =+. Hence (6) = 108, minimum value. ‘The work of finding maximum and minimum values may frequently be simplified by the aid of the following principles, which follow at once from our discussion of the subject. (a) The maximum and minimum vabies of a continuous function must occur alternately. (b) When eis a positive constant, ¢-f (a) is a maximum or a minimum for such values of x, and such only, as make f (a) a maximum or a minimum. Hence, in determining the critical values of # and testing for max- ima and minima, any constant factor may be omitted. When e is negative, ¢-f(a) is a maximum when f(a) is a minimum, and conversely. (©) Ff ¢ is a constant, f(@) and e+ f(a) have maximum and minimum values for the same values of x. Hence a constant term may be omitted when finding critical values of @ and testing, In general we must first construct, from the conditions given in the problem, the function whose maximum and minimum values are required, as was done in the two examples worked out on pp. 103- 106. This is sometimes a problem of considerable difficulty. No rule applicable in all cases can be given for constructing the function, but in a large number of problems we may be guided by the following General directions. (a) Express the function whose maximum or minimum is involved in the problem. (b) If the resulting expression contains more than one variable, the conditions of the problem will furnish enough relations between the varia- bles so that all may be expressed in terms of a single one. MAXIMA AND MINIMA 115 (©) To the resulting function of a single variable apply one of our two rules for finding maximum and minimum values. (A) In practical problems it is usually easy to tell which critical value will give a maximum and which a minimum value, 80 it is not always necessary to apply the fourth step of our rules, (c) Draw the graph of the function (p. 104) in order to check the work, PROBLEMS 1. It is desired to make an open-top box of greatest, possible volume from asquare piece of tin whose side is a, by cutting equal squares out of the corners and then fold- ing up the tin to form the sides, What should be the length of a side of the squares cut out? Solution. Let then a2 side of small square = depth of box ; = side of square forming bottom of box, and volume is, V=(a—22)0; which is the function to be made a maximum by varying z. Applying rule, : First step. ve (—22)? — 4 (a — 22) = a? — Bax 4 120%, Second step. Solving a? — 8 ax + 12.2? = O gives critical values 2 = Sands It is evident from the figure that 2 = A must give a minimum, for then all the tin would be cut away, leaving no material out of which to make a box. By the usual 2 Gis found to give @ maximum volume “ test, + Hence the side of the square to be cut out is one sixth of the side of the given square. The drawing of the graph of the function in this and the following problems is left to the student. 2, Assuming that the strength of a beam with rectangular eross section varies directly as the breadth and as the square of the depth, what are the dimensions of the strongest beam that can be sawed out of a round log whose diameter is d? Solution, If z= breadth and y = depth, then the beam will have maximum strength when the function zy? is a maximum. From the figure, y? = d— 2?; hence we should test the function S(e) = a(@— 24). First step. f(t) =— 202 4 @ — 2? = 32%, Second step. d? — 82° E = critical value which gives a maximum, v3 ‘Therefore, if the beam is cut so that depth = -V¥ of diameter of log, and breadth = Vj of diameter of log, the beam will have maximum strength. , 116 DIFFERENTIAL CALCULUS 3. What is the width of the rectangle of maximum area that can be inscribed in a given segment 04.4’ of a parabola? Huxt. If OC=h, B= h~a and PP’=2y; therefore the area of rectangle PDD'P” is 2h=ay. But since P lies on the parabola y?=2pz, the funetion to be tested is 2(h—-2) V2pe. Z 7 Tj Z. V Ld Ans, Width = jh. 4, Find the altitude of the cone of maximum volume that can be inscribed in a sphere of radius r. B Hrvr, Volume of cone=$2%y. But2?= BCx CD=y (2r-y); there- fore the function to be tested is SW) PCr). ) ——— Ans, Altitude of cone = ¢r. Sa 5, Find the altitude of the cylinder of maximum volume that can be inscribed in a given right cone, Hint. Let AC=rand BC=h. Volume of cylinder= rey, But from similar triangles ABC and DB@ nh), A nm hihn=y. Hence the function to be tested is 2 LO=Fy a v Ans. Altitude = fh. 6, Divide a into two parts such that their product is a maximum, Ans. Each part 2 7. Divide 10 into two such parts that the sum of the double of one and square of ‘the other may be 2 minimum. Ans. 9 and 1. 8. Find the number that exceeds its square by the greatest possible quantity. Ans. 3. 9, What number added to its reciprocal gives the least possible sum? Ans. 1. 10, Assuming that the stiffness of a beam of rectangular cross section varies directly as the breadth and the cube of the depth, what must be the breadth of the stiffest beam that can be cut from a log 16 inches in diameter ? Ans. Breadth = 8 inches, 11. A water tank is to be constrneted with a square base and open top, and is to hold 64 cubic yards, If the cost of the sides is $1 a square yard, and of the bottom $2 a square yard, what are the dimensions when the cost is a minimum ? What is the minimum. cost ? Ans. Side of base = 4 yd., height = 4 yd., cost $06. 12, A rectangular tract of land is to be bought for the purpose of laying out a quarter-mile track with straightaway sides and semicircular ends. In addition a strip 85 yards wide along each straightaway is to be bought for grand atands, training quarters, ete. If the land costs $200 an acre, what will be the maximum cost of the land required ? Ans. $856. MAXIMA AND MINIMA Wt 18. A torpedo boat is anchored 9 miles from the nearest point of a beach, and it is desired to send a messenger in the shortest possible time to a military camp situated 15 miles from that point along the shore. If he cay walk 5 miles an hour but row only 4 miles an hour, required the place he must land. Ans, 3 miles from the camp. 14, A gas holder is a cylindrical vessel closed at the top and open at the bottom, where it sinks into the water. What should be its proportions for a given volume to require the least material (this would also give least weight) Ans. Diameter = double the height. 15, What should be the dimensions and weight of a gas holder of 8,000,000 cubie feet capacity, built in the most economical manner out of sheet iron of an inch thick and weighing 2} Ib, per sq. ft.? Ans. Height = 137 ft., diameter = 278 ft., weight = 220 tons. 16. Asheet of paper is to contain 18 sq. in. of printed matter, The margins at the top and bottom are to be 2 inches each and at the sides 1 inch each, Determine the dimen- sions of the sheet which will require the least amount of paper, Ans. 5 in, by 10 in, 17. A paper-box manufacturer has in stock a quantity of strawboard 30 inches by inches, Out of this material he wishes to make open-top boxes by entting equal squares out of each corner and then folding up to form the sides, Find the side of the square that should be cut out in order to give the boxes maximum volume. Ans. 3 inches. 18. A roofer wishes to make an open gutter of maximum capacity whose bottom and sides are each 4 inches wide and whose sides have the same slope. What should be the width across the top ? Ans. 8 inches, 4 19. Assuming that the energy expended in driving a steamboat through the water varies as the cube of her velocity, find her most economical rate per hour when steam- ing against a current running ¢ miles per hour. Hint. Let v= most economical speed ; then av? = energy expended each hour, a being a constant depending upon the partic- ular conditions, = actual distance advanced per hour. and ve 2 Hence a is the energy expended per mile of distance advanced, and it is therefore the funetion whose minimum is wanted. Ans. v= $e. 20. Prove that a conical tent of a given capacity will require the least amount of canvas when the height is -V2 times the radius of the base, Show that when the canvas is laid out flat it will be a circle with a sector of 152° 9 cut out. A bell tent 10 ft. high should then have a base of diameter 14 ft. and would require 272 sq. ft. of canvas. 21. A cylindrical steam boiler is to be constructed having a capacity of 1000 cu. ft. ‘The material for the side costs $2 a square foot, and for the ends $8 a syuare foot, Find radius when the cost is the least. 10 Ans. —— ft. Var 22, In the corner of a field bounded by two perpendicular roads a spring is situated 6 rods from one road and 8 rods from the other. How should a straight road be ran by this spring and across the corner so as to cut off as little of the field ae possible ? Ans. 12 and 16 rods from corner. ‘What would be the length of the shortest road that could be run across? Ans. (6i 4 8h! rods, 118 DIFFERENTIAL, CALCULUS, 23, Show that a square is the rectangle of maximum perimeter that can be inscribed in a given circle, : 24, Two poles of height a and b féet are standing upright and are c feet apart. Find the point on the line joining their bases such that the sum of the squares of the distances from this point to the tops of the poles isa minimum, Ans, Midway between the poles. When will the sui of these distances be a minimum ? 25. A conical tank with open top is to be built to contain V cubic feet. Determine the shape if the material used is a minimum, 26. An isosceles triangle has a base 12 in, long and altitude 10 in, Find the ree- tangle of maximum area that can be inscribed in it, one side of the rectangle coineid- ing with the base of the triangle. 27. Divide the number 4 into two such parts that the sum of the cube of one part and three times the square of the other shall have a maximuin value, 28. Divide the number a into two parts such that the product of one part by the fourth power of the other part shall be a maxtinum, 29. A can buoy in the form of a double cone fs to be made from two equal cireular iron plates of radius r, Find the radius of the base of the cone when the buoy has the greatest displacement (maximum volume). Ans. rV¥. 80, Into a full conical wineglass of depth a and generating angle a there is care- fully dropped a sphere of such size as to cause the greatest overflow. Show that the radius of the sphere is cane sina-+ cos2a" 31. A wall 27 ft. high is8 ft. from ahouse, Find the length of the shortest ladder that, will reach the house if one end rests on the ground outside of the wall, Ans, 18-V13, 82, A vessel is anchored 3 miles offshore, and opposite a point 5 miles further along the shore another vessel is anchored 9 miles from the shore. A boat from the first vessel is to land a passenger on the shore and then proceed to the other vessel. What is the shortest course of the boat ? Ans, 18 miles, 33, A steel girder 26 ft. long is moved on rollers along a passageway 12.8 ft. wide and into a corridor at right angles to the passageway. Neglecting the width of the girder, how wide must the corridor be? Ans, 5.4 ft, 34, A miner wishes to dig a tunnel from a point A to a point B 800 feet below and 600 feet to the east of A. Below the level of A it is bed rock and above A is soft earth, If the cost of tunneling through earth is $1 and through rock $3 per linear foot, find the minimum cost of a tunnel. Ans, $1348.58. 35. A carpenter has 108 sq. ft. of lumber with which to build a box with a square base and open top. Find the dimensions of the largest possible box he can make, Ans, 6x 6x 8, 36. Find the right triangle of maximum area that can be constructed on a line of length 4 as hypotenuse, h Ans. length of both legs. v2 37. What is the isosceles triangle of maximum area that can be inscribed in a given circle ? Ans. An equilateral triangle, 38, Find the altitude ofthe maximum rectangle that can be inscribed in a right triaugle with base b and altitude h, h Ans, Altitud MAXIMA AND MINIMA 119 39. Find the dimensions of the rectangle of maximum area that can be inscribed in the ellipse ba? + a%y? = ab. Ans. aV2 andb V2; area = 2ab. 40. Find the altitude of the right cylinder of maximum volume that can be inscribed in a sphere of radius r. 2r Ans. Altitude of cylinder ==". V3 3 41, Find the altitude of the right cylinder of maximum convex (curved) surface that can be inscribed in a given sphere. Ans. Altitude of cylinder = r-V2. 42, What are the dimensions of the right hexagonal prism of minimum surface whose volume is 86 cubic feet ? Ans. Altitude = 2-V3; side of hexagon = 2. 43, Find the altitude of the right cone of minimum volume circumscribed about a given sphere. Ans. Altitude = 47, and volume = 2 x vol. of sphere. 44. A right cone of maximum volume is inscribed in a given right cone, the vertex of the inside cone being at the center of the base of the given cone, Show that the altitude of the inside cone is one third the altitude of the given cone. 45. Given a point on the axis of the parabola y? = 2 pz at a distance a from the vertex ; find the abscissa of the point of the curve nearest to it. Ans. @=a—p. 46. What is the length of the shortest line that can be drawn tangent to the ellipse 2? + ay? = ab? and meeting the codrdinate axes? Ans. a+b. 47. A Norman window consists of a rectangle surmounted by a semicircle. Given the perimeter, required. the height and breadth of the window when the quantity of light admitted is a maxtinum. Ans. Radius of circle = height of rectangle. 48. A tapestry 7 feet in height is hung on a wall so that its lower edge is 9 feet above an observer's eye. At what distance from the wail should he stand in order to obtain the most favorable view ? Ans. 12 feet. Hixr. The vertical angle subtended hy the tapestry in the eye of the observer must he ata maximum, 49, What are the most economical proportions of a tin ean which shall have a given capacity, making allowance for waste ? 2Vv38 Ans, Height = 23 x diameter of base. 7 Hivr. There is no waste in eutting ont tin for the side of the can, but for top and bottom a hexagon of tin circumscribing the eireular pieces required is used up. Nore 1, If no allowance is made for waste, then height = diamete Nore 2. We know that the shape of a hee cell is hexagonal, giving a certain capacity for honey with the greatest possible economy of wax. 50. An open cylindrical trough is constructed by bending a given sheet of tin of breadth 2a, Find the radius of the cylinder of which the trough forms a part when the capacity of the trough is a maximum, 2 fae saa Ans, Rad, = ="; i.e, it must be bent in the form of a semicircle 51. A weight Wis to be raised by means of a lever with the force F at one end and the point of support at the other, If the weight is suspended from a point at a distance a from the point of support, and the weight of the beam is w pounds per linear foot, what should be the length of the lever in order that the force required to lift it shall be a minimum ? : Ans. ©=4 ; w 120 DIFFERENTIAL CALCULUS 52. An electric are light is to be placed directly over the center of a cfroular plot of grass 100 feet in diameter. Assuming that the intensity of light varies directly as the sine of the angle under which it strikes an illuminated surface, and inversely as the square of its distance from the surface, how high should the light be hung in order that the best possible light shall fall on a walk along the circumference of the plot? 7 oe ee Va \ 83, The lower corner of a leaf, whose width is a, is folded over so as just to reach the inner edge of the page. (a) Find the width of the part folded over when the length of the crease is a minimum, (b) Find the width when the area folded over isa minimum, Ans. (a) $a; (b) $a. 54, A rectangular stockade is to be built which must have a certain area. If a stone wall already constructed is available for one of the sides, find the dimensions which would make the cost of construction the least. Ans. Side parallel to wall = twice the length of each end. 5B. A cow is tethered by a perfectly smooth rope, a slip noose in the rope being thrown over a large square post, If the cow pulls the rope taut in the direction shown in the figure, at what angle will the rope leave the post ? Ans, 30°, 56, When the resistance of air is taken into account, the inelination of a pendulum to the vertical may be given by the formula, 0 = ae-H cos (nt +). Show that the greatest elongations occur at equal intervals = of time. 57. It is required to measure a certain unknown magnitude « with precision, Suppose that n equally careful observations of the magnitude are made, giving the results a a rr ‘The errors of these observations are evidently B— dy Bay Z—ady + some of which ave positive and some negative, It has been agreed that the most probable value of « is such that it renders the sum of the squares of the errors, namely (= a)? + = a3)? + (@— a)? $0 + (= )?, a minimum. Show that this gives the arithmetical mean of the observations as the most probable value of x. B 58, ‘The bending moment at B of a beam of length 1, uniformly loaded, is given by the formula M=4}ule— }wz?, where w = load per unit length. Show that the maximum bending moment is at the center of the beam, 59. If the total waste per mile in an electric conductor is 2 waertt, [c = constant] Where c= current in amperes, = resistance in ohms per mile, and ¢ =a constant depending on the interest on the investment and the depreciation of the plant, what is the relation between c, r, and t when the waste is a minimum ? Ans. cr=t. MAXIMA AND MINIMA 121 60. A submarine telegraph eable consists of a core of copper wires with a covering made of nonconducting material. If z denote the ratio of the radius of the core to the thickness of the covering, it is known that the speed of signaling varies as a) 7 1 Show that the greatest speed is attained when 2 = 7 ‘e 61. Assuming that the power given out by a voltaic cell is given by the formula ER P=——, w+ where E = constant electromotive force, r= constant inter nal resistance, prove that P is a maximum when r= R. al resistance, 2 = exter- 62. The force exerted by a circular electric current of radius @ on a small magnet whose axis coincides with the axis of the circle varies as. x (@+ah where « = distance of magnet from plane of circle. Prove that the force is a maxi- a mum when 2 = 5- 63. We have two sources of heat at A and B with intensities a and b respectively. ‘The total intensity of heat at a distance of z from A is given by the formula cD 2 Z ¢ x)? : Show that the temperature at P will be the lowest when d-2 % @ that is, the distances BP and AP have the same ratio as the cube roots of the corre- sponding heat intensities. The distance of P from A is ata at ob 64. The range OX of a projectile in a vacuun is given by the formula v2sin2¢. =o initial velocity, g= acceleration due to grav- ity, @ = angle of projection with the horizontal. Find the angle of projection which gives the greatest range for a given initial velocity. Ans. $= 45°. a 65. The total time of flight of the projectile in the last problem is given by the formula Qu, sing gy T At what angle should it be projected in order to make the time of flight a maximum ? Ans. $= 90 122 DIFFERENTIAL CALCULUS 66. The time it takes a ball to roll down an inclined plane AB is given by the fortaula, _— a a yA. T=24/—*_. geindg Neglecting friction, etc., what must be the value of @ to make the quickest descent ? Ans. ¢ = 45°. Ife 67; Examine the function (¢—1)*(¢ +1) for maximum and minimum values, Use the first method, p. 111. Solution. f(e) = (@ —1)2(@ +1). First step. 7'(@) = 2(e —1)(e +1)? +3(@—1)2(¢ +1)? =(@— Ne + 1)%(52—1). Second step. (t —1) (e + 1)2(5¢—1) =0, w=1, —1, }, which are critical values. Third step. F’'@) = 5(@—-1)(@+1%@-}). Fourth step. Examine first for critical value = 1(C in FE) hen 2 <1, Se) = 5(-) (HCE When 2>1, (2) = 6(4)(4)2(4) = +. ‘Therefore, when # = 1 the function has a minimum value f(1)= 0 (= ordinate of C). Examine now for the critical value « = } (B in figure). When 2 <4, (2) = 5(—)(+)*(-) = +. When 2 > 4, f’(e) = 5(—)(+)2(+) = —- ‘Therefore, when # = 4 the function has a maximum value f(}) = 1.11 (= ordinate of B). Examine lastly for the critical value « =— 1 (A in figure). When 2<—1, f(a) = 5(—)(—)*(— When 2 > —1, f’(e) = 5(—) (+)?(—| ‘Therefore, when z =— 1 the function has neither a maximum nor a minimum value. 68. Examine the function a — b(¢— c)# for maxima and minima, Solution. F(a) = a—b@— oft, 2b I'@)=-——. B(e— ot Since z= is a critical value for which /”(z) =e, but for which f (2) is not infinite, let us test the function for maximum and minimum values when z= c. When ao, f’(2) Hence, when z = ¢ = OM the function has a maximum value £(¢) ae Examine the following functions for maximum and minimum values: 69. (« — 8)? (@— 2). Ans, « =}, gives max. = sy; 2 = 8, gives min. = 0. 10 (@—1)(@— 2% w= §, gives max. = .08456; x = 2, gives min, x= 1, gives neithe MAXIMA AND MINIMA 123 TL. (e— 4) (e + 2. Ans, 2 =— 2, gives max.; 2 = 3, gives min.; 2 =4, gives neither, 72. (e — 28 (2a + 1). 2=— }, gives max.; 2 = 4), gives min; 2 =2, gives neither. 73. (2 +)i@— 5). a =}, gives max.; 2=—1and 5, give min, 74, Qe —wte— ah. “ gives max.; =a, gives min; “ » gives neither, 75. e(e— 1)? +1) 4, gives max.; Land — 4, give min.; © =—1, gives neither. 76, x(a +2)?(4 —2)*. —aand P give max.; oc 7 = 5» gives min; a, gives neither, 77. V4 e(e— aye a, gives min, 78, a—b(e—o)*. ‘No max, or min, 2 6 79. eatete. 2 = 4, gives max.; . x =16, gives min. ca £, gives'min. so, f= 78 1—-2+2 i 31, ——_——. a = }, gives min. 8. Se i gi * 82. ae 2 = V3, gives min, = 12-V3—17: e480 : : a =—V2, givesmax, =—12-V2—17; a=—1, — 2, give neither, 83. Goats ead gives max. = a. 2 a OB “, gives min. a a 2=—“_, gives max. att 124 DIFFERENTIAL CALCULUS 85, Examine 2? — 82% — 92 + 5 for ma: a and minima, Use the second method, p. 113. Solution. f(t) =x Bat 945, First step. S'(@) = 822 — ba —9. Second step. 32? —62—9=0; hence the critical valuesare = =— Land 3. Third step. (a) = 62-6. Fourth step. f’(—1) =— 12. +. f(—1) = 10 = (ordinate of A) = maximum value, 78) =+ 12. . f(8) =— 22 (ordinate of B) inimum value, 86, Examine sin?e cose for maximum and minimum values. Solution. Sf (2) = sin?a cosa. First step. f’(e) = 2sinz cost — Second step. 2 sina costa — sin? = 0; Wa, hence the critical values are cour and =n + arctan V2=n7 ba. Third step. F(a) = cos (2 cosa — 7 sin®z), Fourth step. (0) = +. ~. f(0) = 0 = minimum value at 0. ©. £(#) = 0 = maximum value at C. &. F(a) = maximum value at A. f(r — a) = minimum value at B, ete, Examine the following functions for maximum and minimum values. 81. 328 — 92? — 272 + 80, Ans, © =—1, gives max. = 45; 2 = 8, gives min, =— 51. 88, 22° — 2127 + 362 — 20. a =1, gives max.=— 38; 2 =6, gives min, =— 128, 8 89, Froese th 2 =1, gives max. 2 8, gives min, 90. 22° — 152% + 86x +10. = 2, gives max, = 88; = 8, gives min. = 37, 91. a — 9a? + 152-3. , gives max. = 4; 2 = 5, gives mi 28, 92, a? — 82? + 62 +10, No max, or min, 93, 26 — Sat 4 52% 41. 2 =1, gives max, z= 8, gives min, x = 0, gives neither. 94. 325 — 1252% 4 21602, 2 =—4and 8, give max.; a2=—3and 4, give min. 95, 228 — 8at— 12044, 98. at — 4, 96. 22* — 212? + 862 — 20. 99, 2 — 8, 97, at — 22% +10. 100. 4— 2°, 101, 102, 103. 104. 105. 106, 107. 108, 109, 110. mm. 112. 13. 14, 15. 85. called MAXIMA sinz (1+ cosz). loge log cos. ack= + be Bs, sin2z—z. z+ tang. sin? cosa. x eosa. sing + cos2a. 2tang — tanta. sing T+ tang 2 1+atang Ans. 2a 2nm +7, give max. AND MINIMA 125 ™ a= 2nm—F, give min, z= ne, give neither, =e, gives min, =e; z= 1, gives neither. x= nm, gives max, i log e gives min. = 2-Vab. z is gives min. a =e, gives max. a= D gives max. = V2; 2 =, gives min. =— V2, = » gives max,; 6 mo 2=—F: gives min, No max. or min. © 3 ganr +, gives ==V3; +E, gives max. = 5 = nm —F, gives min, = nm, gives neither. z= cota, gives max. are sin}, gives max,; = ‘i a= > gives min, = 7? gives max, ™, gives max. a8 . cosz, gives max. — cos, gives min. Points of inflection. Definition. Points of inflection separate arcs concave upwards from ares concave downwards.* Thus, if a curve y=J(@) changes (as at B) from concave upwards (as at A) to con- cave downwards (as at C’), or the reverse, then such a point as B is a point of inflection. * Points of inflection may also be defined as points where or ay ®y : 2H gana 2 on @) F¥-o.and SY changes sign, ) Fen 0 and 7m changes sign. 126 DIFFERENTIAL CALCULUS From the discussion of § 84 it follows at once that at 4, f"(r)=+, and at C,f"(@)=—. In order to change sign it must pass through the value zero;* hence we have 3) at points of inflection, f(x) = 0. Solving the equation resulting from (28) gives the abscissas of the points of inflection. To determine the direction of curving or direc- tion of bending in the vicinity of a point of in- @ flection, test f"() for values of 2, first a trifle less and then a trifle greater than the abscissa at that point. Iff"(2) changes sign, we have a point of in- flection, and the signs obtained determine if the curve is concave upwards or concave downwards in the neighborhood of each point of inflection. ‘The student should observe that near a point where the curve is concave upwards (as at 4) the curve lies above the tangent, and at a point where the curve is concave downwards (as at ()) the curve lies below the tangent. At a point of inflection (as at B) the tangent: evidently crosses the curve. Following is a rule for finding points of inflection of the curve whose equation is y=/(2). This rule includes also directions for examining the direction of curvature of the curve in the neighborhood of each point of inflection. y| x ¥Fmsr Srer. Find f"(2). Srconp Sruv. Set f"(2) =0, and solve the resulting equation for real roots. Tump Srer. Write f(a) in factor form. Fourrn Srep. Test f"(2) for values of 2, first a trifle less and then'a trifle greater than each root found in the second step. If f!(x) changes sign, we have a point of inflection. When f(x) =+, the curve is concave upwards \+7.1 When fll(x)=—, the curve is eoncave downwards —~. * It is assumed that /’(z) and f”() are continuous. The solution of Ex. 2, p. 127, shows how to discnss a case where f(x) and f’”(z) are both infinite. Evidently salient points (see p. 258) are excluded, since at such points /”(2) is discontinuous. + This may be easily remembered if we say that a vessel shaped like the curve where it is concave upwards will hold (+) water, and where it is concave downwards will spill (-) water. MAXIMA AND MINIMA 127 EXAMPLES Examine the following curves for points of inflection and direction of bending. 1. y= 8at— 42341. Solution. S(@) =8at— 429 41. First step. 36a? — 24a, Second step. 6, critical values. Third step. 360(2 — %). Fourth step, When « <0, f(a) 1 curve is concave upwards to the left and concave downwards to the right of 2 = 0 (A in figure). When 2 <4, f”(c) =—; and when #>4, (2) =+. ©. curve is concave downwards to the left and concave upwards to the right of a= § (Bin figure). ‘The curve is evidently concave upwards everywhere to the left of A, concave dow wards between A (0, 1) and B (3,43), and concave upwards everywhere to the right of B. 2. (y— 2) =(@— 4). +5 and when #>0, f(t) =—. Solution. y=24(e— 4h, x First step. & ie —4y 8, fi2) $e ie 48, q = Second step. When « = 4, both first and second derivatives are infinite, Third step. When x <4, fy ; but when @>4, oe ‘We may therefore conclude that the tangent at (4, 2) is perpendicular to the axis of X, that to the left of (4, 2) the curve is concave upwards, and to the right of (4, 2) it is concave downwards. ‘Therefore (4, 2) must be considered a point of inflection. =a, Ans. Concave upwards everywhere. = 5-22-24, Concave downwards everywhere. bya, Concave downwards to the left and concave up- wards to the right of (0, 0). 6. y= 829-9249, Concave downwards to the left and concave up- wards to the right of (1, — 2). =a4+@—Hs Concave downwards to the left and concave up- wards to the right of @, a). 8 ay = ? — ax? + 2a, Concave downwards to the left and concave up- wards to the right of (« 4), 9% yaa, Concave upwards everywhere. 10. y= a4 — 1228 4 480% Concave upwards to the left of ¢ = 2, concave downwards between «= 2 and z = 4, concave upwards to the right of x = 4. LL. y=sing, Points of inflection are z = nz, n being any integer. | | | 128 DIFFERENTIAL CALCULUS 12, y = tana, Ans. Points of inflection are 2=nz, n being any integer. 13. Show that no conic section can have a point of inflection. 14, Show that the graphs of e and log have no points of inflection, 86. Curve tracing. ‘I'he elementary method of tracing (or plotting) a curve whose equation is given in rectangular codrdinates, and one with which the student is already familiar, is to solve its equation for y (or 2), assume arbitrary values of x (or y), calculate the correspond- ing values of y (or 2), plot the respective points, and draw a smooth curve through them, the result being an approximation to the required curve. This process is laborious at best, and in case the equation of the curve is of a degree higher than the second, the solved form of such an equation may be unsuitable for the purpose of computation, or else it may fail altogether, since it is not always possible to solve the equation for y or «. The general form of a curve is usually all that is desired, and the Calculus furnishes us with powerful methods for determining the shape of a curve with very little computation. The first derivative gives us the slope of the curve at any point; the second derivative determines the intervals within which the curve is concave upward or concave downward, and the points of inflection separate these intervals; the maximum points are the high points and the minimum points are the low points on the curve. As a guide in his work the student may follow the Rule for tracing curves. Rectangular codrdinates. Finsr Srep. Find the first derivative ; place it equal to zero ; solving gives the abscissas of maximum and minimum points. Srconp Srep. Find the second derivative; place it equal to zero ; solv- ing gives the abscissas of the points of inflection. Turp Step, Calculate the corresponding ordinates of the points whose abscissas were found in the first two steps. Calculate as many more points as may be necessary to give a good idea of the shape of the curve. Fill out a table such as is shown in the example worked out. Fourra Srp, Plot the points determined and sketch in the curve to correspond with the results shown in the table. If the calculated values of the ordinates are large, it is best to reduce the scale on the Y-axis so that the general behavior of the curve will be shown within the limits of the paper used. Coérdinate plotting paper should be employed. MAXIMA AND MINIMA 129 EXAMPLES Trace the following curves, making use of the above rule, Also find the equations of the tangent and normal at each point of inflection. 1. y=o— 9a? + 242-7. Solution. Use the above rule. First step. y= 82 — 182424, 8221824 24=0, 2=2,4, Second step. y’ = 62-18, 62—18=0, 2=8 Third step. « | v | w | v7 | Remarks | Dingoriox or Conve : a ‘ = — } concave down 3 yak - oO pt. of infil, 4 9 0 + min, feoncave up 6 29 + + J Fourth step. Plotting the points and sketching in the curve, we get the figure shown, ‘To find the equations of the tangent and normal to the curve at the point of inflec- tion P,(8, 11), use formulas (1), (2), pp. 76, 77. This gives 8 + y = 20 for the tangent and 8y—« = 30 for the normal. 2 y =a) — 62? — 36245. Ans. Max. (— 2, 45); min, (6, — 211); pt. of infl. @, — 88); tan. y + 48¢—18=0; nor, 48y — 2 + 3986 =0. 3. y=at— 2a 410, Ans. Max. (0, 10); min, (41, 9); pt. of infl. 1 88 (9) 4, y=4ot— Bat 42, Ans. Max. (0, 2); min. (+ V8, — §); pt. of infl. (£1,-2)- 6a +a Ans, Max. (1,3); min, (28 _ ¥) 2 1, — 8); pt. of infl, (0, 0), 6 y= 12-2 Ans. Max, (2, 16); min, (—2, —16); pt. of inf, (0, 0). 1. dy to — 822 44=0. Max. (2,0); min, (0, — 1). 180 wy 13, Wy 15. 16. y= aw. 18. 19. DIFFERENTIAL CALCULUS y= 828-92 49, . 2y +a? —92 $6 =0. 10. 11. yao — 6a? 1ha 42. y(l + 2) y=@+Di@—5 42 aett. yao — B22 — 22, y =18 + 862 — 82% — 228, y=a—2eose. 20. y= 84-23, 21. y=a— 9a? 4152-3, 22, dy =442, 23, 4y = at — 622 +5, a Aue 25. yasine +5. Wet z 26, 21, y = 62 — 2a? — pad, 28, yn it®, Qa 29, y=2—2sing. 30. y = log cos. 81. y = log(1 + 2%). CHAPTER IX DIFFERENTIALS 87. Introduction. Thus far we have represented the derivative of y=F(2) by the notation — gy =f'(2). Ya y@ We have taken special pains to impress on the student that the symbol a dx. was to be considered not as an ordinary fraction with dy as numerator and dz as denominator, but as a single symbol denoting the limit of the quotient, Ay Ar as Az approaches the limit zero. Problems do occur, however, where it is very convenient to be able to give a meaning to dz and dy separately, and it is especially useful in applications of the Integral Calculus. How this may be done is explained in what follows. 88. Definitions. If f"(2) is the derivative of f(«) for a particular ivalue of 2, and Az is an arbitrarily chosen increment of 2, then the differ- ential of f(@), denoted by the symbol df (2), is defined by the equation 4 af (@)=f'@) Ax. If now f(z) =a, then f"(x) =1, and (A) reduces to dz=Az, showing that when « is the independent variable, the differential of 2(= de) is identical with Az. Henee, if y=f(2), (A) may in general be written in the form @) dy = f'(x) de® * On account of the position which the derivative /”(z) here occupies, it is sometimes called the differential coeficient. ‘The student should observe the important fact that, since de may be given any arbi- trary value whatever, dic is independent of =. Hence, dy is a function of two independent variables z and dz. 181 132 DIFFERENTIAL CALCULUS, The differential of a function equals its derivative multiplied by the differential of the independent variable. Let us illustrate what this means geometri- cally. Let f'(x) be the derivative of y=f(x) at P. Take dx = PQ, then dy =S'@ite=tanr- PQ =LT. PQ = Or. ‘Therefore dy, or df(z), is the increment (= QT) of the ordinate of the tangent corresponding to dz.* This gives the following interpretation of the derivative as a fraction. If an arbitrarily chosen increment of the independent variable x for a point P(a, y) on the curve y=f(x) be denoted by dx, then in the derivative : "(a) = tan 7, dy denotes the corresponding increment of the ordinate drawn to the tangent. 89. Infinitesimals. In the Differential Calculus we are usually con- cerned with the derivative, that is, with the ratio of the differentials dy and dz, In some applications it is also useful to consider dx as an infinitesimal (see § 15, p. 18), that is, as a variable whose values remain numerically small, and which, at some stage of the investiga- tion, approaches the limit zero. Then by (B), p. 131, and (2), p. 19, dy is also an infinitesimal. In problems where several infinitesimals enter we often make use of the following Theorem. In problems involving the limit of the ratio of two infinites- inals, either infinitesimal may be replaced by an infinitesimal so related to it that the limit of their ratio is unity. Proof. Let a, 8, a’, 8’ be infinitesimals so related that (a) limit amit =1and limit > B * The student should note especially that the differential (= dy) and the increment (= Ay) of the function corresponding to the same value of dz (= At) are not in general equal. For, in the figure, dy=Q7, but Ay= QP”. DIFFERENTIALS 133 We have a7 a identically, and limit = lini limit 5 - vim Th. II, p. 18 sting il, By (¢) (@) os limit ao limit e QED. Now let us apply this theorem to the two following important limits. For the independent variable 2, we know from the previous section that Ax and dz ave identical. Hence their ratio is unity, and also limit oa above theorem, (B) In the limit of the ratio of Ax and a second infinitesimal, Ax may be replaced by dv. 1. That is, by the On the contrary it was shown that, for the dependent variable y, Ay and dy are in general unequal. But we shall now show, however, that in this case also 7 ras limit zr limit Ay _ Since =0Ar@ 2), we way write oe =fl@ +6 where e is an infinitesimal which approaches zero when Az +0. Clearing of fractions, remembering that Ax = dz, Ay=f'(a) dz +e-Az, or Ay=dy+e-Az. (B), p. 131 Dividing both sides by Ay, a ate a Ay Ay limit dy _ =1, “ Ar=0 Ay or and hence limit, g- 1. That is, by the above theorem, (FP) In the limit of the ratio oPay and a second infinitesimal, Ay may be replaced by dy. 184 DIFFERENTIAL CALCULUS 90. Derivative of the arc in rectangular codrdinates. Let » be the length* of the arc AP measured from a fixed point A on the curve. Denote the increment of s (=are PQ) by As. ‘The definition of the length of are depends on the assumption that, as Q approaches P, Timit (BREE Q) _ arePQ ) If we now apply the theorem on p. 182 to this, we get (G) In the limit of the ratio of chord PQ and a second infinitesimal, chord PQ may be replaced by are PQ(=As). From the above figure (D (chord PQ)?=(Az)?-+ (Ay) Dividing through by (Az)*, we get chord PQ\*_, , /Ay\? om Er nah Now let Q approach P as a limiting position; then Ax 0 and we have 5 ds\_ 5, (dy fey) [sce 2 (AML) 8). ny cn] Bul, (BY, (24) ae 1+(z): Similarly, if we divide (27) by (Ay)? and pass to the limit, we get . ds dx 25) S= 1. - dy (3) 7 Also, from the above figure, Az : Ay 6 ne oe ose Chord PQ?” = chord PQ” Now as @ approaches P as a limiting position + 7, and we get a 26) csr =”, snr =. as as [since from (@) timit 22 7o7 mit ete" Ynnie Y= a. * Defined in § 209. DIFFERENTIALS 135 Using the notation of differentials, formulas (25) and (26) may be written [1a (2 an aa[1+(2) fe ax! (28) ds = () + | dy. dy, Substituting the value of ds from (27) in (26), wy 1 dx (29) cos 7 = —_—__, sint = ay anh [+] [*+(a)] An easy way to remember the relations (24)-(26) between the differentials dz, dy, ds is to note that they are as correctly represented by a right triangle whose ay : ; os hypotenuse is ds, whose sides are dz and dy, and whose angle at the base is 7. ‘Then de = Vay + yy, a = and, dividing by dx or dy, gives (24) or (25) respectively. Also, from the figure, i. cos T = sing =—}3 ds the same relations given by (26). 91. Derivative of the arc in polar codrdinates. In the derivation which follows we shall employ the same figure and the same notation used on pp. 83, 84, From the right triangle PRQ (chord PQ)? =(PRY+ (RO? =(psin A8)*+(p + Ap —p cos A0)*. oe Dividing throughout by (A8)%, we get chord PQ\?__/sin A@\?, /Ap , 1—cos A0\* ( ae y=" )+(s5+e: a0 ) 136 DIFFERENTIAL CALCULUS Passing to the limit as A@ diminishes towards zero, we get * ae e4 (40, (io) -#+(3) In the notation of differentials this becomes 1 ‘dpi (31) ds=| p?4(-) | da. o [r+ These relations between p and the differentials ds, dp, and dO are correctly represented by a right triangle Jin whose hypotenuse is ds and whose sides are dp and pd. Then =V(pd0)* + dp)’, * and dividing by d@ gives (30). Denoting by ¥ the angle between dp and ds, we get at once (30) tan p=p & , which is the same as (A), p. 84. Iunvsrrative Exampe 1. Find the differential of the arc of the circle 2? +y2= 12. dy ao i. ‘To find ds in terms of + we substitute in (27), giving . * a= [i+ apa - [eee fen = ke au, ¥. ¥ y. r= a ‘To find ds in terms of y we substitute in (28), giving as=[14 Pay = [£44 fa = [2] ZS Inwstramive Examrre 2. Find the differential of the are of the cardioid p= (1 — cos) in terms of 8. dp Solution, Differentiating, of = asind. Substituting in (81), gives ds = [a2(1— cosd)?-+ a® sin?9}.d9 = a[2—2cos6}td9 = a[ ssines st a= asin Sas. Solution. Differentiating, By (@), p. 134 limit chord PQ jj 0=0 Ag 1 By §22, p.2t si 1-cosA0_jimit 2. seat — limit =0-1=0, By39,p.2,and §22, p21 DIFFERENTIALS 137 EXAMPLES Find the differential of arc in each of the following curves: +2 Ans, ds=+ + ea, = ds =V1+ 4a%a? dx, ds=Vi+ Oa dr, ds = 4V44 Oy dy. sla ds= fay. vie eee b=. ds = seexdr. 8. p= acosd. ds = ado. 9. p? =a? cos 28. ds = aVa0c26 a0. 10. p = acderte, ds = pesca dd. 1. p=ar, ds = a9 V1 + logtads. 12. p= a8. 13, (a) a? — PS (hy aby baad. (i) Y= ax. G) (ie) dea. W p= asec’. () p=2atand sind. 8 (m) p=14 sind. (g) p= asec? 5. (n) e =a. 92. Formulas for finding the differentials of functions. Since the differential of a function is its derivative multiplied by the differen- tial of the independent variable, it follows at once that the formulas for finding differentials are the same as those for finding derivatives given in § 83, pp. 34-86, if we multiply each one by dz. This gives us I d(e)=0. sa d(x) = de. ur dQutv—w)=dut dv—dw. IV d(ov) = edo. Vv dw) =udv+vdu VI dv") =nv™-* do. 138 DIFFERENTIAL CALCULUS Via d(2") = nw" de, ‘u vdu—udv vil a(5)= edad, Vila a(*)= du, eye vill d(log,») = log,e ®. Ix d(a") =a" log adv. Xa de) aed. x d(w’) = we du + log wn’ do. xI din v) = cos vdv. xi GG) eae XII d (tan v) = sectvdy, ete. XVUI dave sin) = 7 = ote The term “differentiation” also includes the operation of finding differentials. In finding differentials the easiest way is to find the derivative as usual, and then multiply the result by dx. Iuusrrative Exameve 1. Find the differential of _2+8 Y" 248 — (243) _ G+ Bde + 8)— @ + 8)d@? +8) Satation ana) eae _ (2 + 8)de— (v + 8) 20dz 6z—a2')de 4. @+3F * Innusrrarive Exampre 2. Find dy from Wa? — aty? = a*l?, Solution. 2ade — 2a%ydy Inuusrrarive Examrre 3. Find dp from p? =a? cos 26, Solution, 2pdp =— a?sin2g.2d0, a®sin20 dp =— S828 ag, 7 P Inusrrarive Exampie 4. Find d[aresin(8t— 40)]. Solution. d[are sin (3 — 4] = Joes = a = Vi-@im Vine DIFFERENTIALS 139 93. Successive differentials. As the differential of a function is in general also a function of the independent variable, we may deal with its differential. Consider the function y=S@). d(dy) is called the second differential of y (or of the function) and is denoted by the symbol #y. Similarly, the third differential of y, d[d(dy)], is written ay, and so on, to the nth differential of y, ay. Since dz, the differential of the independent variable, is independ- ent of x (see footnote, p. 131), it must be treated as a constant when differentiating with respect to 2, Bearing this in mind, we get very simple relations between successive differentials and successive deriva- tives. For dy =f"(a) dx, and @y =f"(a) (day, since de is regarded as a constant. Also, Py =f") (dey’, and in general ay = f(a) (day. Dividing both sides of each expression by the power of dz occur- ring on the right, we get our ordinary derivative notation Pa g@, Yas@, - GHasre. Powers of an infinitesimal are called infinitesimals of a higher order. More generally, if for the infinitesimals a and 8, limit 8 = 0, @ then is said to be an infinitesimal of a higher order than a. Inuusrrative Examrie 1, Find the third differential of yaot—2a8 482-5, dy = (624 — 62? + 8)dx, (20:08 — 12.2) (de), (60 x? — 12) (dz)®. Ans. Nore. This is evidently the third derivative of the function multiplied by the cube of the differential of the independent variable. Dividing through by (dz)*, we get the third derivative Solution, ay ya OO 12. DIFFERENTIAL CALCULUS EXAMPLES Differentiate the following, using differentials: L. y= aa — be? 4 cr +d. Ans, dy = (8 az? — 2bx + 0) dz. 2 y=2at— sb + 62-145. dy = (50! — 207} 62-2) ae 3. y= (@— 2). dy =— 102 (a? — 22)hde. =Vi+e. ay =— Vite aan ncin—1 .yo ne. dy = 20” ae, 8 ap Y= Ty apt Bate ee pea 6. y=logV1—@ dy =a) =(@4e% dy = 2(82— e-2%)dz, 8 y= erloge. dy = (loge + 2) a0, 9 sat-fo a= (Ya gerry ~ Net pet L+sing 10. p = tang + secg. dp cost p —— 11, r=} tant9 + tand, dr = sectd8. 2 12. f(e) = (logz). I @de= Siogs)*de, 2, 13, ¢()=—_*_. oya= ae —al q-at loge a|- logede log (1 — 14, af eee + log( G2 dy 15, d[are tan logy] = ——_Y_. g e1= T+ ow} 1 y ee 16. alr are vers” —V2ry— |= " Vane cose .d — log t 2 Laanes te CHAPTER X RATES 94, The derivative considered as the ratio of two rates. Let y=f@) be the equation of a curve generated by a moving point P. Its coordi- nates @ and y may then be considered as functions of the time, as explained in § 71, p. 91. Differentiating with respect to ¢, by XXV, we have dy dx (32} —=f'(xs—- (82) SO" At any instant the time rate of change © of y (or the function) equals its derivative multiplied by the time rate of change of the independent variable. Or, write (32) in the form (33) The derivative measures the ratio of the time rate of change of y to that of w. © being the time rate of change of length of are, we have from (22), p92, " s\-S which is the relation indicated by the above figure. As a guide in solving rate problems use the following rule: Frsr Srev. Draw a figure illustrating the problem. Denote by x, y, 2, ete., the quantities which vary with the time. Sxconp Srer. Obtain a relation between the variables involved which will hold true at any instant. 141 142 DIFFERENTIAL CALCULUS Turp Srer. Differentiate with reapect to the time. Fourrn Strep. Make a list of the given and required quantities. Firra Srep. Substitute the known quantities in the result found by differentiating (third step), and solve for the unknown. EXAMPLES 1. A man is walking at the rate of 5 miies per hour towards the foot of a tower 60 ft, high, At what rate is he approaching the top when he is 80 ft, from the foot of the tower ? Solution, Apply the above rule. First step. Draw the figure. Let = distance of the man from the foot and y = his distance from the top of the tower at any instant, Second step. Since we have a right triangle, y? = 2? + 3600, Third step. Differentiating, we get mF dy de ' oy o22%, or, L vG = 22%, on, : 4 i (4) & = 22, meaning that at any instant whatever ! : ca (Rate of change of ) = G (rate of change of «). af Fourth step. == 80, “ = 5 miles an hour, = 5 x 5280 ft. an hour. VE +300 dy _ 100. ae Fifth step. Substituting back in (4), 2 S = X 5 x 5280 ft, per hour = 4 miles per hour, Ans, 2. A point moves on the parabola 6y =? in such a way that when 2 = 6, the abscissa is increasing at the rate of 2 ft. per second. At what rates are the ordinate and length of arc increasing at the same instant ? Solution, First step. Plot the parabola. Second step. 6y=a22. , dy_ st Third step. 6G = 2a oF, dy_ 2 de a a 3 a ‘This means that at any point on the parabola (Rate of change of ordinate) = () (rate of change of abscissa). RATES: 143 de Fourth step. Gm? Mt. por second. dy _» oe oo a Fifth step. Substituting back in (B), u =§ x 2=4 ft. per second. Ans. Substituting in (84), p. 141, a V2? + @=2V5 tt. per second, Ans. From the first result we note that at the point P (6, 6) the or as rapidly as the abscissa, If we consider the point P’ (— 6, 6) instead, the result is ate changes twice w — 4 ft. per second, the minus sign indicating that the ordinate is decreasing as the abscissa increases, 3. Accircular plate of metal expands by heat so that its radius increases uniformly at the rate of .01 inch per second, At what rate is the surface increasing when the radius is two inches ? Solution, Let ¢ = radius and y = area of plate, ‘Then yore. dy de W = one t, © a ae ‘Thay is, at any instant the area of the plate is increasing in square inches 2m times as fast as the radius is increasing in linear inches. Substituting in (C), ay at 2m x 2x OL 04 sq, in, per sec. Ans. 4, An are light is hung 12 ft, directly above a straight horizontal walk on which a boy 5 ft. in height is walking. How fast is the boy's shadow lengthening when he is walking away from the light at the rate of 168 ft. per minute ? Solution, Let « = distance of boy from a point directly L under light Z, and y= length of boy's shadow, From the figure, viy-$er:5212, ‘ 1 or ya he. H Differentiating, ay bdr, a 7a mM F i.e. the shadow is lengthening § as fast as the boy is walking, or 120 ft. per minute. 5. In a parabola y* = 122, if z increases uniformly at the rate of 2 in. per second, at what rate is y increasing when z = 3 in. ? Ans. 2 in, per sec. 144 DIFFERENTIAL CALCULUS 6, At what point on the parabola of the last example do the abscissa and ordinate increase at the same rate ? Ans. (8,6). 7. In the function y = 2z°+ 6, what is the value of x at the point where y increases 24 times as fast as 2? Ans. t=42. 8, The ordinate of a point describing the curve a? + y? = 25 is decreasing at the rate of 1} in. per second, How rapidly is the abscissa changing when the ordinate is 4 inches ? Ans, % = 2 in, per see. dt 9, Find the values of x at the points where the rate of change of ah 124? + 450-18 is zero. Ans. ¢=8and5, 10, At what point on the ellipse 162% 4 9y? = 400 does y decrease at the same rate that # increases ? Ans, (8, ¥f). 11. Where in the first quadrant does the arc increase twice as fast as the ordinate ? Ans, At 60°, A point generates each of the following curves. Find the rate at which the are is increasing in each case 12, y=20; Bane Ans. 18, ay = 6; Yaa yas. 14, ee e. 1 y=0; Bo ae=—3. 16 faa; Body 17. The side of an equilateral triangle is 24 inches long, and is increasing at the rate of 3 inches per hour, How fast is the area increasing ? : Ans. 86-V3 sq. in. per hour. 18. Find the rate of change of the area of a square when the side b is increasing at the rate of a units per second. Ans. 2ab sq. units per sec. 19. (a) The volume of a spherical soap bubble increases how many times as fast as the radius? (b) When its radius is 4 in, and increasing at the rate of } in. per second, how fast is the volume increasing ? Ans, (a) 47? times as fast ; (v) 822 eu. in, per see. How fast is the surface increasing in the last case ? 20. One end of a ladder 50 ft, long is leaning against a perpendicular wall stand- ing on a horizontal plane. Supposing the foot of the ladder to be pulled away from the wall at the rate of 8 ft. per minute ; (a) how fast is the top of the ladder descending when the foot is 14 ft. from the wall? (b) when will the top and bottom of the ladder move at the same rate? (c) when is the top of the ladder descending at the rate of 4 it. per minute ? Ans. (a) | ft. per min, ; (b) when 25 -V2 ft. from wall ; () when 40 ft, from wall, 21, A barge whose deck is 12 it, below the level of a dock is drawn up to it by means of a cable attached to a ring in the floor of the dock, the cable being hauled in by a windlass on deck at the rate of 8 ft. per minute. How fast is the barge moving towards the dock when 16 ft. away ? Ans, 10 ft, per minute, RATES 145 22, An clevated car is 40 ft. immediately above a surface ear, their tracks inter- secting at right angles. If the speed of the elevated car is 16 miles per hour and of the surface car 8 miles per hour, at what rate are the cars separating 5 minutes after they meet ? Ans. 17.9 miles per hour. 23, One ship was sailing south at the rate of 6 miles per hour ; another east at the rete of 8 miles per hour, At 4 v.a, the second crossed the track of the first where the first was two hours before ; (a) how was the distance between the ships changing at Bra? (b) how at 5r.a.? (c) when was the distance between them not changing? Ans. (a) Diminishing 2.8 miles per hour ; (b) increasing 8.73 miles per hour ; (©) 8:17 pan. 24, Assuming the volume of the wood in a tree to be proportional to the cube of its diameter, and that the latter increases uniformly year by year when growing, show that the rate of growth when the diameter is 8 ft. is 36 times as great as when the diameter is 6 inches. 25, A railroad train is running 15 miles an hour past a station 800 ft. long, the track having the form of the parabola y= 600, and situated as shown in the figure, If the sun is just rising in the east, find how fast the shadow S of the locomotive L is moving along the wall of the station at the instant it reaches the end of the wall. Solution. y= 6002, North ay — goo tt 2y— =000—, “ut a y ----2—- yf a de_ iv dy a. dt =300dt \ Substituting this value of & in as dey? (dv Fag a= V(a) + (a): vee s\? _ (v wy ny D) ay = (2 WY 5 (mY. @) a) (i a, : (a as . Now = 16 miles per hour = 22 ft, per sec. y= 400 ana YY = 2 a Substituting back in (D), we get 2 = (18 ) ay 2 =G (a 2 dy or, Gf = 184 ft, per second, Ans. 26, An express train and a balloon start from the same point at the same instant. The former travels 50 miles an hour and the latter rises at the rate of 10 miles an hour. How fast are they separating ? Ans, 61 miles an hour, \ : | ' i 146 DIFFERENTIAL CALCULUS 27. A man 6 ft, tall walks away from a lamp-post 10 ft. high at the rate of 4 miles an hour. How fast does the shadow of his head move ? Ans. 10 miles an hour, 28. The rays of the sun make an angle of 30° with the horizon, A ball is thrown vertically upward to a height of 64 ft. How fast is the shadow of the ball moving along the ground just before it strikes the ground ? Ans, 110.8 ft. per sec. 29. A ship is anchored in 18 ft. of water. The cable passes over a sheave on the bow 6 ft. above the surface of the water. If the cable is taken in at the rate of 1 ft. a second, how fast is the ship moving when there are 80 ft. of cable out ? Ans. 1} ft. per sec. 30. A man is hoisting a chest to a window 50 ft. up by means of a. block and tackle. If he pulls in the rope at the rate of 10 ft. a minute while walking away from the building at the rate of 5 it. a minute, how fast is the chest rising at the end of the second minute ? Ans. 10.98 ft. per min, 31, Water flows from a faucet into a hemispherical basin of diameter 14 inches at the rate of 2 cu.in. per second. How fast is the water rising (a) when the water is halfway to the top? (b) just as it runs over? (The volume of a spherical segment =} arth + dah}, where h = altitude of segment.) 82, Sand is being poured on the ground from the orifice of an elevated pipe, and forms a pile which has always the shape of a right circular cone whose height is equal to the radius of the base. If sand is falling at the rate of 6 cu. ft. per sec., how fast is the height of the pile increasing when the height is 5 it. ? 33, An aéroplane is 628 ft. directly above an automobile and starts east at the rate of 20 miles an hour at the same instant the automobile starts east at the rate of 40 miles an hour. How fast are they separating ? 34, A revolving light sending out a bundle of parallel rays is at a distance of $a mile from the shore and makes 1 revolution a minute. Find how fast the light is traveling along the straight beach when at a distance of 1 mile from the nearest point of the shore, Ans, 15.7 miles per min. 35, A kite is 150 ft. high and 200 ft. of string are out. If the kite starts drifting away horizontally at the rate of 4 miles an hour, how fast is the string being paid out at the start? Ans, 2.64 miles an hour. 36. A solution is poured into a conical filter of base radius 6 cm. and height 24 cm. at the rate of 2 cu. cm. a second, and filters out at the rate of 1 cu. em. a second. How fast is the level of the solution rising when (a) one third of the way up? (b) at the top? Ans. (a) .079 em, per sec. ; (b) .009 em. per sec. 87. A horse runs 10 miles per hour on a circular track in the center of which is an are light. How fast will his shadow move along a straight board fence (tangent to the track at the starting point) when he has completed one eighth of the cirouit ? : Ans. 20 miles per hour, 38. The edges of a cube are 24 inches and are inereasing at the rate of .02 in. per minute. At what rate is (a) the volume increasing? (b) the area increasing ? 39, The edges of a regular tetrahedron are 10 inches and are increasing at the rate of 8 in. per hour, At what rate is (a) the volume increasing ? (b) the area increasing? 40. An electric light hangs 40 ft. from a stone wall. A man is walking 12 ft. per second on a straight path 10 ft. from the light and perpendicular to the wall. How fast is the man’s shadow moving when he is 30 ft, from the wall ? Ans, 48 ft. per sec. RATES 147 41, The approach to a drawbridge has a gate whose two arms rotate about the same axis as shown in the figure. The arm over the driveway is 4 yards long and the arm over the footwalk is yards long. Both arms ro- _—C—rti‘CO™sOO_—S—S— tate at the rate of 5 radians per minute, At what rate is the distance between the ex- tremities of the arms chang- ing when they make an angle of 45° with the horizontal ? Ans. 24 yd. per min, 42. A conical funnel of radius 8 inches and of the same depth is filled with a solu- tion which filters at the rate of 1 cu. in. per minute. How fast is the surface falling when it is 1 inch from the top of the funnel ? . : Ans. Zo in, per min, 4a 43. An angle is increasing at a constant rate. Show that the tangent and sine are increasing at the same rate when the angle is zero, and that the tangent increases eight times as fast as the sine when the angle is 60°. CHAPTER XI CHANGE OF VARIABLE 95. Interchange of dependent and independent variables. It is some- times desirable to transform an expression involving derivatives of y with respect to « into an equivalent expression involving instead deriv- atives of « with respect to y. Our examples will show that in many cases sch a change transforms the given expression into a much simpler one. Ox perhaps « is given as an explicit function of y in a problem, and it is found more ee to use a formula involving & os =, ete., than one involving 2 iy, Fy to find the formulas necessary for ue such transformations. Given y =f(z), then from XXVI we have + ete. We shall now proceed dy 4 de (35) ae dy ay dy: de giving in terms of 77- Also, by XXY, Py _ a (dy\_ ad (dy\dy, . de? dc\da)~ dy \de) de? ) dy _1,, But a ok from (35). dy Substituting these in (4), we get ax . ay 6) oe ey a) 148 CHANGE OF VARIABLE 149 giving £4 Yin terms a $ a and a Similarly, @xde_4(@2y ay a dy dy} 7) Si=- ~ ; dy, and so on for higher derivatives. ‘This transformation is called changing the independent variable from x to y. Invsrrative Exampne 1, Change the independent variable from « to y in the equation : (2y- dy Py ty 1 (0 ‘ay Wy dz dx dx? \dz, Solution, Substituting from (85), (86), (87), er 2 @x de Reducing, we get Px Px a ape a much simpler equation, 96. Change of the dependent variable. Let (4) y=F@) and suppose at the same time y is a function of z, say ® y=o@ We may then express wy, *, ete., in terms of & Ss, ete. as follows. In general, 2 is a function of y by (B), p. 453 and since y is a fune- tion of x by (A), it is evident that z is a function of 2. Hence by xxv we have Go Hu8E gS. ae 7 z(#@ ) i) ay But Z '@=-5 # ge) & =$"® &. By xxv ® - Bagna(2)+9@S. 150 DIFFERENTIAL CALCULUS Similarly for higher derivatives. This transformation is called changing the dependent variable from y to 2, the independent variable remaining 2 throughout. We will now illustrate this process by means of an example. Intusrrative Examere 1, Having given the equation @y _ 1, 20+) (y: (B) a ay, @) ada? EF 1+y¥ \de, change the dependent variable from y to z by means of the relation (F) y=tanz, Solution. From (F), dy dz @ @z (az\? mre, BY ste 4 aetetans (2) 8 in (B), (dz ee ( ey, sel 4 at 14 REIN (ogre whe 4 anttane (Yar + 2 MY te zy (ae! and redueing, we get 5 — 2 (@)= = costz, Ans, 97. Change of the independent variable. Let y be a function of 2, and at the same time let 2 (and hence also y) be a function of a new variable t. It is required to express dy @ i at, 4, elon in terms of new derivatives having t as the independent variable. By XXV dy_dyde dt dx dt" dy dy _ dt (A) 7Aarr a ay 2 (2 ayy ila) de? da \dc)~ dt \de) de de But differentiating (A) with respect to ¢, dy dey dy de 4(@)- ajdt) dtd dt ae dt\de)~ dt de} (2) dt CHANGE OF VARIABLE 151 Therefore dxd’y dyd’x dy ad dde ae (ey G) and go on for higher derivatives. This transformation is called changing the independent variable from x to t. It is usually better to work out examples by the methods illustrated above rather than by using the formulas deduced. @) Invusrrative Exampce 1, Change the independent variable from 2 to t in the equation. () ott yet ~ +y=0, by means of the relation () et Solution. dt uot (E) un? dy _ dy dt. Also Za Gg therefore dy _ dy 2 ae at wy a (2) dy dt A (aa dy |, At ‘Also PY net S (MD) We Bx gS (MVE _ WO da? dx \dt at dz i/dx dt dz Substituting in the last result from (E), (@) Substituting (D), (F), (@) in (C), a(n _ Hea) 4 e(e ‘ and reducing, we get oe oo dae Since the formulas deduced in the Differential Calculus generally involve derivatives of y with respect to a, such formulas as (A) and (B) are especially useful when the parametric equations of a curve are given. Such examples were given on pp. 82, 83, and many others will be employed in what follows. 152 DIFFERENTIAL CALCULUS 98. Simultaneous change of both independent and dependent variables. It is often desirable to change both variables simultaneously. An im- portant case is that arising in the transformation from rectangular to polar codrdinates. Since =p cos6 and y=psin6, S@H)=9 becomes by substitution an equation between p and 6, defining p as a function of 6. Hence p, 2, y are all functions of 8. the equation Ittusrrative Exampte 1. Transform the formula for the radius of curvature (42), p. 159, fis cay } (4) ro Ow, ay a into polar eosrdinates. = Solution, Since in (A) and (B), pp. 150, 151, t is any variable’on which « and y depend, we may in this case let t = 6, giving dy ay _ a6 (B) am and wo dedy dy Px Cy 0 a a6 a © fue “es a6) Substituting (B) and (G) in (4), we get (h+(@ Pode dy dy @e ta) | 06 aoe 9 ae “el [eet de Py dy Px” 6 ae? a6 a But since « = p cos and y = . sind, we have @) $e =— psind + cose; 4 By = p cost + sino 2 Dp a © __ 5 cos —2 sing & 4 cose = psind + 2cos0 4 8 + sino ZB ae a6 age’ ae Substituting these in (D) and reducing, [eT (is) - Pag m CHANGE OF VARIABLE 153 EXAMPLES Change the independent variable from z to y in the four following equations: a GT dy dy? dy ) dy By @y @ ) 4. — + 2)(—) =(a— + 1)— —. —-) =({— —_—- (oaif+ ) (a) (*)) tea a?) ~ Nay * “Da Change the dependent variable from y to z in the following equations: 3 5 +n (Gt—20)+(%) =2(i40) 2, yoteon dx, dx? @z_ de dx Ans. j)—=-—— A+ 22, ms. @ +N 57 ga te tee @y 2(1+n) (2 @z ey 6 Ha 4 ARTY Y oy tane. Ans, < — 2 (=) = costs, ae yy la) Y= it ( ae ay ( dy \ (2) dy aly PU (gy MY 5 ony) TY ay) WY 4. gq2y2\ tu Ste ee Bug, t eau! wt elas + Pays + Baty aa Oy=e 2, Ans. © 95% 4 g 92% 4 98-0, aes as tO ae Change the independent variable in the following eight equations: Ans. 2 cose. nae @y | 2a dy ee oy ex tan. 3 tiga Te 2 4, Os sul + sect s=aretant, ase 7 as Ans. a+) Sh 4 eet ware tang 41 as. ot TY + ay =0, 154 In DIFFERENTIAL CALCULUS the following seven examples the equations are given in parametric form. 2 Find “ and oy in each case: 16. 2=740, y=B4P—BU. dy 11, 2=cott, y= ain't, Ans, a= ott, y = sin ns, ‘ f 1 18, 2 =a(cost + tsind), y = a(sint— t cost). : 2 =a(cost + tsing), y= a(sint — t cost), aH 1-t 2t 19, c=——, y=——. =e Tat 2, 2= 21, y=2-0 QM. 2=1-8, y=. 2. = acost, y=bsint. 23, ‘Transform 24, Let f(@, y)=0 be the equation of a curve. Find an expression for its slope (® in terms of polar codrdinates. mo 6 + sing pcosd + sind E a Anu, ¥. —_<. a ap psind + 00505 CHAPTER XIT CURVATURE. RADIUS OF CURVATURE 99. Curvature. The shape of a curve depends very largely upon the rate at which the direction of the tangent changes as the point of contact describes the curve. This rate of change of direction is called curvature and is denoted by K. We now proceed to find its analytical expression, first for the simple case of the circle, and then for curves in general. 100. Curvature of a circle. Consider a circle of radius R. Let 7=angle that the tangent at P makes with OX, and 7+Ar=angle made by the tangent at a neighboring point P’. Then we say Ar = total curvature of are PP. If the point P with its tangent be supposed to move along the curve to P', the total curvature (= Ar) would measure the total change in direction, or rotation, of the tangent; or, what is the same thing, the total change in direction of the are itself. Denoting by s the length of the are of the curve measured from some fixed point (as 4) to P, and by As the length of the are PP’, then the ratio aa ae measures the average change in direction per unit length of arc.* Since, from the figure, As=R-Ar, Ar_1 or e=P * Thus, f Ar=T radians (~ 90°), and Ae~3 centimeters, then = = Z radians por centi- meter ~ 10° per centimeter = average rate of change of direction. 155 156 DIFFERENTIAL CALCULUS it is evident that this ratio is constant everywhere on the circle. This ratio is, by definition, the curvature of the circle, and we have 4 (38) K=-. (38) R The curvature of a circle equals the reciprocal of its radius. 101. Curvature at a point. Consider any curve. As in the last section, : Ar = total curvature of the are PP', Ar and — =averaye curvature of the are PP', As More important, however, than the notion of the average curvature of an are is that of curvature at a point. This is obtained as follows. Tagine P’ to approach P along the curve; then the limiting value of the average curvature a) as P' ap- ; proaches P along the curve is defined as the curvature at P, that is, .,_ limit /At\_ dr Curvature at a point =As=0 (=) dr (39)... K=— =curvature. ds Since the angle Ar is measured in radians and the length of are As in units of length, it follows that the unit of curvature at a point is one radian per unit of length. 102, Formulas for curvature. It is evident that if, in the last sec- tion, instead of measuring the angles which the tangents made with OX, we had denoted by + and ++ Ar the angles made by the tangents with any arbitrarily fixed line, the different steps would in no wise have been changed, and consequently the results are entirely independent of the system of codrdinates used. However, since the equations of the curves we shall consider are all given in either rectangular or polar codrdinates, it is necessary to deduce formulas for K in terms of both. We have dy r=, 82, p. 81 tan § 82, p. dy, or 7 =are tan CURVATURE 157 Differentiating with respect to a, using XX 4) ® dy o. = = Also oe de From (24), p. 134 Dividing (4) by (B) gives But (40) ay deat as at. ia y m [+(Z)] at ee ae ds ds & - ae [+@T If the equation of the curve be given in polar codrdinates, K may be found as follows: From (B), p. 84, (¢) Bat 1=0+4. Differentiating, ! ae ad tan y= e From (A), p. 84 a6 he oh opm are tan 7 a6 Differentiating by XX with respect to @ and reducing, Q) ‘app a6) Page 158 DIFFERENTIAL CALCULUS Substituting (D) in (C), we get 2 pH? 5 9 (dey ee ae “(a6 ® : + Also i: aoe mC) as fing (ay (F) i [? +(%) From (80), p. 136 Dividing (#) by (7) gives a pp th 4 of4ey wo ° eaget 2(55) @ |. ave @ +, (ap wo |r+(a)| dr But g. & =K. Hence wo vp! 3 (4) K= & iuwusrrative Examere 1. Find the curvature of the parabola 7 = 4pz at the upper end of the latus rectum, we 2p dy __ 4p ] a 4p? w+ 4pyt giving the curvature at any point, At the upper end of the latus rectum (p, 27) 2 2 . ee ee (4p? + 4p) 16-V2p? 4 V2 p Solution. Substituting in (40), Inuusrrative Examene 2, Find the curvature of the logarithmic spiral p = ¢@ at any point. A Oe Solution. Wg = = Ps Tye = Se = wp. i Substituting in (41), —_. ans. pVi+e * While in our work it is generally only the numerical value of K that is of importance, yet we can give a geometric meaning to its sign. Throughout our work we have taken the : postive sign ofthe rain [t+ (22) ‘Therefore wil be posiv or mgatve atthe ame time as fu, that is (§ 85, p. 125); according as the curve is concave upwards or concave downwards. CURVATURE 159 In laying out the curves on a railroad it will not do, on account of the high speed of trains, to pass abruptly from a straight stretch of track to a circular curve. In order to make the change of direction gradual, engineers make use of transition curves to connect the straight part of a track with a circular curve. Ares of cubical parabolas are generally employed as transition curves. Inwstrative Exampie 8, ‘The transition curve on a railway-track has the shape of an arc of the cubical parabola y = }2°. At what rate is a car on this track changing its direction (1 mi. = unit of length) when it is passing through (a) the point (8, 9)? {b) the point (2, 4)? (c) the point (1, 4)? &y ey Solution. hax, Ga=2e Substituting in (40), Sa atayt (a) At @,9), -K =—®— radians per mile = 28” per mile. (82) ) Ate, radians per mile = 8° 167 per mile, Ans. ant WaAtd,p, Ka radians per mile = 40° 30° per mile, Ans, ei vi 103. Radius of curvature. By analogy with the circle (see (38), p. 156), the radius of curvature of a curve at a point is defined as the reciprocal of the curvature of the curve at that point. Denoting the radius of curvature by Ft, we have a R= K or, substituting the values of K from (40) and (41), [ [reat = 3 “Parag 6, * Hence the radius of curvature will have the same sign as the curvature, that Is, + or ~, according as the curve is concave upwards or concave downwards. + In § 98, p. 152, (43) is derived from (42) by transforming from rectangular to polar vodrdinates. (42) R= (43) 160 DIFFERENTIAL CALCULUS Intusrrative Exampce 1, Find the radius of curvature at any point of the cate- nary y= 5 +e 4, Solution. Substituting in (42), ET Be) R=————_——__= ete 2a If the equation of the curve is given in parametric form, find the first and second derivatives of y with respect to x from (A) and (B), pp- 150, 151, namely : @ ¥ dx dy dy dx dt dt? dt de (ay and then substitute the results in (42)." dp) Tuuustravive Examrre 2, Find the radius of curvature of the cycloid z=a(t—sing), y=a(1—cost). Solution. & = a(1— cost), a =asint; 2 ey _ Sat ening, eT este Substituting in (@) and (IZ), and then in (42), p. 150, we get dy__sint_ @y _a(1—cost)acost—asintasint dc 1—cosi’ dz? a (1 — cost)® a(1— cost? . [r+ & sint ) i rote 2aV2—2cost. Ans. ~ a — cost? (ay @yT dz ay dy Pax * Substituting (@) and (H) in (42) gives R. CURVATURE 161 104, Circle of curvature. Consider any point P on the curve C. The tangent drawn to the curve at P has the same slope as the curve itself at P (§ 64, p. 73). In an analogous man- ner we may construct for each point of the curve a circle whose curvature is the same as the cur- vature of the curve itself at that point. To do this, proceed as follows. Draw the normal to the curve at P on the concave side of the curve. Lay off on this normal the distance PC =radius of curvature (=) at P. With C as a center draw the circle passing through P. The curvature of this circle is then 1 Say which also equals the curvature of the curve itself at P. The circle so constructed is called the circle of curvature for the point P on the curve. In general, the circle of curvature of a curve at a point will cross the curve at that point. This is illustrated in the above figure. Just as the tangent at P shows the direction of the curve at P, so the circle of curvature at P aids us very materially in forming a geo- metric concept of the curvature of the curve at P, the rate of change of direction of the curve and of the circle being the same at P. In a subsequent section ($ 116) the circle of curvature will be defined as the limiting position of a secant circle, a definition analo- gous to that of the tangent given in § 82, p. 81. Inuusrrative Examrce 4. Find the radius of curvature at the point (8, 4) on the equilat- eral hyperbola ay = 12, and draw the corre- sponding circle of curvature, wy Py _2y, Solution. The circle of curvature crosses the curve at two points. 162 DIFFERENTIAL CALCULUS EXAMPLES 1, Find the radius of curvature for each of the following curves, at the point indi- cated ; draw the curve and the corresponding circle of curvature : (a) Bx? + a%y? = ab, (a, 0). Ans. (b) Bx? + ay? = a, (0, 6). “S@) y= at— 423 — 182%, (0, 0). (a) 16,2 = 4x4 — 2°, (2, 0). ©) V=2, ey 9). ©) #=2, 4,8). () = 82, 8). : o + @i=100. (i) a = day, (0, 0). ©) W224, 0,0). (8) Bet —aty? = 0%, (,, 1). Q) #=siny, (@,%4)+ (P) 9y=2%, 2= 8, (m) v=sina, (5.1). (@) 47 =2,254. (a) y= cose, ( @) @-yYa=a,y (0) y= log, 2 = 6. () a4 27°=9, (1, —2). 2. Determine the radius of curvature of the curve a*y = ba® + cx*y at the origin, : i An, R=—. 2b 2(a— 3. Show that the radius of curvature of the witch 2 = “(*=®) at the vertex is o 4, Find the radius of curvature of the curve y = log sec # at the point (%, 14). 5, Find K at any point on the parabolaz? + yi=ai, Ans. K= 6, Find B at any point on the hypocycloid 2? + yt=a%, Ans. R= 8 (aay). 7. Find R at any point on the cycloid z = rare vers“ — V2ry— ¥ _ r ‘Ans, R=2Viry. Find the radius of curvature of the following curves at any point : 8. The circle p = asin @. Ans. 9. The spiral of Archimedes p = a. 10. The cardioid p = a(1— cos 4). 11. The lemniscate p? = a? cos 20. 12, The parabola p 28, ascot’ 13, The curve p = asin’? CURVATURE 14. The trisectrix p = 2a cos 8 — a. 15. The equilateral hyperbola p? cos2.6 . oe i 16, The coniep = 20= 2), ee ge Ce) eecusene)a 1—ecosd (= ecos OF " 2=8e, 17. ‘The curve { WEA a i. a ., fe acos*t, 18. The hypocycloid { ee oe Ao eee a (cost + taint), a a(sint —teost). t : a 19. ‘The curve { Ans, R= 2 : y a (m-cost + cos mt), a(msint—sin mt), t= ty ‘Ans. R= A sin * 2) ty 2 m—1 20. The curve i 21. Find the radius of curvature for each of the following curves at the point indicated ; draw the curve and the corresponding cirele of curvature : (a) & (e) =ty=6trh;t (b) & ey (c) & sint,y @) a ay 22, An automobile race track has the form of the ellipse 2? + 16y = 16, the unit being one mile. At what rate is a car on this track changing its direction (a) when passing through one end of the major axis ? (b) when passing through one end of the minor axis? (©) when two miles from the minor axis? (a) when equidistant from the minor and major axes ? Ans. (a) 4 radians per mile ; (b) 7s radian per mile, 23. On leaving her dock a steamship moves on an are of the semicubical parabola 4y°= 4%, If the shore line coincides with the axis of y, and the unit of length is one mile, how fast is the ship changing its direction when one mile from the shore ? Ans. 4s radians per mile. A battleship 400 ft, long has changed its direction 30° while moving through a distance equal to its own length. What is the radius of the circle in which it is moving ? Ans. 764 ft. 25. At what rate is a bicycle rider on a circular track of half a mile diameter changing his direction ? Ans, 4 rad. per mile = 48’ per rod. 26. The origin being directly above the starting point, an aéroplane follows approximately the spiral p = 8, the unit of length being one mile, How rapidly is the aéroplane turning at the instant it has eireled the starting point once? 27. A railway track has curves of approximately the form of arcs from the follow- ing curves. At what rate will an engine change its direction when passing through the points indicated (1 mi, = unit of length) : @ y=2, 2,8)? @) y=e,2=07 | () v= 2%, G9)? © v= cosa,e= 7? (©) — F =8, G1)? () p@=4,0=12 CHAPTER XIII THEOREM OF MEAN VALUE. INDETERMINATE FORMS 105. Rolle’s Theorem. Let y=/(z) be a continuous single-valued function of x, vanishing for e=a and x=, and suppose that f’(z) changes continuously when x varies from a to &. The function will then be rep- resented graphically by a continuous curve as in the figure. Geometric intuition shows us at once that for at least one value of 2 be- tween a and the tangent is parallel to the axis of X (as at P); that is, the slope is zero. This illustrates Rolle’s Theorem : If f(a) vanishes when x =a and x=b, and f(a) and f'(x) are con- tinuous for all values of x from x=a to x=B, then f'(x) will be zero for at least one value of x between a and b. This theorem is obviously true, because as z increases from a to b, (2) cannot always increase or always decrease as x increases, since ‘f(a)=0 and f(6)=0. Hence for at least one value of « between a and 6, f(x) must cease to increase and begin to decrease, or else cease to decrease and begin to increase; and for that particular value of z the first derivative must be zero (§ 81, p. 108). ‘That Rolle’s Theorem does not apply when f(z) or f/(z) are discontinuous is illus- trated as follows: y Fig. a shows the graph of a function which is discontimous (= 0) for z=, a value lying be- tween a and b. Fig. b —G] shows a continuous func- tion whose first derivative is discontinuous (= «) for such an intermediate Fic. a value andz = Fie. b -. In either case it is seen that at no point on the graph between ¢ =a does the tangent (or curve) become parallel to OX. 164 THEOREM OF MEAN VALUE 165 106. The Theorem of Mean Value.* Consider the quantity @ defined by the equation (co) fO2f@ ah =Q, or () FO)-F@—O-aQ=0. Let (2) be a function formed by replacing & by @ in the left-hand member of (B); that is, ce) F@=f@-F@®-C@- 9) From (B), —-F(b) = 0, and from (€), F(a) =0; therefore, by Rolle’s Theorem (p. 164) (2) must be zero for at least one value of x between a and b, say for x, But by differentiating (C) ae F@=s@-e Therefore, since F(x, = 0, then also f"(z) — Q= 0, and Q=f'(@)- Substituting this value of Q in (4), we get the Theorem of Mean Value, fH ae a (44) = =f), a. f"(a,)+ a =0. By § 118, p. 174, ogy = Be =e when = 0. — eset cotz sing By § 112, p. 174, logy = 0, wh y§ ee ad —esereots costa ely Since y = (cota), this gives log, (cota) = 0; ie, (cotz)=z = 1. Ans. EXAMPLES Evaluate the following expressions by differentiation : Tint _ Hmlt gi ey i, Setar Ans. &. a, Hint on: _ 8. Tie (oa 3. 7s limit 9 ny (Lt nz) a 3, limit (in gyno, 1 of 5 gaz 10. Tite = (ante _ o=1\""G e 4, limit (ue sy. e, uw. ton (cosa) echam, 5, limit (1+ sinayests, e. com Tae eae 7h @ oo e- ne 2 tan 7 2 2a, o z=a ) CHAPTER XIV CIRCLE OF CURVATURE. CENTER OF CURVATURE 116. Circle of curvature.* Center of curvature. If a circle be drawn through three points B, R, B on a plane curve, and if B and B be made to approach B along the curve as a limiting position, then the circle will in general approach in magni- tude and position a limiting circle called. the circle of curvature of the curve at the point P. The center of this circle is ealled the center of curvature. Let the equation of the curve be @ y=f@s and let 2, 2 2, be the abscissas of the points B, B, B respectively, (a', A’) the codrdinates of the center, and 2! the radius of the circle passing through the three points. Then the equation of the circle is @-ayY+y—BY=R*; and since the codrdinates of the points 2, 2, B must satisfy this equa- tion, we have (%— 0+ (y— BR" = 0, @) (@,— «+ (y,- BY k= 0, (@,— dy + (y,— BY RP = 0. Pe ava) Now consider the function of x defined by F@)=(@-ay+y—BY— RB, in which y has been replaced by f(«) from (1). ‘Then from equations (2) we get F@)=0, F@)=0, F@)=0. * Sometimes called the osculating circle. The circle of curvature was defined from another point of view on p. 161. 178 CIRCLE AND CENTER OF CURVATURE 179 Hence, by Rolle’s Theorem (p. 164), #’() must vanish for at least two values of 2, one lying between 2, and z, say 2’, and the other lying between «, and 2, say 2”; that is, F@)=0, Fie") =0. Again, for the same reason, (2) must vanish for some value of x between 2! and 2", say a,3 hence F"(x,) =0. Therefore the elements a’, 6', 2! of the circle passing through the points 2, 2, B must satisfy the three equations FQ@,)=0, Fi@)=0, FM, =0. Now let the points R and & approach F as a limiting position ; then 24%» 2, 2", x, will all approach «, as a limit, and the elements a, 8, R of the osculating circle are therefore determined by the three equations F@)=0, F@)=0, FM@,)=05 or, dropping the subscripts, which is the same thing, ~) @—ay+(y—ByY=h, @) @-a)+(y—8)L=0, differentiating (4). (©) 1+ (2) + —Ay Gh, aitorentiating (A). Solving. (B) and (C’) for 2 — @ and y —8, we get (@ # 0), @) hence the codrdinates of the center of curvature are 2 z+ (a) | ae dz} |. ce dy” ae ® 180 DIFFERENTIAL CALCULUS Substituting the values of 2—a and y— from (D) in (A), and solving for R, we get ' 7 da, &y aa? which is identical with (42), p. 159. Hence Theorem. The radius of the circle of curvature equals the radius of curvature. 117. Second method for finding center of curvature. Here we shall make use of the definition of circle of curvature given on p. 161. Draw a figure showing the tangent line, circle of curvature, radius of curvature, and center of curvature (a, 8) corresponding to the point P(z, y) on the curve. ‘Then R=+ a@=0A=0D—AD=0D—BP=z— BP, X B=AC=AB+BC=DP +BC=y4BC. But BP=Zsint, BC=K cost. Hence (4 a@=2—Rsint, B=yt Roost. From (29), p. 135, and (42), p. 159, z all : de 1 [1 Tae, sin tT =———»_ cost=———| R= aT eT a [1 + ( 2) | 1+(¢ os Substituting these back in (4), we got ay ‘ay (50) one Het] B= ae From (28), p. 126, we know that at a point of inflection (as @ in the next figure) ayy, dat CIRCLE AND CENTER OF CURVATURE 181 Therefore, by (40), p. 157, the curvature K=0; and from (42), p- 159, and (50), p. 180, we see that in general a, 8, R increase without limit as the second derivative approaches zero. That is, if we suppose P with its tangent to move along the curve to P’, at the point ofp, inflection Q the curvature is zero, the rotation of the tangent is momentarily arrested, and as the c direction of rotation changes, the center of cur- vatiure moves out indefinitely and the radius of curvature becomes infinite. Iniusrrative Exampte 1, Find the codrdinates of the center of curvature of the parabola y? = 4 px. corresponding (a) to any point on the eurve ; (b) to the vertex. dy _2p_ dy 4p? ga aa (a) Substituting in (E), p. 179, Solution. aaet e p oo ee e ‘Therefore (32 + 2p, — : >) is the center of curvature Fi corresponding to any point on the curve. (b) (2p, 0) is the center of eurvature corresponding to the vertex (0, 0). 118. Center of curvature the limiting position of the intersection of normals at neighboring points. Let the equation of a curve be (A) y=f@) The equations of the normals to the curve at two neighboring points PR and P are* @a)+(4—-¥) Cap) 0, dq GG V)GB=0. Sp) If the normals intersect at C’(a', A’ Fe@eate) the coérdinates of this point must satisfy both equations, giving a+ Gn BD GE= 0, ® a) +4, 0) B= 0. * From (2), p. 77, Xand ¥ being the variable codrdinates. 182 DIFFERENTIAL CALCULUS Now consider the function of x defined by qd $@=@-e)+y-B) in which y has been replaced by (2) from (A). Then equations (B) show that $@)=9, $@)=0. But then, by Rolle’s Theorem (p. 164), $/() must vanish for some value of 2 between #, and x, say 2’. ‘Therefore a! and A! are deter- mined by the two equations d@)=9, P@)=0. If now B approaches F as a limiting position, then 2’ approaches z,, giving $@)=9, $m) = 95 and C'(a', 8’) will approach as a limiting position the center of cur- vature C'(a, 8) corresponding to B on the curve. Vor if we drop the subscripts and write the last two equations in the form @-a)+y— By B= 0, 1+ ()+@-2) S4=0, it is evident that solving for a’ and ' will give the same results as solving (B) and (C), p. 179, for @ and 8. Hence Theorem. The center of curvature C corresponding to a point P on a curve is the limiting position of the intersection of the normal to the curve at P with a neighboring normal. 119. Evolutes. The locus of the centers of curvature of a given curve is called the evolute of that curve. Consider the circle of curvature corre- sponding to a point P on a curve. If P moves along the given curve, we may suppose the corresponding circle of eurva- ture to roll along the curve with it, its radius varying so as to be always equal to the radius of curvature of the curve at the point P. The curve CC, described by the center of the circle is the evolute of PR. It is instructive to make an approximate construction of the evolute of a curve by estimating (from the shape of the curve) the lengths CIRCLE AND CENTER OF CURVATURE 183 of the radii of curvature at different points on the curve and then drawing them in and drawing the locus of the centers of curvature. Formula (£), p. 179, gives the codrdinates of any point (a, 8) on the evolute expressed in terms of the codrdinates of the corresponding point (2, y) of the given curve. But y is a function of 2; therefore eal [ea deg aes ss . ae et give us at once the parametric equations of the evolute in terms of the parameter x. To find the ordinary rectangular equation of the evolute we elimi- nate x between the two expressions. No general process of elimination can be given that will apply in all cases, the method to be adopted depending on the form of the given equation. In a large number of cases, however, the student can find the rectangular equation of the evolute by taking the following steps: General directions for finding the equation of the evolute in rectangular coordinates. Fist Ser. Find a and B from (50), p. 180. Stconp Srer. Solve the two resulting equations for x and y in terms of « and B. Tump Srer. Substitute these values of x and y in the given equation. This gives a relation between the variables « and B which is the equation of the evolute. Innustrative Exampre 1. Find the equation of the evolute of the parabola y* = 4 pz. Solution. 2 = 2p, ey First step. ande42p, p= 2. Second step. z =, y=— (4p%e)'. Third step (At=)¥ = 4(=*2), or, ppt =A (a 2p). 27 Remembering that a denotes the abscissa and # the ordinate of a rectangular system of codrdinates, we see that the evolute of the parabola A OB is the semicubical parabola DC’E; the centers of curvature for 0, P, P,, P, being at 0’, C, C,, Cy respectively. 184 DIFFERENTIAL CALCULUS Iuusrrative Exampre 2, Find the equation of the evolute of the ellipse xt + ay? = ab? Ye a solution, W-_ Be, Py__ First step. Second step. Third step. (aa)! + (op)! = (a? — 02)3, the equa- tion of the evolute HHE’H’ of the ellipse ABA’B’. B, E’, H’, H are the centers of curvature corresponding to the points 4, A’, B, B’, on the curve, and C, 0’, 0” corre- spond to the points P, P’, P’. When the equations of the curve are given in parametric form, we _ 5 proceed to find S¥ and 54> as on p. 160, from dy dey dy ds dy _dt dy dtd? dt dt (A) dc de’ a @ dt dt. and then substitute the results in formulas (50), p. 180. ‘This gives the parametric equations of the evolute in terms of the same parameter that oceurs in the given equations. I:tustrative Exampre 8, The parametric equations of a curve are P41 raz (B) Find the equation of the evolute in parametric form, plot the curve and the evolute, find the radius of curvature at the point where ¢ = 1, and draw the corresponding cir- cle of curvature, 7 dt @e il Solution. . olution. 72 @y 2) ae Substituting in above formulas (A) and then in (50), p. 180, gives 1a#-2u ,_4e4ae © , $ CIRCLE AND CENTER OF CURVATURE 185 the parametric equations of the evolute. Assuming values of the parameter t, we cal- culate «, y; a, 8 from (B) and (C); and tabulate the results as follows : Now plot the curve and its evolute, ‘The point (Z, 0) is common to the given curve and its evolute, The given curve (sémicubical parabola) lies entirely to the right and the evo- | 8 lute entirely to the left of 2 = 4. 2 The circle of curvature at A (J, 4), where | $ 1 0 t=1, will have its center at A’ (— 4, #) on the evolute and radius= AA’. To verify our work find radius of curvature at A. From | —1 (42), p. 159, we get -4t = t= v2, when t= ‘This should equal the distance AA = VG497 + EDP = V2. ‘Amriy 4. Find the parametric equations of the evolute of the ty = a(t—sint), y = a(1— cost). Solution, As in Intusrrarive EXxampe 2, p. 160, we got vdy__sint @ 1 dz 1—cost’ da? a(1— cost)?” Substituting these results in formulas (50), p. 180, we get a =a(t+sing, 2 =—a(1—cost). Ans, 186 DIFFERENTIAL CALCULUS Nore. If we eliminate ¢ between equations (D), there results the rectangular equa- tion of the evolute 00’Q" referred to the axes O’a and 0’, The codrdinates of O with respect to these axes are (— wa, — 2a). Let us transform equations (D) to the new set of axes OX and OY. Then a=“2—-7a, p=y—2a, Substituting in (D) and reducing, the equations of the evolute become ‘2 = a(t —sint), @ fF y=a(1— cost’). Since (#) and (C) are identical in form, we have : The evolute of a cycloid is itself a cycloid whose generating circle equals that of the given cycloid. 120. Properties of the evolute. From (A), p. 180, (4) | @=2—-Rsint, B=y+Roosr Let us choose as independent variable the lengths of the are on the given curve; then 2, y, R, 7, a, 8 are functions of s. Differentiating (A) with respect to s gives () ce) : oo re But Fos 7, S4=sin 7, from (26), p. 184; and =, from (38) and (39), p. 156. Substituting in (B) and (€), we obtain (D) «cost —Reost-4— sin _ sine &, (zB) $B _ sing —Rsint-+ + cost = ose. as RE ds ds Dividing (Z) by (D) gives : an a () a cotT= tar CIRCLE AND CENTER OF CURVATURE 187 But iB = tan 7! =slope of tangent to the evolute at C, and da # = tan 7 =slope of tangent to the given curve at the corre- 2 sponding point P(x, y). Substituting the last two results in (7"), we get 1 tan 7! : tant Since the slope of one tangent is the negative reciprocal of the slope of the other, they are perpendicular. But a line perpendicular to the tangent at P is a normal to the curve. Hence A normal to the given curve is a tangent to its evolute. Again, squaring equations ()) and (2) and adding, we get day, (aby _(ae\? (3 = —)=(—}). @ (i) + (Za) =e) But if «= length of are of the evolute, the left-hand member of i (G) is precisely the square of * (from (34), p. 141, where t= 8, &=#,2=4, y=8). Hence (D) asserts that dd [aR del. aR (B)-(a) ea a ‘That is, the radiua of curvature of the given curve inereases or decreases as fast as the are of the evolute increases. In our figure this means that PC,—PC=are CC, The length of an are of the evolute is equal to the difference between the radii of curvature of the given curve which are tangent to this are at its extremities. Thus in Illustrative Example 4, p. 186, we observe that if we fold QP’ (=4 4) over to the left on the evolute, P’ will reach to O', and we have: The length of one are of the cycloid (as 0O'Q") is eight times the length of the radius of the generating circle. 121. Involutes and their mechanical construction. Let a flexible ruler be bent in the form of the curve C,C,, the evolute of the curve BP, and suppose a string of length Z, with one end fastened at C,, to 188 DIFFERENTIAL CALCULUS. be wrapped around the ruler (or curve). It is clear from the results of the last section that when the string is unwound and kept taut, the free end will describe the curve PR. Hence the name evolute. The curve RR is said to be an invo- lute of 0,C,, Obviously any point on the string will describe an involute, so that a given curve has an infinite number of involutes but only one evolute. The involutes RR, RR’, R"R" ave called parallel curves since the distance between any two of them measured along their common normals is con- stant. ‘The student should observe how the parabola and ellipse on pp. 183, 184 may be constructed in this way from their evolutes. EXAMPLES Find the codrdinates of the center of curvature and the equation of the evolute of each of the following curves. Draw the curve and its evolute, and draw at least one circle of curvature. 2 24 8) zt 2 4 bey? 1. The hyperbota Ans. an Gt, = eer, evolute (aa)$ — (bp)# = (a? + b*)#. 2. The hypocycloid xt +y% = ab. Ans. a= + 3atyt, Bay + 8atyti evolute (a + f)# + (a —p)# = 2at. 3. Find the codrdinates of the center of curvature of the cubical parabola y° = ate. at + 15 yt aty — 9y% Ans, a= Ste, pity iy ™ 6ay 2at 4, Show that in the parabola ct 4 yt = at we have the relation a+ = 8(e+y). 5. Given the equation of the equilateral hyperbola 22y = a*; show that 3 = ss @ @ From this derive the equation of the evolute (a + 6)t — (a — p)# = 2at. Find the parametric equations of the evolutes of the following curves in terms of the parameter ¢, Draw the curve and its evolute, and draw at least one circle of curvature. . © = a. cost, @ 6. The hypooyeloid {Fo inte, Ans. { ; acos*t + 8a cost sin®t, Bacosttsint + asin®t, 2 a 7. The curve ua (ce #42e—%, Bt—8, —40, CIRCLE AND CENTER OF CURVATURE 189 a(cost + tsing), be a(sint —t cost), [ fa= 4a, 9. The curve aa 1, . — p=se—2. U 2 @ 8e, 10. The curve { _ w= 28, 11. The curve p=3t 12. The curve =-8, 5 1438, a=T-8e, 13. ‘The curve B=— 20, ( 4t—6 aa, 14, The curve pat se 68 404126 a=}, [a= = 15. The curve 3 4 eee yak. = : y le 7 au o=2t, oe 16. The curve a oo . yak a74+4t t B= 6t VW. 2=4-8, y=2t. Re=ty=t. 18. x= 2t, y=16—2. 23. 2=sint, y = 8cost, We=hy ‘ %. 2=1—cost, y=t—sint. 4 Wea y=st 25. x= costt, y = sintt, Weal, y= i. 26. «= asect, y= btant. CHAPTER XV PARTIAL DIFFERENTIATION 122. Continuous functions of two or more independent variables. ‘A function f(2, y) of two independent variables x and y is defined as continuous for the values (a, b) of (a, y) when limit 22 aF(e N=S(% DY; y=d no matter in what way « and y approach their respective limits a and 6, This definition is sometimes roughly summed up in the state- ment that a very small change in one or both of the independent variables shall produce a very small change in the value of the function.* We may illustrate this geometrically by considering the surface represented by the equation ,_ r(, y), Consider a fixed point P on the surface where z= a and y =o. Denote by Ax and Ay the increments of the independent variables x and y, and by Az the corresponding increment of the dependent variable 2, the codrdinates of P’ being (w+ Aa, y + Ay, 2+ Az). At P the value of the function is z=f (a, b)=MP. If the funetion is continuous at P, then, however Av and Ay may approach the limit zero, Az will also approach the limit zero. ‘That is, MP’ will approach coincidence with MP, the point P’ approaching the point P on the surface from any direction whatever. ‘A similar definition holds for a continuous function of more than two independent variables. In what follows, only values of the independent variables are considered for which a function is continuous. * This will he better understood if the student again reads over § 18, p.14, on continuous functions of a single variable, : 190 PARTIAL DIFFERENTIATION 191 123. Partial derivatives. Since x and y are independent in z=f@Ys x may be supposed to vary while y remains constant, or the reverse, ‘The derivative of z with respect to 2 when 2 varies and y remains constant* is called the partial derivative of z with respect to x, and is denoted by the symbol 2. We may then write 2 @ limit [f(@+Az, ¢ 2). “ ax” Ar=0 [ Ax Similarly, when 2 remains constant* and y varies, the partial derivative of z with respect to y is ez limit [2 (a y+ Ay) ~F 2). “ dy Ay=0 Ay x, == . Fe i8 also written = F(a y), or Z Similarly, 2 is also written af yor 7 In order to avoid confusion the round + has been generally adopted to indicate partial differentiation, Other notations, however, which are in use are (2): (FB) LEMKE Ds EDGY! DL DAF tay ay, Our notation may be extended to a function of any number of independent variables. Thus, if u= Fe, y, 2; then we have the three partial derivatives bu du tu. 9, OF OF OF ax’ dy’ de’? Ox’ By’ bz : Iuwsrramive Exanpre 1, Find the partial derivatives of z= az? + 2hzy + ey? Solution, 2% _ oan + 2by, treating y as a constant, er a oy * The constant values are substituted in the function before differentiating. + Introduced by Jacobi (1804-1851). = 2be + 2cy, treating 2 asa constant. 192 DIFFERENTIAL CALCULUS Inusrrative Exampne 2. Find the partial derivatives of u = sin (ax + by + cz). Solution. = = «008 (we + by + cz), treating y and z as constants, a = bcos (ax + by + 02), treating « and z as constants, YY = = ccos (ae + by + cz), treating y and x as constants, 2 Again tuning to the function FG »» we have, by (4), p- 191, defined = as the limit of the ratio of the increment of the function (y being constant) to the increment of 2, as the increment of 2 approaches the limit zero. Similarly, (B), p. 191, a: : . has defined eS It is evident, however, that if we look upon these partial derivatives from the point of view of § 94, p. 141, then ae ba may be considered as the ratio of the time rates of change of z and zx when y is constant, and 22 ey as the ratio of the time rates of change of z and y when « is constant. 124, Partial derivatives interpreted geometrically. Let the equa- tion of the surface shown in the figure be 2=fe 9) Pass a plane ZVG@H through the point P (where =a and y=6) on the surface parallel to the XOZ-plane. Since the equation of this plane is y=, the equation of the section JPK cut out of the surface is 2=f(@ D), if we consider HF as the axis of Zand EH as the axis of X. In this a 2 tan MTP = slope of section JK at P. 2 de plane & means the same as @, and we have " re PARTIAL DIFFERENTIATION 193 Similarly, if we pass the plane BCD through P parallel to the vozplane, its equation is gg, and for the section DPI, a means the same as gz. Hence ly 7 = 7 =~ tan MT'P = slope of section DI at P. 2 oy 2 Inpusrrative, Examrie 1. Given the ellipsoid nts +55 1; find the slope of the section of the ellipsoid made (a) by the plane y = 1 at the point where « = 4 and z is positive ; (b) by the plane z = 2 at the point where y = 8 and z is positive. Solution, Considering y as constant, (b) When ¢ =2 and y=3,2=-L. v2 EXAMPLES 1. usa 4 8at%y —y*, Ans. S808 + 6ay; au —=82-8y*, : i. 2. u= Aa? + Bry + Cy + De + By + FP. ade + By + D; OU _ Be + 2Cy +E. oy ou 2ancu = (a8 + yf + oat oS oan 3. u = (ax + by? + cz?) ashe au ___Bbmyu Oy” cat + by? + cat 4, u=aresin=. eo ay 6. w= aa'ytz + beytet + cy? + daz’, = = Baxtytz + byte! + ae; Ge axbya + Sbay%et + Bey; a Sa ante + Abayted + Bdzz?, 194 DIFFERENTIAL CALCULUS 1. w= aty? — 2ayt + 82%9; show that om 4 y= bu, ae Yay 8. u=—_; show that 2! 4 . B4+y oat ay 9% w= (y—2) (2-2) @—y); show that % 4 % 4 2 be by” au, ow 10. u = log (e&* + ev); sh that — + — w= log (e+ 0%); show that 4 ew eu a 1. u=—_; show that & 4 4 — =1u ws yg! show that += @ty— Iu eu, ou 12, u= my; show that 2 4 y= log u) u. us ane; show that e+ y5°= (ety + loguu ou, bu bu 8 13. u = log (z* + 9° + 2° — Bayz); show that 424% ___3 Ace v8) eet yt ae ety te 14, u=esiny + evsinz; show that 2 2 &) +@) =e 4 ety 4 Qertysin (x + y). 15, w= log (tana + tany + tanz); show that sinaz™ 4 sinay@ 4 singe = ae wy az 16. Let y be the altitude of a right circular cone and # the radius of its base. Show (a) that if the base remains constant, the volume changes } wz? times as fast as the altitude ; (b) that if the altitude remains constant, the volume changes 3 may times as fast as the radius of the base. 2 4 17. A point moves on the elliptic paraboloid z [ + [ and also in a plane par- allel to the XOZ-plane. When =8 ft. and is increasing at the rate of 9 ft. per second, find (a) the time rate of change of z; (b) the magnitude of the velocity of the point ; (¢) the direction of its motion, = Ans. (a) vz = 6 ft, per sec.; (b) v= 8-13 ft, per sec. ; (©) t= aretan 3, the angle made with the XOY-plane, 18, If, on the surface of Ex. 17, the point moves in a plane parallel to the plane YOZ, find, when y =2 and increases at the rate of 6 ft. per sec., (a) the time rate of change of z; (b) the magnitude of the velocity of the point; (c) the direction of its motion. Ans. (a) 5 ft. per sec. ; (b) 5-V2 ft. per sec. ; (or - the angle made with the plane XOY. 125. Total derivatives. We have already considered the differ- entiation of a function of one function of a single independent variable. Thus, if y= FC) and v= b(2), it was shown that dy dy dv ae” do ds" PARTIAL DIFFERENTIATION 195 We shall next consider a function of two variables, both of which depend on a single independent variable. Consider the function u=F@ 9); where « and y are functions of a third variable ¢. Let ¢ take on the increment A, and let Az, Ay, Au be the corre- sponding increments of 2, y, u respectively. ‘Then the quantity Au=f(e+ Az, y+ Ay) —f@Y) is called the total increment of u. Adding and subtracting f(2, y + Ay) in the second member, (A) Au=[f@+As, ytAy)—f(@ yt Ay) ]+[F@ yt Ay) Sf 9))- Applying the Theorem of Mean Value (46), p. 166, to each of the two differences on the right-hand side of (4), we get, for the first difference, (B) F@ + Ax, y+ AY) —F@ yt AY) =fi@+ 0, Ax, y + Ay) Ac. to 80 a sae wares ley resin [eenstant, we get the parla Uoivaive with ropes to = For the second difference we get () f@ y+ Ay —F@ =H y+ 0,-Ay) Ay. [ay Atay and sine varios while remains con) stant, we get the partial derivative with respect to 1. Substituting (B) and (€) in (A) gives (D) Au=fi(e+ 0, Ax, y+ Ay) Ae +fi(a y + Oy Ay) Ay, where @, and 8, are positive proper fractions. Dividing @ “ At, Cy ANG +O de y AYN ZAG yt Ody) Now let At approach zero as a limit, then ) Fane NS+H@ yU Since Ax and ay converge to zero with Af, we get [2ivceran ery an)= fears and BPH ji, y sae = fie »| ale, y) and f(z, y) being assumed continnons. Replacing f(a, y) by u in (F), we get the total derivative Hf - au dx. au dy. e) a ax dt t oy at 196 DIFFERENTIAL CALCULUS In the same way, if Uu=$@ ID, and 2, y, z are all functions of t, we get du odudx odudy dudz ) at — ox dt + dy dt * a2 at’ and so on for any number of variables.* In (61) we may suppose t=2; then y is a function of 2, and u is really a function of the one variable 2, giving du au audy .) ae ox oy ae In the same way, from (52) we have du ou au dy , au dz 4) ae ox Oy det oe ‘The student should observe that a and * have quite different meanings. The partial derivative oe is formed on the supposition that the particular variable x alone varies, while du _ limit (Aw dz Av=0 (x) where Aw is the total increment of u caused by changes in all the vari- ables, these increments being due to the change Ag in the independent variable. In contradistinetion to partial derivatives, &, e are called a total derivatives with respect to t and 2 respectively.t This is really only a special case of a general theorem which may be stated as follows: If wis a function of the independent variables z, y, z, ---, each of these in turn being a function of the independent variables r, s, f, ---, then (with certain assumptions as to continuity) du _ dude du dy , dude Or” Oz Gr dy ar Oz or” ou and similar expressions hold for 2, , ete, ae" a a # It should be observed that S* has a perfectly definite valuefor any point (e, v), while 5 depends not only on the point (2, 7), but also on the particular direction chosen to reach that point. Hence, $e is called a point function; while % 55 not ealled a point function unless it is agreed to approach the point from some particular direction. PARTIAL DIFFERENTIATION 197 Iniusrrative Examrce 1. Given u=sin2,2=¢, y=; find *. gui a bu az 2, de dy ution, 2 — 2 e9g%, MH = — = eos?; =o, 21, oem rere yeerers cern au e cog itutis —=(-—2)— —. Ans, Substituting in (61), F¥=(@—2) Goose. Ans Intwernarive Exampie 2. Given u= e(y—2), y= asing, 2 = cos; find &. au eu dy a sotution, % = gem (y — 2), er Ya cose, @ =~ sing. es oe oe a de Substituting in (54), & = ee (y — 2) + ae cose + ett sing = et(a? + I)sinz. Ans. Nore. In examples like the above, u could, by substitution, be found explicitly in terms of the independent variable and then differentiated directly, but generally this process would be longer and in many cases could not be used at all. Formulas (51) and (52) are very useful in all applications involv- ing time rates of change of functions of two or more variables. The process is practically the same as that outlined in the rule given on p- 141, except that, instead of differentiating with respect to ¢ (Third Step), we find the partial derivatives and substitute in (51) or (52). Let us illustrate by an example. Inuusrrative Examrte 3. The altitude of a circular cone is 100 inches, and decreasing at the rate of 10 inches per second ; and the radius of the base is 50 inches, and increasing at the rate of 5 inches per second. At what rate is the volume changing ? Solution, Let 2 = radius of base, y = altitude; then u = arty . volume, 2% pr i Substitute in (51), du 2 de Tay aa ta a But 37" 5000-5 — iv 2500-10 = 15.15 cu. ft. per sec., increase. Ans. 126, Total differentials. Multiplying (51) and (52) through by dt, we get ou ou (65) = FE de 4 a, . ou ou ou (56) = Fd 4 Fd Ses and so on.* Equations (85) and (56) define the quantity du, which is called a total differential of u or a complete differential, ou ou ou Fede Gedy Gade * A geometric interpretation of this result will be given on p. 264. and 198 DIFFERENTIAL CALCULUS are called partial differentials. These partial differentials are some- times denoted by 1% d,u dz, so that (56) is also written dus dutdut dau Tntusrnative Examree 1, Given u=aretan¥, find du. : ou y ou x Solution. Sa-—4 — a B+ ey eth Substituting in (55), du= dy — yd de Ans. ety Intusrrative Exampre 2, The base and altitude of a rectangle are 5 and 4 inches respectively. At a certain instant they are increasing continously at the rate of 2 inches and 1 inch per second respectively, At what rate is the area of the rectangle increasing at that instant ? Solution. Let « = base, y = altitude; then u= ay = area, = ane Me y Substituting in (51), (A) But 2=5in., y= 4 in. @ =9 in. per sec., @ =1 in, per seo. dt "de | : e = (8 + 5)sq. in, per sec, = 18 sq. in. per sec. Ans. : Nore. Considering du as an infinitesimal increment of area due to the infinitesimal increments dz and dy, du is evidently the sum of two thin strips added on to the two sides. For, in du = yd + ady (multiplying (A) by dé), yde = area of vertical strip, and ay dy = area of horizontal strip. But the total increment Au due to the increments dz and yy dy is evidently ay — yar + dy + drdy. Hence the small rectangle in the upper right-hand corner = de (= dedy) is evidently the difference between Au and du. ‘This figure illustrates the fact that the total increment and the total differential of a function of several variables are not in general equal. 127. Differentiation of implicit functions. The equation 4) F@Y=0 defines either x or y as an implicit function of the other.* It repre- sents any equation containing x and y when all its term¢ have been transposed to the first member. Let ® u=f@ ys du then (63), p. 196 * We assume that a small change in the value of z causes only a small change in the value of y. PARTIAL DIFFERENTIATION 199 But from (4), f(a, y)=0. w= 0 and a0; that is, ou, du dy %) . ax” by dx : Solving’ for i »* we get au dy ax ou 57) ee M0 67) ee ae oy a formula for differentiating implicit functions. This formula in the form (@) is equivalent to the process employed in § 62, p. 69, for differentiating implicit functions, and all the examples on p. 70 may be solved by using formula (57). Since Q) F@Y=9 for all admissible values of 2 and y, we may say that (57) gives the relative time rates of change of x and y which keep f(a, y) from changing at all. Geometrically this means that the point (2, y) must move on the curve whose equation is (D), and (57) determines the direction of its motion at any instant. Since u=f@ Ys» we may write (57) in the form af (57a) Inwsrrative Examrre 1. Given 2tyt + siny = 0, find %. Solution. Let f(x, y) = 2°y! + siny. of of 2ayt Sa2ny, 2 = 42% + cosy. wt, = y' Ys ~ Faty® + cosy” Innusrrative Exapre 2, If @ increases at the rate of 2 inches per second as it passes through the value x = 8 inches, at what rate must y change when y = 1 inch, in order that the function 2y* — 82%y shall remain constant ? Solution, Let /(, y) = 2ay?— 3aty; then oF oye aay, ox from (574), x S Substituting in (57 a), dy __ 2y?— bay dz ~~ Sey—8ae” By (88), p. 141 Bute=8, y=1, FH 2 oa on ft. per second. Ans. du du . and & exist. Tea aumumed that 5 and 7, exit 200 DIFFERENTIAL CALCULUS Let P be the point (2, y, 2) on the surface given by the equation (4) u= F(a, y 2)=0, and let PC and AP be sections made by planes through P parallel to the ¥OZ and XOZ-planes respectively. Along the curve AP, y is constant; therefore, from (2), z is an im- 2 plicit function of x alone, and we have, from (574), oF (58) = ox oF Oz giving the slope at P of the curve AP, § 122, p. 190, & is used instead of = in the first member, since 2 was originally, z from (2), an implicit function of 2 and y; but (58) is deduced on the hypothesis that y remains constant. Similarly, the slope at P of the curve PC is oF (59) a ay OF e EXAMPLES Find the total derivatives, using (61), (62), or (58), in the following six examples: Lu=24P4ey2=sing,yse. Ans. Hasery e(sinz + cos) + sin 22, du _e@(1+2) w= ,y=e Ans, “= 2. u=are tan (zy), y = &. mM. Tae 3, w= log(a?— p%), p =asing. @ du 4 uav4 my, v= logs y =e. = u + ry, v= logs, y oy 5. u=aresin(r—s),r=8t,s= 40, du _ at 6 w= = U9, y= asing, 2 = cose. ee Using (55) or (56), find the total differentials in the next eight examples: 7. w= dye tent} gy®ter. Ans, du = (by? + 2ex + e)dx + (2bye + 3gy*)dy. 8. u=logav. du= Yan + log edy. PARTIAL DIFFERENTIATION 201 9. w= ysine, Ans. du= yin log y cos 2dz + jon 10, w= alory, du =u (Ear + 8 ay). “ y uustt!, dy = Bled — te) (— 12. w= sin (pq). du = cos (pa) {gdp + pag]. 13, u= 2, du = a1 (yzde + ax log ady + ay log 242), 14, w= tan?¢ tan26 tanty. du = 4( oe o a ) sind * sin2d * sin dy 15. Assuming the characteristic equation of a perfect gas to be wp = Rt, where » = volume, p = pressure, t= absolute temperature, and Ra constant, what is the relation between the differentials do, dp, dt? Ans. vdp + pdv = Rat. 16. Using the result in the last example as applied to air, suppose that in a given case we have found by actual experiment that t= 800°C., p = 2000 Ib. per sq, ft., v= 14.4 cubic feet. Find the change in p, assuming it to be uniform, when t changes to 301°C., and v to 14.5 cubie feet, R= 96, Ans. — 7.22:1b. per sq. ft. 17, One side of a triangle is 8 ft. long, and increasing 4 inches per second ; another side is 5 ft., and decreasing 2 inches per second. ‘The included angle is 60°, and increasing 2° per second, At what rate is the area of the triangle changing ? Ans, Increasing 71.05 sq. in. per sec. 18. At what rate is the side opposite the given angle in the last example increasing? Ans, 4,93 in, per sec. 19, One side of a rectangle is 10 in, and increasing 2 in, per sec. The other side is 15 in. and decreasing 1 in. per sec. At what rate is the area changing at the end of two seconds ? Ans. Increasing 12 sq. in, per sec. 20. The three edges of a rectangular parallelepiped are 8, 4, 5 inches, and are each increasing at the rate of .02 in. per min, At what rate is the volume changing ? 21. A boy starts flying a kite. If it moves horizontally at the rate of 2 ft. a sec. and rises at the rate of 5 ft. a sec., how fast is the string being paid out ? Ans. 5.38 ft. a sec. 22, A nan standing on a dock is drawing in the painter of a boat at the rate of 2 ft. asec. His hands are 6 ft. above the bow of the boat. How fast is the boat moving when it is 8 ft, from the dock ? Ans. § ft.a sec, 28. The volume and the radius of a cylindrical boiler are expanding at the rate of 1 eu. ft. and .001 ft. per min, respectively. How fast is the length of the boiler changing when the boiler contains 60 cu. ft. and has a radius of 2 ft. ? Ans. .078 ft. a mi 24, Water is running out of an opening in the vertex of a conical filtering glass, 8 inches high and 6 inches across the top, at the rate of .005 cu. in. per hour. How fast is the surface of the water falling when the depth of the water is 4 inches ? 202 DIFFERENTIAL CALCULUS 25. A covered water tank is made of sheet iron in the form of an inverted cone of altitude 8 ft. surmounted by a cylinder of altitude 6 ft. ‘The diameter is 6 ft. If the sun's heat is increasing the diameter at the rate of .002 ft. per min., the altitude of the cylinder at the rate of .008 ft. per min., and the altitude of the cone at the rate of .0025 ft. per minute, at what rate is (a) the volume increasing; (b) the total area increasing ? In the remaining examples find &, using formula (57 a): Q(t + vy at 26. (22 + y2)2— at (2 7%) =0. eee +t eet) : aera dy_e—y QT. ee 4 ay = as : : a ete 28. sin (zy) — ew — 2ty =0. dy _ y[oos(ey) — ev — 22] | dz x(x + ev— cos(zy)] 128. Successive partial derivatives. Consider the function u=F@y)s then, in general, % ong SH an 8 ay are functions of both # and y, and may be differentiated again with respect to either independent variable, giving successive partial deriva- tives. Regarding x alone as varying, we denote the results by tu tu du ou a?’ Ga" Gat?” Oat? or, when y alone varies, Fu au au ay? ay” oy the notation being similar to that employed for functions of a single variable. If we differentiate u with respect to , regarding y as constant, and then this result with respect to y, regarding « as constant, we obtain Ou ayon a ma)" which we denote by ay 2a Similarly, if we differentiate twice with respect to # and then once with respect to y, the result is denoted by the symbol eu ayeat PARTIAL DIFFERENTIATION 203 129. Order of differentiation immaterial. Consider the function F(a, y). Changing « into z+Az and keeping y constant, we get from the Theorem of Mean Value, (46), p. 166, (A) f@t+Ae, Y—S(m Y=Aw-fi (at O-Any). 0<0<1 ‘a=2,Sa= Az, and since x varies while y remains con stant, we get the partial derivative with respect to 2. If we now change y to y+Ay and keep 2 and Az constant, the total increment of the left-hand member of (4) is (B) [f@+Az, y+ Ay) —F@ yt Ay] -[P@+Ae, y)-f@y)]- The total increment of the right-hand member of (A) found by the Theorem of Mean Value, (46), p. 166, is (C) Aaft (w+. Aa, y +Ay) —Aafl (@+0-Ax,y) 0<6,<1 =AyAaf! (w+ 0,-Ax,y+0,-Ay). 0<6,<1 a= y, Aa= Ay, and since y varies while z and Az remahy constant, we get the partial derivative with respect to y. Since the increments (B) and (@) must be equal, (D) [F@+ Ae, y+ Ay) —F@ y+ Ay)]—[FE+42, Y-F@YI =AyAafl! (a +0, . Ax, y+0,- Ay). In the same manner, if we take the increments in the reverse order, (4) [f@+Aa, y+ Ay) —F@+Ax, ¥)]—[f@ytAN—-f@Y]) =Ardyfll (w+ 0,- Ax, y + 0,- Ay), 6, and @, also lying between zero and unity. The left-hand members of (D) and (#) being identical, we have CP) fy (@ +8, Ax, y +8,- Ay) = fi, (@+8,- Ax, y +8, Ay). ‘Taking the limit of both sides as Ax and Ay approach zero as limits, we have (@) Si @ D=Fiy @Y)> since these functions are assumed continuous. Placing u=f@ y) (@) may be written are (60) Sa ayax axay That is, the operations of differentiating with respect to x and with respect to y are commutative, 204 DIFFERENTIAL CALCULUS. ‘This may be easily extended to higher derivatives. For instance, since (58) is true, fu _ o(Pu)_ ou _ (au) _ (eu) _ we extay bx\xdy) dxdyex dxdy\@x) eyox\ex) Bydax* Similarly for functions of three or more variables. Tntisrnarive Exawpnn 1, Given w= 2ty — 82%; verity 2t — @u , oyou onty Suiatin, SE = Baty — 6a, =- Bz! 182y, 2 =a—oa%y, 2 — 322—182y; hence verified. vy EXAMPLES ae 1, w=cos(e + y)3 verity ey" 2422 ou at 2unkt#, verity 24 _ 2u | foe dyon exoy au . w= ylog (1+ ay) 5 = 3. ylog(1 + ay) faay r se au 4, u=are =F ; verify a8 = bao” 1 44 Ou Bu 5. usin (69); very as aad" otu 6. w= Cety% + Senrtz? + 2erety — xyz; show that = 12 (ey + 2 + ex). eaoyoz au 7. w= er; show that. = (14 Baye + oytztyu. u h He ht Saye + clare) wy? eu eu ou w= ; show that ¢ <4 4 y 2% — 9% Su a+y how that at Yay i au au = (22 +y°)3; show that 82 + Sue au ear eyrea? : — au, u | Bu Th wat + AHS show that TS + To + = ¥ : 10. u= y22e + zante? + xty®e?; show that CHAPTER XVI ENVELOPES 130. Family of curves. Variable parameter. The equation of a curve generally involves, besides the variables x and y, certain con- stants upon which the size, shape, and position of that particular curve depend. For example, the locus of the equation “ @-otty=r is a circle whose center lies on the axis of X at a distance of « from the origin, its size depending on the radius r. Suppose « to take on a series of values; then we shall have a . corresponding series of circles differing ma in their distances from the origin, as CE shown in the figure. ‘Any system of curves formed in this way is called a family of curves, and the quantity a, which is constant for any one curve, but changes in pass- ing from one curve to another, is called a variable parameter. ‘As will appear later on, problems oceur which involve two or more parameters. The above series of circles is said to be a family depending on one parameter. To indicate that @ enters as a variable parameter it is usual to insert it in the functional symbol, thus: I@ =. 131, Envelope of a family of curves depending on one parameter. The curves of a family may be tangent to the same curve or groups of curves, as in the above figure. In that case the name envelope of the family is applied to the curve or group of curves. We shall now explain a method for finding the equation of the envelope of a family of curves. Suppose that the curve whose parametric equations are envelope (A) r=$(@), y=) touches (i.e. has a common tangent with) each curve of the family () SH Y=, 205 206 DIFFERENTIAL CALCULUS, the parameter @ being the same in both cases. The slope of (4) at any point is dy_¥'@) (ec) ae $a) ’ (), p. 80 and the slope of (B) at any point is ; @) wa eo . 2. (57a), p.199 Hence if the curves (4) and (B) are tangent, the slopes ((’) and (D) will be equal (for the same value of «), giving ¥O@Q)_ Fano, $'@). LGA ®) @) SA He \P COL] Ys 2) '(@) = 0. By hypothesis (4) and (B) are tangent for every value of a; hence for all values of « the point (2, y) given by (4) must lie on a curve of the family (B). If we then substitute the values of z and y from (A) in (B), the result will hold true for all values of a; that is, ) SIS @ ¥@, @]=0. The total derivative of (F) with respect to « must therefore vanish, and we get GC LEA OI@O+HhG 4 OVY@O+hG vp O=9, where r=$(a), y=¥(a). Comparing (2) and (@) gives CD TLC, yy @) = 0. Therefore the equations of the envelope satisfy the two equations (B) and (ZD, namely, (ea) F(a yr @)=0 and fia, y, @)=05 that is, the parametric equations of the envelope may be found by solving the two equations (Z) for x and y in terms of the parameter a. General directions for finding the envelope. First Step. Differentiate with respect to the variable parameter, con- sidering all other quantities involved in the given equation as constants. Sxconp Srep. Solve the result and the given equation of the family of curves for x and y in terms of the parameter. These solutions will be the parametric equations of the envelope. ENVELOPES 207 Nore. In case the rectangular equation of the envelope is required we may either eliminate the parameter from the parametric equations of the envelope, or else eliminate the parameter from the given equation (B) of the family and the partial derivative (H). Inusrrative Examvie 1, Find the envelope of the family of straight lines zcosa + ysina =p, @ being the variable parameter. Solution. (A) eoosa+ysina=p. First step. Differentiating (A) with respect to a, (B) —zsina +y cosa =0, Second step. Multiplying (A) by cos a and (B) by sin a and subtracting, we get n= pose. Similarly, eliminating x between (A) and (B), we get y=psing. ‘The parametric equations of the envelope are therefore © ie a being the parameter. Squaring equations (C) and add- ing, we get apy poosa, psina, the rectangular equation of the envelope, which is a circle. Iutvsrrative Exams 2. Find the envelope of a line of constant length a, whose extremities move along two fixed rectangular axes, Solution, Let AB =a in length, and let (4) zeosa +ysina—p=0 be its equation, Now as AB moves always touching the two axes, both a and p will vary. But p may be found in terms of @. For AO = AB eos a =acosa, and p=AOsina =asina cosa. Substituting in (4), (B) xcosa +ysina —asina cosa =0, where @ is the variable parameter. Differentiating (B) with respect to a, (0) —asina + y cosa + asin? a — acosta = 0, Solving (B) and (C) for « and y in terms of a, we get 2 =asinta, @ ie meee the parametric equations of the envelope, a hypocycloid. ‘The corresponding rectangular equation is found from equations (D) by eliminat- ing @ as follows: aba atsinta. yk = ak cos? x. Adding, at + yt = al, 208 DIFFERENTIAL CALCULUS Intusrrarive Examece 1, Find the rectangular equation of the envelope of the straight line y = me + z, where the slope m is the variable parameter, i = Ze Solution. ya mete First step. O=2-2. mt Solving, ma+ VE Substitute in the given equation, =p : pau heayi =42-Vpz, and squaring, y* = 4pz, a parabola, is the equation of the envelope. The family of straight lines formed by varying the slope m is shown in the figure, each line being tangent to the envelope, for we know from Analytic Geometry that y = mz +2 is the tangent to the parabola y? = 4 px expressed in terms of its own slope m. 132. The evolute of a given curve considered as the envelope of its aormals. Since the normals to a curve are all tangent to the evolute, § 118, p. 181, it is evident that the evolute of a curve may also be defined as the envelope of its normals; that is, as the locus of the ultimate intersections of neighboring normals. It is also interesting to notice that if we find the para- metric equations of the envelope by the method of the previous section, we get the coérdinates x and y of the center of curvature; so that we have here a second method for finding the e dinates of the center of curvature. Tf we then eliminate the variable parameter, we have a relation between x and y which is the rectan- gular equation of the evolute (envelope of the normals). 2 Cen curve Inwusrragive Examre 1, Find the evolute of the parabola y? = 4px considered as the envelope of its normals. Solution, ‘The equation of the normal at any point (2’, y’) is wa ) y-¥ =e) from (2), p. 77. As we are considering the normals all along the curve, both z’ and yf will vary. Eliminating 2’ by means of y/?= 4 pr’, we get the equation of the normal to be (4) ENVELOPES 209 Considering y’ as the variable parameter, we wish to find the envelope of this family of normals, Differentiating (A) with respect: to 2, By? _ ow Bp? 2p and solving for 2, (By By? + 8p? 4p Substituting this value of x in (A) and solving for y, a C) ee, © (B) and (€) are then the codrdinates of the center of curvature of the parabola, ‘Taken together, (B) and (C) are the parametric equations of the evolute in terms of the parameter y’. Eliminating y’ between (B) and (0) gives 27 py? = 4(@ — 2p), the rectangular equation of the evolute of the parabola. This is the same result we obtained in Illustrative Example 1, p. 183, by the first method. 133. Two parameters connected by one equation of condition. Many problems occur where it is convenient to use two parameters con- nected by an equation of condition. For instance, the example given in the last section involves the two parameters 2’ and y! which are connected by the equation of the curve. In this case we eliminated z', leaving only the one parameter y/. However, when the elimination is difficult to perform, both the given equation and the equation of condition between the two param- eters may be differentiated with respect to one of the parameters, regarding either parameter as a function of the other. By. studying the solution of the following problem the process will be made clear. Inusrrative Examene 1, Find the envelope of the family of ellipses whose axes coincide and whose area is constant. 2 Solution. (4) Si is the equation of the ellipse where a and b are the variable parameters connected by the equation (B) mab = k, rab being the area of an ellipse whose semi- axes are a and b, Differentiating (A) and (B), regarding @ and} as variables and «and y as constants, we have, using differentials, ada , y*db a + Gp = trom (4), and bda + adb = 0, from (B). 210 DIFFERENTIAL CALCULUS, ‘Transposing one term in each to the second member and dividing, we get Y = : wo. vil ‘Therefore, from (A), Brg G5 M1 giving a=4£2V2 and b=4y V3. Substituting these values in (B), we get the envelope k wakes a pair of conjugate rectangular hyperbolas (see last figure). EXAMPLES 1. Find the envelope of the family of straight lines y =2mzx + mf, m being the variable parameter. Ans. £=— 2m, y =— Bm; or 16y° + 2744 = 0." 2. Find the envelope of the family of parabolas y* = a(e—a), a being the variable parameter, Ans. £=2a,y=44; ory=+}e. 3. Find the envelope of the family of circles 2? + (y — )*=9%, 6 being the variable parameter, Ans, 2=41. 4. Find the equation of the curve having as tangents the family of straight lines = mz + Van? + 1, the slope m being the variable parameter. Ans. ‘The ellipse 22? + ay? = a°b*, 5. Find the envelope of the family of circles whose diameters are double ordi- nates of the parabola y* = 4 px. Ans. The parabola y? = 4p (p + 2). 6. Find the envelope of the family of circles whose diameters are double ordi- nates of the ellipse 082 + ay? = a2, a Ans, The ellipse 7. A circle moves with its center on the parabola y? = 4 az, and its circumference passes through the vertex of the parabola, Find the equation of the envelope of the circles. Ans. The cissoid y? (@ + 2a) +23 = 8, Find the curve whose tangents are y = le + Val? +01 + ¢, the slope ! being supposed to vary. Ans. 4 (ay? + bay + ex*) = 4ac— 0% 9. Find the evolute of the ellipse Da? + ay? inal in the form 20, taking the equation of nor- by = ax tan p — (a? — 4) sing, the eccentric angle ¢ being the parameter. a 2 a? ORF og, y Ue Ans. @ sin? 5 or (az)t + (yt = (@ — vy. 10. Find the evolute of the hypocycloid zt + 4% = aé, the equation of whose normal is yeost — xsint = acos2r, r being the parameter. Ans. (x + y)t + (@@— yt = 2b. + When two answers are given, the first is in parametric form and the second in ree- tangular form. ENVELOPES 211 11. Find the envelope of the circles which pass through the origin and have their centers on the hyperbola 2? — y? = c%. Ans. The lemniscate (2? + y%)? = 402 (e? — y%). 12. Find the envelope of a line such that the sum of its intercepts on the axes equals c. Ans, The parabola zt + yt = ch. 13, Find the equation of the envelope of the system of circles z* + y* — 2(a + 2) x +a? = 0, where a is the parameter. Draw a figure illustrating the problem. Ans. y? = 42. 14. Find the envelope of the family of ellipses b%? + ay? = a0, when the sum of its semiaxes equals c. Ans. The hypocycloid at + y# = ct. 15. Find the envelope of the ellipses whose axes coincide, and such that the dis- tance between the extremities of the major and minor axes is constant and equal tol. ‘Ans. A square whose sides are (x + y)? = 16, Projectiles are fired from a gun with an initial velocity ». Supposing the gun can be given any elevation and is kept always in the same vertical plane, what is the envelope of all possible trajectories, the resistance of the air being neglected ? Hiwr, The equation of any trajectory is y=ztna-—_ _, 2vg cos? x @ being the variable parameter. . Ans. The parabola y = 17. Find the equation of the envelope of each of the following family of curves, t being the parameter ; draw the family and the envelope: (@) @—-tP+y=1-#. @) @-H?+y=4t (b) 24+ y—HP=2t () 2+ y—9=4—8. (©) @—?+y=4e-1 (%) @— 98+ — 0? () 4+ y— P= 48. @ y=a+e. () = 2y 4th () yat4i. (h) ate +20. ) @— HF + +O? (m) y= Cr +t. (a) y= t@e— 20. (0) onten @) @- O° + 47 CHAPTER XVII SERIES 134, Introduction. A series is a succession of separate numbers which is formed according to some rule or law. Each number is called a term of the series. Thus 2 4 8 +, 2? is a series whose law of formation is that each term after the first is found by multiplying the preceding term by 2; hence we may write down as many more terms of the series as we please, and any particu- lar term of the series may be found by substituting the number of that term in the series for n in the expression 2"~1, which is called the general or nth term of the series. EXAMPLES In the following six series: (a) Discover by inspection a law of formation ; (b) write down several terms more in each ; (c) find the nth or general term. Series nth term 1.1, 8,9, 2%, +++ a3, 2a, ta, al, Fate (- a). 3.1, 4, 9, 16, n. ae = 234 7 Cory 4(- ye. a CM — 2’ 5’ 10 w+ ‘Write down the first four terms of each series whose nth or general term is given below : nth term Series 1. wan, x, 4a, 923, 1624, en eae ao ae “14a 2 14Vvo 14V8 14-V4 9, 22, oe ned 2) 9" 28° 65 212 SERIES 213 4 10. 2. a an, (ogaynan loga-a? logta-2* logta zt TR 2 C 2. an . aa “EB OE 135. Infinite series. Consider the series of m terms 111 1 (4) Ly pyc ge and let S, denote the sum “ the series. Then ao 1 () S.= 141 Grades te got Evidently S, is a function of x, for when n=1, S,=1 1 when n= 2, s=14} =i, 3 Td when n= 3, 14543 =1}, ta 1 when n=4, S=14+5+7+3 =1f, 1 1 1 when n=, S,= 142 ptatgte +a mi" Mark off points on a straight line whose distances from a fixed point 0 correspond to these different sums. It is seen that the point n corresponding to any sum bisects the distance between the preceding point and 2. Hence it appears geometrically that when n increases without limit limit S,= 2. We also see that this is so from sth considerations, for limit g _ limit (9 _ n=o n=O Q- [since wen » nrenses without imte;1 approaches sero a8 tnt] * Found by 6, p. 1, for the sum of a geometric series. + Such a result is sometimes, for the sake of brevity, called the sum of the series; but the student must not forget that 2 is not the sum but the limit of the sum, as the number of terms increases without limit. 214 DIFFERENTIAL CALCULUS We have so far discussed only a particular series (4) when the number of terms increases without limit. Let us now consider the general problem, using the series (¢) Up Uy Uy te whose terms may be either positive or negative. Denoting by S, the sum of the first x terms, we have SU bu, + Ube ty and S, is a function of m. If we now let the number of terms (=) increase without limit, one of two things may happen: either Casn I. S, approaches a limit, say u, indicated by limit a init gus or Cask II. S, approaches no limit. In either case (C) is called an infinite series. In Case I the infinite series is said to be convergent and to converge to the value u, or to have the value u, or to have the sum u. The infinite geometric series dis- cussed at the beginning of this section is an example of a convergent series, and it converges to the value 2. In fact, the simplest example of a convergent series is the infinite geometric series a, ar, ar’, ar, art, +, where 7 is numerically less than unity. The sum of the first n terms of this series is, by 6, p. 1, gad) . l-r 1— ar* Ir If we now suppose n to increase without limit, the first fraction on the right-hand side remains unchanged, while the second approaches zero as a limit. Hence limit n=0 =F r a perfectly definite number in any given case. In Case II the infinite series is said to be nonconvergent.* Series under this head may be divided into two classes. Firsr Crass. Divergent series, in which the sum of n terms increases indefinitely in numerical value as n increases without limit; take for example the series S=14+ 2484-040 * Some writers use divergent as equivalent to nonconvergent. SERIES 215 As n increases without limit, S, increases without limit and there- fore the series is divergent. Suconp Cass. Oscillating series, of which |. ees is an example. Here S, is zero or unity according as n is even or odd, and although $, does not become infinite as increases without limit, it does not tend to a limit, but oscillates. It is evident that if all the terms of a series have the same sign, the series cannot oscillate. Since the sum of a converging series is a perfectly definite number, while such a thing as the sum of a nonconvergent series does not ex- ist, it follows at once that it is absolutely essential in any given prob- lem involving infinite series to determine whether or not the series is convergent. This is often a problem of great difficulty, and we shall consider only the simplest cases. 136. Existence of a limit. When a series is given we cannot in general, as in the case of a geometric series, actually find the number which is the limit of S, But although we may not -know how to compute the numerical value of that limit, it is of prime importance to know that a limit does exist, for otherwise the series may be non- convergent. When examining a series to determine whether or not it is convergent, the following theorems, which we state without proofs, are found to be of fundamental importance.* Theorem I. If S, is @ variable that always increases as n increases, but always remains less than some definite fixed number A, then as n increases without limit, S, will approach a definite limit which is not greater than A. Theorem II. If S, is a variable that always decreases as n increases, but always remains greater than some definite fixed number B, then asn increases without limit, S, will approach a definite limit which is not less than B. Theorem Ill. The necessary and sufficient condition that 8, shall approach some definite fixed number as a limit as n increases without limit is that a ne Srp 8, = 0 nse for all values of the integer p. * See Osgood’s Introduction to Infinite Series, pp.4, 14, 64. 216 DIFFERENTIAL CALCULUS 137. Fundamental test for convergence. Summing up first » and then n+p terms of a series, we have (4) SS Ut Uy t Uy ob Ue (B) Spay = Ut UH Ut byt Uy yah tty ps Subtracting (4) from (B), () Sap Si= Mags t aga h ot Ugg From Theorem III we know that the necessary and sufficient condi- tion that the series shall be convergent is that HE, Sere 5 =0 for every value of p. But this is the same as the left-hand mem- ber of (C); therefore from the right-hand member the condition may also be written ) ee Cntr t tinea tot ty) = Oe Since (D) is true for every value of p, then, letting p=1, a necessary condition for convergence is that i Claas) = 05 n=0 or, what amounts to the same thing, ( ee 0. Hence, if the general (or nth) term of a series does not approach zero as approaches infinity, we know at once that the series is non- convergent and we need proceed no further. However, (2) is not a sufficient. condition; that is, even if the nth term does approach zero, we cannot state positively that the series is convergent; for, consider the harmonic series 114 1 [20 oe eG a ron ae limit = ili =0: Here mit (a= sit (7) =0; that is, condition (Z) is fulfilled. Yet we may show that the harmonic series is not convergent by the following comparison: CP) UAE +HFE +E HEHEHE tds] t CG) E+EHU+FETS+ E+E Hie tet el toe SERIES 217 ‘We notice that every term of (@) is equal to or less than the cor- responding term of (7), so that the sum of any number of the first terms of (/") will be greater than the sum of the corresponding terms of (@). But since the sum of the terms grouped in each bracket in (G) equals 4, the sum of (@) may be made as large as we please by taking terms enough. The sum (G) increases indefinitely as the num- ber of terms increases without limit; hence (@), and therefore also (P), is divergent. We shall now proceed to deduce special tests which, as a rule, are easier to apply than the above theorems. 138. Comparison test for convergence. In many cases, an example of which was given in the last section, it is easy to determine whether or not a given series is convergent by comparing it term by term with another series whose character is known. Let (A) Ub Up Ube be a series of positive terms which it is desired to test for convergence. If a series of positive terms already known to be convergent, namely, (By aQtatater, can be found whose terms are never less than the corresponding terms in the series (A) to. be tested, then (A) is a convergent series and its sum does not exceed that of (B). Proof. Let 8S Ut Ut UH + Uy and Sat 0+ 04+ +4,3 and suppose that ao. 4 Then, since S,<4 and 8,55, it follows that s,1, Voici -_. 2p 3p 2p Bp 2p Beni’ 1 1 1 1 1 1 1 1 4 1 \2 bt tp tpt ete w= (Gh) u 1 50 < ae * 7 2 Be _ 1 7 2 1 \3 at at ant ant an tant ae ae (aes) and soon. Construct the series (H) I++ (Ga)'+ (Ga) When p>1, series (E) is a geometric series with the common ratio less than unity, and is therefore convergent, But the sum of (@) is less than the sum of (JZ), as shown by the above inequalities ; therefore (@) is also convergent. When p =1, series (@) becomes the harmonic series which we saw was divergent, and neither of the above tests apply. When p <1, the terms of series (@) will, after the first, be greater than the corre- sponding terms of the harmonic series; hence (@) is divergent, BP 139. Cauchy’s ratio test for convergence. Let “4 wbuybayte be a series of positive terms to be tested. SERIES 219 Divide any general term by the one that immediately precedes it; ite. form the test ratio “#1. u, As n increases without limit, let Hilt Mast — p I. When p <1. By the definition of a a ($18, p. 11) we can choose n so large, say n= m, that when n= m the ratio “=#1 shall differ from p by as little as we please, and therefore be less’ than a proper fraction 7, Hence Ung SUS Unga S Ung 2p k PSF Mg SUS and so on. Therefore, after the term x,, each term of the series (4) is less than the corresponding term of the geometrical s (@) tart ya aye boo ies But since r <1, the series (B), and therefore also the series (4), is convergent." II. When p> 1 (or p=). Following the same line of reasoning as in I, the series (A) may be shown to be divergent. III. When p=1, the series may be either convergent or divergent ; that is, there is no test. For, consider the p series, namely, ti a 1 i ltgtgtptotst (a+1y eee Ge “\a+1) n-+1)’ limit /t4.4;\__ limit i and Haale st) = meta = OTE Hence p=1, no matter what value p may have. But on p. 218 we showed that when p> 1, the series converges, and when p = 1, the series diverges. ‘Thus it appears that p can equal unity both for convergent and for divergent series, and the ratio test for convergence fails. ‘There are other tests to apply in cases like this, but the scope of our book does not admit of their consideration. * When examining a series for convergence we are at liberty to disregard any finite number of terms; the rejection of such terms would affect the value but not the existence of the limit, 220 DIFFERENTIAL CALCULUS Our results may then be stated as follows: Given the series of positive terms UAUA UH UH yg bees find the limit a (+) p. n=0\ u, I. When p<1,* the series is convergent. Il. When p>1, the series is divergent. III. When p=1, there is no test. 140, Alternating series. This is the name given to a series whose terms are alternately positive and negative. Such series occur’ fre- quently in practice and are of considerable importance. If UUs — Ub is an alternating series whose terms never increase in numerical value, and if limit w= 0, then the series is convergent. Proof. ‘The sum of 2n (an even number) terms may be written in the two forms CA) Say =, — Hy) + ly — HE) Fly — FF Cand — Mays OF (BY San= ty = (Ug Uy) = y= He) = Mane Since each difference is positive (if it is nob zero, and the assump- tion | w,= 0 excludes equality of the terms of the series), series (4) shows that S,, is positive and increases with n, while series (B) shows that ,, is always less than w,; therefore, by Theorem I, p. 215, S,,, must approach a limit less than w, when n increases, and the series is convergent. Tuwusrrative Examrne 4, Test the alternating series 1— Solution. Since each term is less in numerical value than the preceding one, and limit limit /1\_ nee = AE G)=% the series is convergent. 141. Absolute convergence. A series is said to be absolutely or unconditionally convergent when the series formed from it by making all its terms positive is convergent. Other convergent series are said # Tt is not enough that wn+1/tm becomes and remains less than unity for all values of 7, but this test requires that the dimit of w+1/un shall be less than unity. For instance, in the case of the harmonic series this ratio is always less than unity and yet the series diverges as we have seen. The limit, however, is not less than unity but equals unity. + The terms of the new series are the numerical (absolute) values of the terms of the given series, SERIES 221 to be not absolutely convergent or conditionally convergent. To this latter class belong some convergent alternating series. For example, the series i Bee is absolutely convergent, since the series (C), p. 217, namely, 1,1,1,1 ltgtptptpt is convergent. The series ie ie - ooo Yoetsas is conditionally convergent, since the harmonic series ti i. Sateen eee is divergent. A series with terms of different signs is convergent if the series deduced from it by making all the signs positive is convergent. The proof of this theorem is omitted. Assuming that the ratio test on p. 219 holds without placing any restriction on the signs of the terms of a series, we may summarize our results in the following General directions for testing the series UA UA UA UA FU AU gree When it is an alternating series whose terms never increase in numer- ical value, and if itt 40, then the series is convergent. In any series in which the above conditions are not satisfied, we deter- mine the form of u, and u,,, and calculate the limit limit (24.41) _ waa “s#)=6 I. When|p|<1, the series is absolutely convergent. IL. When |p|>1, the series is divergent. IIL. When |p|=1, there is no test, and we should compare the series with some series which we know to be convergent, as atartar+ar+-3 r<1, (geometric series) l4gtEtptes p>, Cp series) 222 DIFFERENTIAL CALCULUS or compare the given series with some series which is known to be di ivergen os DAL te 2°34 1,1,1 . a (p series) Intusrative Exampre 1, Test the series Solution. Here limit (“2 ’) — limit =m | n= 0) and by I, p. 221, the series is convergent. ue B Inpusrrative Examrre 2. Test the series — + = 4+— + 10" 108 * 108 ln Solution. Hi . lution, Here =e z, limit (um+2) _ limit (MAT 10° “ns ol, )~ n= ott * Tr, and by II, p. 221, the series is divergent, Inusrrative Examrie 3, Test the series (e) 1 1 Grp’ =" Gap nenty oft (Ss)=22S[ettnactal 2 ‘This being an indeterminate form, we evaluate it, using the rule on p, 174, Solution. Here wu, = oan limit sn) o ferentiat Bees) Differentiating, limit G2= 3 2 Differentiating again, = @ Ss This gives no test (III, p. 221). But if we compare series (@) with (@), p. 218, making p = 2, namely, 14 D) if @) tats we see that (0) must be convergent, since its terms are less than the corresponding terms of (D), which was proved convergent. L t+gtes SERIES 223 EXAMPLES Show that the following ten series are convergent: 1,1,1 Latgtgt Show that the following four series are divergent : 1 Be 1. ded qtgtes 1B. Gtigtigte 142 143 1+ 12, 14— + —— taetietige 14. 1ltioteteters. +iehete 142. Power series. A series of ascending integral powers of a vari- able, say 2, of the form (4) ataetag tag'+-, where the coefficients a,, a, a,,--- are independent of a, is called a power series in x. Such series are of prime importance in the further study of the Calculus. In special cases a power series in 2 may converge for all values of 2, but in general it will converge for some values of 2 and be divergent for other values of z. We shall examine (4) only for the case when the ficients are such that)... coefficients are such that yj: ni (x Jan 4, n= where L is a definite number. In (A) limit (=) limit (s2e")- limit Hee tet) a= Tn n=l wu, =o as" n=l a, Referring to tests I, II, IIL, on p. 221, we have in this case p= La, and hence the series (A) is I. Absolutely convergent when | Z2|<1, or |2|< lz IL. Divergent when | Zz|>1, or | III. No test when | Lz 1, or 224 DIFFERENTIAL CALCULUS We may then write down the following General directions for finding the interval of convergence of the power series, (A) a+ ae +a + age +o. Finsr Svar. Write down the series formed by coefficients, namely, AH A+ AF aH HOA yy bos Seconp Step. Calculate the limit limit (a41) _ 7, n=o\ a, ° Turep Srer. Then the power series (A) is I. Absolutely convergent for all values of x lying between 1 1 md +5 L IL. Divergent for all values of x less than —| =] or greater than +| = IU. No test when w= + t but then we substitute these two values of x inthe power series (A) and apply to them the general directions on p. 221. Nore. When L=0, + ; =40 and the power series is absolutely convergent for all values of x. Inuusrrative Exampte 1, Find the interval of convergence for the series () e-S+ Solution. First step. The series formed by the coefficients is 1 1 7 (©) 1-3t+po-at limit (dn Second step. ae (2) Differentiating, Differentiating again, Third step. ac By I the series is absolutely convergent when z lies between —1 and +1. By IL the series is divergent when z is less than —1 or greater than + 1. By III there is no test when z = +1. SERIES 225 Substituting = 1 in (B), we get which is an alternating series that converges. Substituting # =— 1 in (B), we get - n Be which is convergent by comparison with the p series (p >1). ‘The series in the above example is said to have [— 1, 1] as the interval of conver- gence. ‘This may be written — 1 z $1, or indicated graphically as follows : EXAMPLES For what values of the variable are the following series Graphical representations of Intervals of convergence * convergent ? 15. lt atatgatpee. ee We 2 ee Oh, Ans, —1 t ‘Hint. Neither the sine nor cosine can exceed 1 numer- ically. alogta , 2 logta -2 +0 24, 142k = oo —_1—_> teloga tt + Ans, All values of 2, 26, 2T. 28, 29, 10x + 10022 + 10002% ++ 30. 1424 [2a? + [328 +. * End points that are not included in the interval of convergence have circles drawn about them. CHAPTER XVII EXPANSION OF FUNCTIONS 143, Introduction. The student is already familiar with some methods of expanding certain functions into series. Thus, by the Binomial ‘Theorem, (4) Catay=at+datet 6 are + 4ac?+2%, giving a finite power series from which the exact value of (a+)! for any value of 2 may be calculated. Also by actual division, (By oe tote tete (cle, we get an equivalent series, all of whose coefficients except that of 2" are constants, n being a positive integer. Suppose we wish to calculate the value of this function when x=.5, not by substituting directly in 1 l-« but by substituting «=.5 in the equivalent series co Gteteta tte y+ (ee er =8, (C) gives for 7=.5 (2) — =1.9921875 + .0078125. Assuming 1 Jf we then assume the value of the function to be the sum of the first eight terms of series (C), the error we make is .0078125. However, in case we need the value of the function correct to two decimal places only, the number 1.99 is as close an approximation to the true value as we care for, since the error is less than .01. It is evident that if a greater degree of accuracy is desired, all we need to do is to use more terms of the power series (2) Ltt orate 227 228 DIFFERENTIAL CALCULUS Since, however, we see at once that [-l. there is no necessity for the above discussion, except for purposes of illustration. As a matter of fact the process of computing the value of a function from an equivalent series into which it has been expanded is of the greatest practical importance, the values of the elementary transcendental functions such as the sine, cosine, logarithm, ete., being computed most simply in this way. So far we have leamed how to expand only a few special forms into series; we shall now consider a method of expansion applicable to an extensive and important class of functions and called Zaylor’s Theorem. 144, Taylor’s Theorem * and Taylor's Series. Replacing 6 by x in (2), p. 167, the extended theorem of the mean takes on the form 61) £) =O r+ TE ro+ a+ Gaon po-9(@) + Sarre: where «, lies between a and 2. (61), which is one of the most far- reaching theorems in the Calculus, is called Taylor's Theorem. We see that it expresses f(z) as the sum of a finite series in (2 — a). ‘The last term in (61), namely Ga oy 'F(a,), is sometimes called the remainder in Taylor's Theorem after n terms. If this remainder converges toward zero as the number of terms increases without limit, then the right-hand side of (61) becomes an infinite power series called Taylor's Series.t In that case we may write (61) in the form ©) SO) =f@)+ Sere + Pre + re tony and we say that the function has been expanded into a Taylor's Series. For all values of x for which the remainder approaches zero as 2 increases without limit, this series converges and its sum gives the exact value of f(z), because the difference (= the remainder) between the function and the sum of x terms of the series approaches the limit zero (§ 15, p. 18). * Also known as Taylor's Formula. 7 Published by Dr. Brook Taylor (1685-1731) in his Methodus Inerementorum, London, 115. EXPANSION OF FUNCTIONS 229 If the series converges for values of « for which the remainder does not approach zero as n increases without limit, then the limit of the sum of the series is not equal to the function f(a). The infinite series (62) represents the function for those values of 2, and those only, for which the remainder approaches zero as the num- Ber of terms increases without limit. It is usually easier to determine the interval of convergence of the series than that for which the remainder approaches zero; but in simple cases the two intervals are identical. When the values of a function and its successive derivatives are known for some value of the variable, as 2 =a, then (62) is used for finding the value of the function for values of 2 near a, and (62) is also called the expansion of f(2) in the vicinity of x =a. Iunusreative Examvze 1, Expand logs in powers of (¢ — 1). Solution. Sic) =log2, fl) =0; res raya; P@a-4, sy=-1; f'@= ‘ fl) =2. Substituting in (62), log =2—1—}(e—1)? 4+ 4(e—1)8— +. Ans, This converges for values of 2 between 0 and 2 and is the expansion of log in the vicinity of x =1, the remainder converging to zero, When a function of the sum of two numbers a and z is given, say f(a +), it is frequently desirable to expand the function into a power series in one of them, say z. For this pixpose we use another form of Taylor’s Series, got by replacing x by a +2 in (62), namely, 3) flat =K@ +O + po + Bre oon Invusrrative Examene 1, Expand sin (a+ 2) in powers of 2, Solution. Here S (a+ 2) =sin(a +2), Hence, placing 2=0, f(@ =sina, F(a) = cosa, F(a) =— sina, ra) Substituting in (61), sin (a + 2) = sina + Feosa — 230 DIFFERENTIAL CALCULUS. EXAMPLES * 1, Expand e* in powers of z—2. Ans, CREF 4 Ee bee y }. Expand 23 — 22? + 52—7 in powers of z — 1. Ans. — 34 4(@—1)+ @—1)? + @—1). 3, Expand $y? — 14y + 7in powers of y—3. Ans. — 8+ 4(y— 3) + 3(y— 3). 4, Expand 52 +7243 in powers of z—2 Ans. 874 27(¢—2) + 5(z—2)%. 5, Expand 42° — 1722 + 11a 4 2 in powers of # — 4, 6, Expand 5y! + Gy! —17y? + 18y — 20 in powers of y + 4, 7. Expand e in powers of 2 +1. 8, Expand sing in powers of # — a. 9, Expand cosz in powers of — a. 10, Expand cos (a + 2) in powers of «. 2 : Ans, cos(a+-2) = cosa —asina — Peon + Fy sing ++ 11, Expand log (x + h) in powers of a. oe Ans, log (e+) = logh + 5 — oa + toes. 12, Expand tan (x + 2) in powers of h. Ans, tan(e +h) = tang + hsects + Msectz tang +++». 13. Expand the following in powers of A. n(n—1) 4 ne=—Din—2) 2 3 (a) @ 4H) art meth wna 3)e BPTIE poe re 18 eto = — (b) (: 00 ) 145, Maclaurin’s Theorem and Maclaurin’s Series. A particular case of Taylor's Theorem is found by placing a= 0 in (61), p. 228, giving F=f) + ir +Br"® + wo ae _ where 2, lies between 0 and a, (64) is called Maclaurin’s Theorem. The right-hand member is evidently a series in x in the same sense that (61), p. 228, is a series in «—a. Placing a= 0 in (62), p. 228, we get Maclaurin’s Series,+ (8) Ff) =10)+ FF OF SF'O +E"® ten * In these examples we assume that the functions ean be developed into a power series. + Named after Colin Maclaurin (1698-1746), being first published in his Treatise of Fluzions, Edinburgh, 1742. The series is really due to Stirling (1692-1770). EXPANSION OF FUNCTIONS 231 a special case of Taylor’s Series that is very useful. The statements made concerning the remainder and the convergence of Taylor's Series apply with equal force to Maclaurin’s Series, the latter being merely a special case of the former. The student should not fail to note the importance of such an expansion as (65). In all practical computations results correct to a certain number of decimal places are sought, and since the process in question replaces a function perhaps difficult to calculate by an ordinary polynomial with constant coefficients, it is very useful in sim- plifying such computations. Of course we must use terms enough to give the desired degree of accuracy. In the case of an alternating series (§ 139, p. 218) the error made by stopping at any term is numerically less than that term, since the sum of the series after that term is numerically less than that term. Intusrrative Examrce 1. Expand cosz into an infinite power series and determine for what values of z it converges. Solution, Differentiating first and then placing z= 0, we got Fe) S(e) Fe) Fe) F(a) F*@) Pie) = ete., Substituting in (65), a at af (A) iG heel Comparing with Ex. 20, p. 225, we see that the series converges for all values of 2. In the same way for sina. (B) sin eo es 2 e-Rt ppt which converges for all values of « (IDx. 21, p. 225). * Since here f(z) = sin(x + 3) and f(a) =sin, (a . *): we have, by substituting in the last term of (64), p. 231, * nw mainder = sin(z, +22). O P: Pi aot ee OF AB a dt | @8 dt But from (#), . @) on a ba bx or, since from (F), and or oF or _ oF DM en aqg me wre Substituting in (@) from (7) and (I), ar, er FIQ=ha-+ka- ~) Oe at ay Replacing ro by Fo in (J), we get Or Or Or He ne pe 4 EE EF oF). PI) = ne oe +e rae sah tel ar mW EE Som EE 4 EE. (K) PM) = i a+ EE ee In the same way the third derivative is 7 te : (Ly) PMG) = WE 3 ee +8 we Bee and co on for higher derivatives. 242 DIFFERENTIAL CALCULUS When t= 0, we have from (D), (@); (7); (), (E), F(O)=f(, y), ie. F(A) is replaced by f(a, y)s oan get, RQOV=hE tee : F"(0)= wy a ot oF af ef, 7 ; : m= WEL + Bee ome EL ew, and so on. Substituting these results in (#), we get FCM UR) = Sew) + t(H n+ hy) Pip ot oF pot rc 2 tee +p(* ot thay t 5 oy? at (86) To get f(wth, y +h), replace t by 1 in (66), giving Taylor's Theorem ‘for a function of two independent variables, 6) Fath yt Haren +aZ sat Ot amt pitt +a(? pat t ht oe + which is the required expansion in powers of hand & Evidently (67) is also adapted to the expansion of f(z +h, y + #) in powers of x and y by simply interchanging 2 with h and y with k ‘Thus Sth y+ =I +e py (67a) 20 of of os +al*s oe +? Y Shoat vet Similarly, for three variables we shall find Ke + a + it (68) Seth y+h 24D =f Hz ths +p("Ss+ +echee TF one FE oxoy 7 af ane 2 . + at ute) , and go on for any number ef variables. EXPANSION OF FUNCTIONS 243 EXAMPLES L. Given f(e, y) = Aa? + Bry + Cy?, expand f(t +h, y + k) in powers of hand k. Solution, Loeacy By, Lapet2cy; en ey ey ee exoy ey? ‘The third and higher partial derivatives are‘all zero. Substituting in (67), Sth y +h) = Ast + Bry + Cy? + (2 Ax + By)h + (Be + 2Cy)k + Alé + Bk + C2. Ans. 2, Given f(z, y, 2)= Ax? + By? + C2°, expand f(z +1, y+ m, z+) in powers of 1, m,n. Solution. Loos, Loopy, Faoce; ou oy Gz op iso, Fw _ ae ae aeay ~ dyaa oxen ‘The third and higher partial derivatives are all zero. Substituting in (68), StL y tm, z+ n)= Act + By + C2 4 2 Axl + 2 Bym + 20m 4+ AP + Bn? + Cn? Ans, 3. Given f(e, y) = Va tan y, expand f(@ + h, y + &) in powers of h and k. 4, Given f(t, y, 2) = Aa? + By? + Oz? + Day + Eyz + Fen, expand f(e +h, y +k, 2+ D in powers of h, hy l 149. Maxima and minima of functions of two independent variables. The function f(a, y) is said to be a maximum at x= a, y=b when F(a 5) is greater than f(a, y) for all values of z and y in the neigh- borhood of a and 8. Similarly, f(a, 8) is said to be a minimum at x=a, y=6 when f(a, 6) is less than f(2, y) for all values of 2. and y in the neighborhood of a and 6. These definitions may be stated in analytical form as follows: If, for all values of & and & numerically less than some small posi- tive quantity, (A) Fath, B+K) f(a 6) =a negative number, then f(a, b) is a maximum value of f(a, y). TE (B) f(ath, b+) — f(a, b) =a positive number, then f(a, 6) is a minimum value of f(a 9)« ‘These statements may be interpreted geometrically as follows: a point P on the-surface oO 244 DIFFERENTIAL CALCULUS is a maximum point when it is “higher” than all other points on the surface in its neighborhood, the codrdinate plane XOY being assumed horizontal. Similarly, P’ is a minimum point on the surface when it is “ower” than all other points on the surface in its neighborhood. It is therefore evident that all vertical planes through P cut the surface in curves (as APB or DPE in the figure), each of which has a maximum ordinate 2(= MP) at P. In the same manner all vertical planes through P' cut the surface in curves (as BP'C or FP'G), each of which has a minimum ordinate 2(=NP’) at P’. Also, : any contour (as JIZJIC) cut out of the surface by a horizontal plane in the immediate neighborhood of P must be a small closed curve. Similarly, we have the contour LSRT near the minimum point P’. It was shown in §§ 81, 82, pp. 108, 109, that a necessary condition that a function of one variable should have a maximum or a minimum for a given value of the variable was that its first derivative should be zero for the given value of the variable. Similarly, for a function (2, y) of two independent variables, a necessary condition that f(a, 6) should be a maximum or a minimum (ie. a turning value) is that for e=a,y=b, © Foo oo ou oy Proof. Evidently (4) and (B) must hold when k= 0; that is, Fath, 1-F 5) is always negative or always positive for all values of A sufficiently small numerically. By §§ 81, 82, a necessary condition for this is that a F(z, B) shall vanish for z= a, or, what amounts to the same thing, 2 F(a y) shall vanish for x =a, y=6. Similarly, (4) and (B) must hold when =0, giving as a second necessary condition that 7 f(& y) shall vanish for 2 =a, y = 6. EXPANSION OF FUNCTIONS 245, In order to determine sufficient conditions that f(a, 6) shall be a maximum or a minimum, it is necessary to proceed to higher deriva- tives. To derive sufficient conditions for all cases is beyond the scope of this book.* ‘The following discussion, however, will suffice for all the problems given here. Expanding f(a +h, +4) by Taylor's Theorem, (67), p. 242, re- placing « by a and y by 0, we get @) Fath b+H= =F nant end + @(otram ee (te) R, where the partial derivatives are evaluated for 2a, y=b, and R denotes the sum of all the terms not written down. All such terms are of a degree higher than the second in A and k Since az =0 and z =0, from (€), p. 244, we get, after transpos- ing F(a, 8), 2) FO+L EDF D=5(PE +2 EE + wit)ee. If f(a, b) is a turning value, the expression on the left-hand side of (Z) must retain the same sign for all values of hand & sufficiently small in numerical value, —the negative sign for a maximum value (see (A), p- 243) and the positive sign for a minimum value (see (B), p. 243 ite. f(a, 6) will be a maximum or a minimum according as the right- hand side of (Z) is negative or positive. Now 2 is of a degree higher than the second in 4 and & Hence as A and & diminish in numerical value, it seems plausible to conclude that the nwmerical value of R will eventually become and remain less than the numerical value of the sum of the three terms of the second degree written down on the right-hand side of (E).+ Then the sign of the right-hand side (and therefore also of the left-hand side) will be the same as the sign of the expression af ef | per ' 2 Of (Fy Ieee De oe +e But from Algebra we know that the quadratic expression A+ 2 MKC +B always has the same sign as 4 (or B) when AB—C*> 0. * See Cours a’ Analyse, Vol. I, by C. Jordan. + Peano has shown that this conelusion does not always hold. See the article on “ Maxima, and Minima of Functions of Several Variables,” by Professor James Pierpont in the Bulletin of the American Mathematical Society, Vol. IV. 246 DIFFERENTIAL CALCULUS ef a a Applying this to (), 4=%%, p= -o c= = , and we see that (2), and therefore also the leftland Member of £2, has the same ay? a LC aa? ay? Baay Hence the following rule for finding maximum and minimum values of a function f(x, y)- Firsr Srue. Solve the simultaneous equations # 0. Sxconp Svar. Calculate for these values of x and y the value of i (S) a? ay? Brey, Tump Srer. The function will have a maximum if A>0 ond malo & eB) 0 and “(op F)\>0, BA” ay neither a maximum nor a minimum if A < 0. The question is undecided if A = 0.* ‘The student should notice that this rule does not necessarily give all maximum and minimum values. For a pair of values of x and y determined by the First Step may canse A to vanish, and may lead to a maximum or a minimum or neither. Further investigation is therefore necessary for such values. ‘Ihe rule is, however, sufficient for solving many important examples. The question of maxima and minima of functions of three or more independent variables must be left to more advanced treatises. Intustrative Examrne 1, Examine the function 8 azy — 2° — y% for maximum and minimum values. Solution. SG, y) = Bary af $_ ys, First step. $= Bay — 8a" = 0, 2 s02—ay'=0, Solving these two simultaneous equations, we get 0, aaa, y=0; ysa * The discussion of the text merely renders the given rule plausible. ‘The student should observe that the ease 4=0 is omitted in the discussion. EXPANSION OF FUNCTIONS 247 , ar ef Second step. men 8% pe 8 ah 2L_( BLY © ea? ey? \axay, Third step. When z = 0 and y=0, A= mum nor a minimum at (0, 0). When 2 =aand y =a, A=+ 27a; and since 94%, and there can be neither a maxi- er — 6a, we have the conditions for a maximum value of the function fulfilled at (a a). Substituting « the given function, wo get its maximum value equal to a°, , y=ain ILLUSTRATIVE be a maximum, ari 2. Divide a into three parts such that their product shall Solution. Let z = first part, y = second part ; then a— (« + y) —a2—y= third part, and the function to be examined is IG y= ty (a—2—Y). First step. aay 2a —¥ 9, Fae 2my—2=0. é * Solving simultaneously, we get as one pair of values «= z or or Second step. Sfa—2y, “2 =a-22-2y, 22; at ae % Ray : * A= 4ay—(a—22—2y). 2 2 Third step. When z= Sandy apa $ and since Hf = 24, it is seen that gs e ‘Therefore the third part is also > and 2 the maximum value of the product is _ : our product is a maximum when ¢ = EXAMPLES 1. Find the minimum value of 22 + zy + y® — ae — by. Ans. }(ab—a?— 0%). 2. Show that sing + siny + cos (¢+y) is a minimum when z=y = 5, and a 7 maximum when #=y =. 3. Show that zev+=siny has neither a maximuin nor a minimum, vp (ar + by +c ety td 5, Find the greatest rectangular parallelepiped that can be inscribed in an ellipsoid, ‘That is, find the maximum value of 8:xyz(= volume) subject to the condition 4. Show that the maximum value o isa +c. 8 abe 3v8 , and substitute the value of z from the equation of the ellipsoid. Thi e tr). *2=0, y=0 are not considered, since from the nature of the probleni we would then have a minimum, Ans. Hinr. Let w=2yz, gives where w is a function of only two vai 248 DIFFERENTIAL CALCULUS 6, Show that the surface of a rectangular parallelepiped of given volume is least when the solid is a cube, 7. Examine 2t + yt — 2? + ay —y? for maximum and minimum values. Ans. Maximum when 2 = 0, y= 0; minimum when z= y = + }, and when z =— y =44-V3. 8. Show that the most economical dimensions for a rectangular tank to hold a given volume are a square base and a depth equal to one half the side of the base. 9, The electric time constant of a cylindrical coil of wire is mayz any by +c where z is the mean radius, y is the difference between the internal and external radii, z is the axial length, and m, a,b, ¢ are known constants. The volume of the coil is neyz =g. Find the values of z, y, 2 which make ua minimum if the volume of the coil is fixed, Ans, an = by = c= yO" CHAPTER XIX ASYMPTOTES. SINGULAR POINTS 150. Rectilinear asymptotes. An asymptote to a curve is the limit- ing position* of a tangent whose point of contact moves off to an infinite distance from the origin.t Thus, in the hyperbola, the asymptote ¥ AB is the limiting position of the tangent PT as the point of contact P moves off to the right to an infinite distance. In the case of algebraic curves the following definition is useful: an asymptote is the limiting position of a secant as two points of intersection of the secant with a branch of the curve move off in the same direction along that branch to an infinite distance. For example, the asymptote AB is the limiting posi- tion of the secant PQ as P and Q move upwards to an infinite distance. 151. Asymptotes found by method of limiting intercepts. The equa- tion of the tangent to a curve at (2, y,) is, by (1), p. 76, (P d y-9,= B@-a). 1 First placing y=0 and solving for 2, and then placing z= 0 and solving for y, and denoting the intercepts by 2, and y, respectively, we get de, — Igy intercept on OX Ww=H— ae = intercept on OY. Since an asymptote must pass within a finite distance of the origin, one or both of these intercepts must approach finite values as limits when the point of contact (2,, y,) moves off to an infinite distance. If limit(z)=a and limit (y) =6, *A line that approaches a fixed straight line as a limiting position cannot be wholly at infinity ; hence it follows that an asymptote must pass within a finite distance of the origin. It is evident that a curve which has no infinite branch can kaye no real asymptote. tr, less precisely, an asymptote to a curve is sometimes defined as a tangent whose point of contact is at an infinite distance. 249 250 DILFERENTIAL CALCULUS then the equation of the asymptote is found by substituting the limit- ing values @ and 6 in the equation tive oo If only one of these limits exists, but limit () =m, then we have one intercept and the slope given, so that the equation of the asymptote is yometh, or x Lea m : 2 yf Incusrrative Exanrry 1. Find the asymptotes to the hyperbola ~ _. 2 8 dl dy Solution, SY = : ae @ eB ; Also z;=“ and y;=——; hence these intercepts are zero when ¢=y =o. @ Therefore the asymptotes pass through the origin (see figure on p. 249) and their equations are : y—O0=4-(e—O2), or ay=4be. Ans. a This method is frequently too complicated to be of practical use. The most convenient method of determining the asymptotes to alge- braic curves is given in the next section. 152, Method of determining asymptotes to algebraic curves. Given the algebraic equation in two variables, ) S@Y=0- If this equation when cleared of fractions and radicals is of degree n, then it may be arranged according to descending powers of one of the variables, say y, in the form (BY ay"+ bat oy t+ dat eat fy tte For ‘a given value of 2 this equation determines in general n values of y. =0.* * For use in this section the attention of the student is called to the following theorem from Algebra: Given an algebraic equation of degree , Ay" + By—1 4 Cy" 24 Dy 84 ‘When A approaches zero, one root (value of y) approaches 2. ‘When 4 and B approach zero, two roots approach ‘When 4, B, and C approach zero, three roots approach ©, ete. ASYMPTOTES 251 Case I. To determine the asymptotes to the curve (B) which are parallel to the codrdinate axes. Let us first investigate for asymptotes parallel to OY. The equation of any such asymptote is of the form (Ce) aah, and it must have two points of intersection with (2) having infinite ordinates. First. Suppose a is not zero in (2), that is, the term in y" is present. ‘Then for any finite value of 2, (B) gives n values of y, all (finite. Vence all such lines as (@) will intersect (B) in points having finite ordinates, and there are no asymptotes parallel to OY. Second. Next suppose a=0, but b and ¢ are not zero. Then we know from Algebra that one root (= y) of (2) is infinite for every finite value of 2; that is, any arbitrary line (C) intersects (B) at only one point having an infinite ordinate. If now, in addition, be te=0, or (D) e=—5s then the first two terms in (B) will drop out, and hence two of its roots are infinite, That is, (D) and (B) intersect in two points having infinite ordinates, and therefore (D) is the equation of an asymptote to (B) which is parallel to OY. Third. Vi a=b=c=0, there are two values of 2 that make y in (B) infinite, namely, those satisfying the equation (4) da? + ex + f= 0. Solving (2) for x, we yet two usymptotes parallel to OY, and so on in general. In the same way, by arranging f(z, y) according to descending powers of z, we may find the asymptotes parallel to OX. Hence the following rule for finding the asymptotes parallel to the codrdinate axis : Fist Srev. Eguate to zero the covficient of the highest power of x in the equation. This gives all asymptotes parallel to OX. Seconp Strep. Hquate to zero the coefficient of the highest power of y in the equation. This gives all asymptotes parallel to OY. Nore. Of course if one or both of these coefficients do not involve 2 (or y), they cannot be zero, and there will be no corresponding asymptote. 252 DIFFERENTIAL CALCULUS In.usrrative Exampte 1, Find the asymptotes of the curve a% = y (x — a)? Solution, Arranging the terms according to powers of z, ye? — (2ay + a*)2z + ay = Equating to zero the coefficient of the high- est power of 2, we get y = 0 as the asymptote parallel to OX, In fact, the asymptote coin- cides with the axis of z, Arranging the terms according to the powers of y, (w—a)?y — a = 0. Placing the coefficient of y equal to zero, we get z=a twice, showing that AB is a double asymptote parallel to OY. If this curve is examined for asymptotes oblique to the axes by the method explained below, it will be seen that there are none. Hence y= 0and x =a are the only asymptotes of the given curve. Case II. To determine asymptotes oblique to the codrdinate axes. Given the algebraic equation @) F@ Y= Consider the straight line @ yam ke It is required to determine m and & so that the line (G) shall be an asymptote to the curve (F). Since an asymptote is the limiting position of a secant as two points of intersection on the same branch of the carve move off to an infinite distance, if we eliminate y between (7) and (G), the resulting equa- tion in 2, namely, ® SF (@ ma +k)=0, must have two infinite roots. But this requires that the coefficients of the two highest powers of z shall vanish. Equating these coefti- cients to zero, we get two equations from which the required values of m and k may be determined. Substituting these values in (@) gives the equation of an asymptote. Hence the following rule for finding asymptotes oblique to the coordinate axes: Finsr Srer. Replace y by ma +k in the yiven equation and expand. Sxcon Srup. Arrange the terms according to descending powers of x. Turrp Srer. Equate to zero the coefficients of the two highest powers* of x, and solve for m and k. If the term involving 2"—1 is missing, or if the value of m obtained by placing the first coefficient equal to zero causes the second coefficient to vanish, then by placing the coefti- cients of 2» and z"~2 equal to zero we obtain two equations from which the values of m. and & may be found. In this case we shall, in general, obtain two k's for each m, that is, pairs of parallel oblique asymptotes. Similarly, if the term in 2"~? is also missing, each value of m furnishes three parallel oblique asymptotes, and so on. ASYMPTOTES 253 Fourrn Srv. Substitute these values of m and k in y=meth This gives the required asymptotes. Inuusrrarive Examene 2, Examine y = 22% — 28 for asymptotes. Solution. Since none of the terms involve both 4 wand y, it is evident that there are no asymptotes parallel to the codrdinate axes. ‘To find the oblique asymptotes, eliminate y between the given equation and y = mx +k, This gives y| (me + 18 = 2 ax? — 08; and arranging the terms in powers of 2, (14 m?)23 + (8m — 2a)a% + 3i2ma + ke = 0. Placing the first two coefficients equal to zero, 14m?=0 and 8m%—2a=0, > Solving, we get m=—1, k= x. Substituting in y = ma + k, we have y =— 24 a the equation of asymptote AB. EXAMPLES Examine the first eight curves for asymptotes by the method of § 150, and the remaining ones by the method of § 151: lyse Ans. y = 0. Ans. y =0. 3. y= loge. 4. v=(142)- z 5. y= tang. n being any odd integer, 2 = = 1 6 y=e—1. a=0,y=0. 1 Pao 4a, ya=at2 8, Show that the parabola has no asymptotes. 9 aa —x, ytu=0. 10. The cissoid y? = —™ 2=2r Qr—-a LL. ya = yt +29, 12. (a? +1) = 28 @? — 1). 18. ¥? (@ — 2a) = 2 — a8. 2=2a, yak (eta). 14, aty? = at (o? $y’). n=da,y=ia. 1B. y(e?— Bbe + 20%) = 25— Bax? + a8, a=b, = 2b, y+ 8a=2480, a 16. y=c +gcaF 17, The folium x + y° — 3axy = 0. 18. The witch a%y = 4a2(2a—y). 19, zy? + 2%y =a? 2=0,y=0,c4+y=0. 2. a8 4 Qaty —ay?—2y8 +44? 4 2ayty=1. c+2y=0,e+y=1e—y= 254 DIFFERENTIAL CALCULUS 153, Asymptotes in polar codrdinates. Let,f(p, 8) =0 be the equa- tion of the curve PQ having the asymptote CD. As the asymptote must pass within a finite distance (as O#) of the origin, and the point of contact is at an infinite distance, it is evident that the radius vector O drawn to the point of contact is parallel to the asymptote, and the subtangent OZ is perpendicular to it. Or, more precisely, the distance of the asymptote from the origin is the limiting value of the polar subtangent as the point off contact moves off an infinite distance. To determine the asymptotes to a polar curve, proceed as follows: First Srer. Find from the equation of the curve the values of 8 which make p= oo.* These values of 8 give the directions of the asymptotes. Suconv Sree. Find the limit of the polar subtangent oe by (1), p- 86 as 6 approaches each such value, remembering that p approaches co at the same time. Tur Srer. If the limiting value of the polar subtangent is finite, there is a corresponding asymptote at that distance from the origin and parallel to the radius vector drawn to the point of contact. When this limit is pos- itive the asymptote is to the right of the origin, and when negative, to the left, looking in the direction of the infinite radius vector. EXAMPLES 1, Examine the hyperbolic spiral p = ; for asymnptotes. Solution, When @=0, p=o. Also“? =~; hence 2 : oe a Be wie # q ‘bt: Co ee ee subtangent = pt =F, — Fa o. limit [p29] __ q, which is finite. “4 ol? ap. It happens in this case that the subtangent is the same for all values of 8. ‘The curve has therefore an asymptote BC parallel to the initial line OA and at a dis- tance a above it. * If the equation can be written as a polynomial in p, these values of @ may be found by ‘equating to zero the coefficient of the highest power of p. SINGULAR POINTS 2 oe Examine the following curves for asymptotes: 2 pcos = acos24. Ans. There is an asymptote perpendicular to the initial line at a distance a to the left of the origin. 3. p=atand, Ans. ‘There are two asymptotes perpendicular to the initial line and at a dis- tance a from the origin, on either side of it, 4, The lituus p94 = a. Ans. The initial line. 5. p =asec20. a Ans. ‘There are four asymptotes at the same distance 5 from the origin, and inclined 45° to the initial line. 6. (p—a)sind =». Ans. There is an’asymptote parallel to the initial line at the distance b above it, 1. p= a(sec26 + tan26). Ans, Two asymptotes parallel to @ = , at distance a on each side of origin. 8, Show that the initial line is an asymptote to two branches of the curve p?sin @ = a? cos20. 9, Parabola p = Ans. There is no asymptote. a 154, Singular points. Given a curve whose equation is I@ y=. Any point on the curve for which Fo and Foo ax ay is called a singular point of the curve. All other points are called ordinary points of the curve. Since by (57a), p. 199, we have af dy & it is evident that ab a singular point the direction of the eurve (or tangent) is indeterminate, for the slope takes the form - In the next section it will be shown how tangents at such points may be found. 155. Determination of the tangent to an algebraic curve at a given point by inspection. If we transform the given equation to a néw set of parallel codrdinate axes having as origin the point in question on the curve, we know that the new equation will have no constant term. Hence it may be written in the form (A) SO y) = ax + by + (cx? + day + ey*) + Ft + gry + hay? + iy) ++ 256 DIFFERENTIAL CALCULUS ‘The equation of a tangent to the curve at the given point (now the origin) will be A () y=(2)x By (1), p- 76 Let y= mex be the equation of a line through the origin O and a second point P on the locus of (4). If then P approaches O along the curve, we have, from (B), ee © limit m=. Let 0 be an ordinary point. Then, by § 155, a and } do not both vanish, since at (0, 0), from (4), p. 255, Fug Yua ex ay Replace y in (A) by ma, divide out the factor 2, and let 2 approach zero as a limit. Then (4) will become * a+bm=0. Hence we have, from (B) and (C), ax + by = 0, the equation of the tangent, The left-hand member is seen to consist of the terms of the first degree in (4). When 0 is not an ordinary point we have a=b=0. Assume that ¢, d, ¢ do not all vanish, Then, proceeding as before (except that we divide out the factor 2*), we find, after letting 2 approach the limit zero, that (A) becomes co dat ne 0. or, from (C), : . ay oe ” eaal ol Substituting from (B), we see that (2) cx’ + day + ey* =0 is the equation of the pair of tangents at the origin. The left-hand member is seen to consist of the terms of the second degree in (4). Such a singular point of the curve is called a double point from the fact that there are two tangents to the curve at that point. * After dividing by x an algebraic equation in m remains whose coefficients are functions of z. If now z approaches zero as a limit, the theorem holds that one root of this equation in m will approach the limit - a+ b. SINGULAR POINTS ' 257 Since at (0, 0), from (4), of 26, oy it is evident that (D) may be written in the form 2 2, 2 om Bashan In the same manner, if a=b=e= =0, there is a triple point at the origin, the equation of the three tangents being SE + yxy + hay + iy’= 0, and so on in general. If we wish to investigate the appearance of a curve at a given point, it is of fundamental importance to solve the tangent problem for that point. The above results indicate that this can be done by simple inspection after we have transformed the origin to that point Hence we have the following rule for finding the tangents at a given point. First Srer. Trangform the origin to the point in question. Szconn Srer. Arrange the terms of the resulting equation according to ascending powers of x and y. Turrp Srer. Set the group of terms of lowest degree equal to zero. This gives the equation of the tangents at the point (origin). Iviusrrative Exampte 1, Find the equation of the tangent to the ellipse 5a? 4 5y? + 2ay —12¢—12y=0 at the origin, Solution. Placing the terms of lowest (first) degr equal to zero, we get —122-12y=0, or a+y=0, which is then the equation of the tangent PT'at the origin, Inusrrarive Examrie 2, Examine the curve y B Ba? — ay — 2y? + 23 — By = 0 for tangents at the origin, Solution. Placing the terms of lowest (second) degree equal to zero, ¥ 82?— ay — 2y°=0, or (@— y) (82 + 2y) =0, ‘D 2z—y=0 being the equation of the tangent AB, and 32+ 2y=0 the equation of the tangent CD. The origin is, then, a double point of the curve. 258 DIFFERENTIAL CALCULUS Since the roots of the quadratic equation (), p. 257, namely, oF (AY 1 9 OF (aw) | oF ieae) . ion) : may be real and unequal, real and equal, or imaginary, there are three cases of double points to be considered, according as ay aes @ (a) ant ay? is positive, zero, or negative (see 8 p. 1). 156. Nodes af\ _ PF PF 4 7 : axoy) ax ay" In this case there are two real and unequal values of the slope (- w) found from (FF), so that we have two distinct real tangents to the curve at the singular point in question. This means that the curve passes through the point in two different directions, or, in other words, two branches of the curve cross at this point. Such a singular point we call a real double point of the curve, or a node. Hence the conditions to be satisfied at a node are we ve, (25) afore = = 9 ,y)=0, = = pe EEE FH 9) ox % oy oxoy, ox? oy? > Intusrrative Exawpre 1, Examine the lemniscate y* = 2% — a! for singular points, Solution, Here S@ Y= Al Yo ex44a0, Lioay=o. 1s0 a+ 425 nl Ya 4at=0. ‘The point (0, 0) is a singular point, since its codrdinates satisfy the above three equations. We have at (0, 0) B py led ry Led a ' . (4) -#8- * Naway) ~ ant ay? Fi : and the origin is a double point (node) through which two branches of the curve pass in different directions. By placing the terms of the lowest (second) degree equal to zero we get Y—a=0, or y=zandy=—z, the equations of the two tangents AB and CD at the singular point or node (0, 0). SINGULAR POINTS 259 af otf of 157. Cusps. eae ae ay In this case there are two real and equal values of the slope found from (J); hence there are two coincident tangents. ‘This means that the two branches of the curve which pass through the point are tangent. When the curve recedes from the tangent in both directions from the point of tangency, the singular point is called a point of osculation ; if it recedes from the point of tangency in one direction only, it is called a cusp. There are two kinds of cusps. First kind. When the two branches lie on opposite sides of the common. tangent. Second kind. When the two branches lie on the same side of the common. tangent.* The following examples illustrate how we may determine the nature of singular points coming under this head. Invusrrative Examrne 1. Examine aly? @x4 — 2° for singular points. Solution. Here Sey) = ay? — xt + 28 = 0, F gaat 4 oct=0, Yo 2aty ex. ey and (0, 0) is a singular point, since it satisfies the above three equations. Also, at (0, 0) we have or wee 2, of _oas, a and since the curve is symmetrical with respect to OY, the origin is point of osculation. Placing the terms of lowest (second) degree equal to zero, we get y? = 0, showing that the two common tangents coincide with OX, y Innustrative Examere 2, Examine y? = 2* for singular points. Solution. Here Ie yn=¥— B= af of a L_-se=0, Za2y=0, a y y O| = showing that (0,0) is a singular point. Also, at (0, 0) we have ere ae oy : (2)- ef ef _ 9 aray* ey? * \avay) ~ ex? ay?” ‘This is not a point of osculation, however, for if we solve the given equation for y we get y=avai, ‘* Meaning in the neighborhood of the singular point. 260 DIFFERENTIAL CALCULUS which shows that the curve extends to the right only of OY, for negative values of 2 make y imaginary. ‘The origin is therefore a cusp, and since the branches lie on oppo- site sides of the common tangent, it is a cusp of the first kind, Placing the terms of lowest (second) degree equal to zero, we get y = 0, showing that the two common tangents coincide with OX. Ippusrrative Exampie 8. Examine (y—2")? =a for * singular points. Solution. Proceeding as in the last example, we find a cusp at (0, 0), the common tangents to the two branches coinciding with OX. Solving for y, yowsal. If we let « take on any value between 0 and 1, y takes on two different positive values, showing that in the vicinity of the origin both branches lie above the common tangent. Hence the singular point (0,0) is a cusp of the second kind, en 158. Conjugate or isolated points. ( oF ) aes axay) ~ ax? ay’ In this case the values of the slope found are imaginary. Hence there are no real tangents; the singular point is the real intersection of imaginary branches of the curve, and the codrdinates of no other real point in the immediate vicinity satisfy the equation of the curve. Such an isolated point is called a conjugate point. Invusrramive Exanre 1, Examine the curve y?=23—2" for singular points. Solution. Here (0, 0) is found to be a singular point of the curve at which @ =4-V=T. Hence the origin is a conjugate point. Solving the <0. x al dx equation for 7, yoteVe-1. This shows clearly that the origin is an isolated point of the curve, for no values of x between 0 and 1 give real values of y. 159, Transcendental singularities. A curve whose equation involves transcendental functions is called a transcendental curve. Such a curve may have an end point at which it terminates abruptly, caused by a discontinuity in the function; ora salient point at which two branches of the curve terminate without having acommon tan- gent, caused by a discontinuity in the derivative. Inuusrrative Exanrie 1. Show that y = 2 logs has an end point at the origin. Solution. x cannot be negative, since negative numbers ~O} have no logarithms; hence the curve extends only to the right of OY, When 2=0, y=0. There being only one value of y for each positive value of «, the curve consists of a single branch terminating at the origin, which is therefore an end point. SINGULAR POINTS 261 Innusrraive Examrie 2, Show that y= zhas a salient point at the origin, 7 14é Solution, Here = —— +". lee elton? If « is positive and approaches zero as a limit, we have ultimately | y=0 and =o. ‘4 If z is negative and approaches zero as a limit, we get ultimately wy =0 ama & : ae Hence at the origin two branches meet, one having OX as its tangent and the other, AB, making an angle of 45° with OX, EXAMPLES 1. Show that 7? = 22? + 2° has a node at the origin, the slopes of the tangents being +-V2. 2. Show that the origin is a node of y2(a? + 2°) gents bisect the angles between the axes. 3. Prove that (a, 0) isa node of y* = 2 (« — a), and that the slopes of the tangents are Va. 4, Prove that a®y?— 2 aba*y — 25 = 0 has a point of osculation at the origin, 5, Show that the curve y* = z* + <* has a point of osculation at the origin. a (a — and that the tan 6. Show that the cissoid has a cusp of the first kind at the origin, Qa—a 7. Show that y? = 2a2* — a has a cusp of the first kind at the origin. 8, In the curve (y — 2°) = 2* show that the origin is a cusp of the first or second kind according as n is < or > 4, 9. Prove that the curve et — 2az%y — azy? + ay? =0 has a cusp of the second kind at the origin. 10, Show that the origin is a conjugate point on the curve y? (2? — a’) = 2? 11. Show that the curve 9 = 2 (a + 2)* has a conjugate point at (— a, 0). 12. Show that the origin is a conjugate point on the curve ay? — a? + be and b have the same sign, and a node when they have opposite signs. 13. Show that the curve at + 2az%y — ay? = 0 has a trizle point at the origin, and that the slopes of the tangents are 0, +-V2, and —V2. 14, Show that the points of intersection of the curve (2) (“f= 1 with the axes are cusps of the first kind, oA 15. Show that no curve of the second or third degree in « and y can have a cusp of the second kind, 16. Show that y=e * has an end point at the origin. 0 when 17. Show that y = zarctan 3 has a salient point at the origin, the slopes of the tangents being +: a CHAPTER XX APPLICATIONS TO GEOMETRY OF SPACE 160, Tangent line and normal plane to a skew curve whose equations are given in parametric form. The student is already familiar with the paranietric representation of a plane curve. In order to extend this notion to curves in space, let the codrdinates of any point P (2, y, 2) on a skew curve be given as functions of some fourth variable which we shall denote by #, thus, A) 2=6©, y=¥O, 2=xO- The elimination of the parameter ¢ between these equations two by two will give us the equations of the pro- jecting cylinders of the curve on the ¥ coordinate planes. Let the point P(2, y, 2) correspond to the value ¢ of the param- eter, and the point P!(@+Ax, yt+Ay, 2+Az) correspond to the value t+At; Az, Ay, Az being the increments of 2, y, z due to the incre- ment At as found from equations (4). From Analytic Geometry of three dimensions we know that the direction cosines of the secant (diagonal) PP! are proportional to Az, Ay, Aes or, dividing through by At and denoting the direction angles of the pee ye secant by a’, B!, 9’, Ae Ay Ae cos a! : cos fs eos ys: 53 Us SS pAzutAyz+Az) Now let P! approach P along the curve. Then Aé, and therefore also Ax, Ay, Az, will approach zero as a limit, the secant PP' will approach the tangent line to the curve at P as a limiting position, and we shall have cos : cos 8: cos yi: 2; WY dt dt dt’ 262 APPLICATIONS TO GEOMETRY OF SPACE 263 where a, 8, y are the direction angles of the tangent (or curve) at P. Hence the equations of the tangent line to the curve z=$(), y=¥O, 2=xXO at the point (2, y, 2) are given by X-x (69) ae dt dt dt and the equation of the normal plane, i.e. the plane passing through (2, y, 2) perpendicular to the tangent, is dx dy a _ (70) ee a Ce ae ee X, ¥, Z being the variable coordinates. Intusrrative Exampre 1, Find the equations of the tangent and the equation of the normal plane to the helix (9 being the parameter) = a.c0s8, y=asind, 2=0, {a) at any points (b) when @ = 27. ae, dy Solution. Brn Ng =— vy, h= ac089 = Substituting in (69) and (70), we get at (2, y, z) 2 Zant _ You _ 2-2» tangent line; # -y oe o and —y(X—2) +2(¥—y) + 6(Z—2) =0, normal plane, When 0 = 27, the point of contact is (a, 0, 20m), 7 giving 2b Ip, N 8 a or, X=a, b¥=aZ—2abn, y ‘the equations of the tangent line; and a¥ + 0Z— 20% =0, the equation of the normal plane, * The helix may be defined as a curve traced on a right circular cylinder so as to cut all the elements at the same angle. Take OZ as the axis of the cylinder, and the point of starting in OX at Pp. Let a= radius of base of cylinder and = angle of rotation. By definition, ve Be = =k(const.), or z= ako. ao Let ak=b; then 2=0. Also y=MN=asin8, t= OM=acos6. 264 DIFFERENTIAL CALCULUS EXAMPLES Find the equations of the tangent line and the equation of the normal plane to each of the following skew curves at the point indicated : Le=24yat,2=4t; t=1. Ans. QesP—Lyattlz=8; 3. c=8—1,y=t+O,2=48—8t41; t=1. Ans, 2 = % yatt eye + ns. B= ; t+y4+82—8=0. 42—m_ V2y—-1_ V2z2-1 1 =1 162 4-V2y —V22—44=0. s . &= ty =sint, z= cost; t= 7. Ans. 5. o=at,y =P, 2 =e; t= 1. 6 o=t,y=1—-A,2=80 +40; ¢ Testysezse ts t=0. 8. =asint, y = beost, z 9. Find the direction cosines of the tangent to the curve 2 =, y=, z= tt at point ¢=1. 161. Tangent plane to a surface. A. straight line is said to be tan- gent to a surface at a point P if it is the limiting position of a secant: through P and a neighboring point P' on the surface, when P’ is made to approach P along the surface. We now proceed to establish a theorem of fundamental importance. Theorem. All tangent lines to a surface at a given point® lie in general in a plane called the tangent plane at that point. Proof. Let (4) F(x, y,2)=0 be the equation of the given surface, and let P(x, y, 2) be the given point on the surface. If now P! be made to approach P along a curve C lying on the surface and passing through P and P’, then evidently the secant PP’ approaches the position of a tangent to the curve C at P. Now let the equations of the curve C be ® t=o), y=¥O, 2=xO- + The point in question is assumed to be an ordinary (nonsingular) point of the surface, 0, FF, OF aro not all zero at the point. Ox! by’ ae . APPLICATIONS TO GEOMETRY OF SPACE 265 Then the equation (4) must be satisfied identically by these values, and since the total differential of (4) when 2, y, 2 are defined by (B) must vanish, we have OF dx | oF dy , oF d: (© fede t dy att ed ae di + ay de * 2 ai ~=—-BY (62), p. 196 This equation shows that the tangent line to C, whose direction cosines are proportional to aa aye as ade He is perpendicular * to a line whose direction cosines are determined by the ratios and since Cis any curve on the surface through P, it follows at once, if we replace the point P(2, y, 2) by R(x, yy #,), that all tangent lines to the surface at F lie in the planet oF, oF, oF, an (a, FO Ty OW ta E-4)=% which is then the formula for finding the equation of a plane tangent at (2p Yy %) to @ surface whose equation is given in the form F(a, y, 2=0. In case the equation of the surface is given in the form z = f(a, y), let Y FO nD=FG y) z= 0. oF of _a, OF _af_ ar _ = xtc oy by ay’ a -1 * From Solid Analytic Geometry we know that if two lines having the direction cosines 208 a4, €08 81, €08°7; And Cos Gy, CoS Ba, C08 “72 ate perpendicular, then 605 @ C08 ata + 608 By C05 By + C08 71 C08 Y2=0. | ' Fy OF aFy # The direetion cosines of the normal to the plane (71) are proportional to =") ty t+ ry’ Oy,’ Oe Hence from Analytic Geometry we seo that (C) is the condition that the tangents whose direction cosines are cos «, cos, cosy are perpendicular to the normal; i.e. the tangents must lie in the plane. } In agreement with our former practice, ORT OR OR ea oe Om," Oy,’ 04” Ox" On denote the values of the partial derivatives at the point (1, Y1, 41) 266 DIFFERENTIAL CALCULUS. If we evaluate these at (a, y, 2,) and substitute in (71), we get Zz, oz, (72) ax, 8%) + 5y, UW) — 4) = 0, which is then the formula for finding the equation of a plane tangent at (fp Yo %) t0 a surface whose equation is given in the form 2=f (a, y). In § 126, p. 197, we found (55) the total differential of a function u (or 2) of z and vy, namely, oz bz =—dz+—dy. (2) ety ‘We have now a means of interpreting this result geometrically, For the tangent plane to the surface z =/(2, y) at (¢, v, 2) is, from (72), &% F) er () +5 Fy, X, Y, Z denoting the variable codrdinates at any point on the plane. If we substitute X=a4de and Yoy+dy P(XY, in (F), there results 2 a bz ez @) Z— de + ay, (@) are Comparing (Z) and (@), we get i (H) dz=Z—z. Hence fe ‘Theorem. ‘The total differential Z i of a function f (w, y) corresponding 2 : to the increments dz and dy equals Z i the corresponding increment of the Z ¥ z-codrdinate of the tangent plane to the surface z = f(t, ¥)- o Ae BE ‘Thus, in'the figure, PP’ is the h plane tangent to surface PQ at “Y y POY 2)- . D Let AB=de and CD=dy; then. dz=Z—2=DP’— DE= EP’. 162. Normal line to a surface. The normal line to a surface at a given point is the line passing through the point perpendicular to the tangent plane to the surface at that point. The direction cosines of any line perpendicular to the tangent plane (71) are proportional to a Coys 0s: X-% YW _ 2-4 oF, oF, oF, ox, «= Y, OR, (73) “. APPLICATIONS TO GEOMETRY OF SPACE 267, are the equations of the normal line* to the surface F(x, y, 2)=0 at @y Yy &) Similarly, from (72), (74) are the equations af the normal line* to the surface 2 =f (a, y) at (2p Yyy2,)- EXAMPLES 1, Find the equation of the tangent plane and the equations of the normal line to the sphere 2? + y? + z= 14 at the point (1, 2 8). Solution. Let F(x, y, z) =a? + y?+22—14; oF oF oF Fmt ah Ske maa oF, 9 Fy _ 4 OF ae, ty Oy Substituting in (71), 2@—1)+4(y—2)+6@—8)=0, 2+2y482=14, the tangent plane, Substituting in (73), then 6. -1_y-2 2 4 giving z = 8 and 2z = 8y, equations of the normal line, 6 2, Find the equation of the tangent plane and the equations of the normal line to the ellipsoid 42? + 9y% + 862 = 36 at point of contact where z= 2, y= 1, and z is positive, Ans. Tangent plane, 8 (« — 2) + 9(y—1) + 6V11(¢ — 4 V11) = 0; @—-2 y-1_2-43vul normal line, 8 ovat 3, Find the equation of the tangent plane to the elliptic parabola z= 22% + 444 at the point (2, 1, 12). Ans, 82 4+ 8y—z2=12., 4, Find the equations of the normal line to the hyperboloid of one sheet at 4y? 422 =6 at Q, 2 8). Ans, y +40 =10, 82— 5. Find the equation of the tangent plane to the hyperboloid of two sheets a yt gt ac My Bo eT aT) At Gy ty a) Ans, 2 — it 6. Find the equation of the tangent plane at the point (x,, y,, 2,) on the surface ax? + by? + cz? +d=0, Ans. axe + yy +cz2+d= 7. Show that the equation of the plane tangent to the sphere py te 4 2le42My+2Nz+D=0 at the point (¢,, ¥4, 2) is aetyytzet+Letey+Metyyt+N@et+a)+D= * See second footnote, p. 265. 268 DIFFERENTIAL CALCULUS 8. Find the equation of the tangent plane at any point of the surface ait yly A=al, and show that the sum of the squares of the intercepts on the axes made by the tangent plane is constant. 9. Prove that the tetrahedron formed by the codrdinate planes and any tangent plane to the surface «yz = a? is of constant volume, 10. Find the equation of the tangent plane and the equations of the normal line to the following surfaces at the points indicated : (a) 2a? + 4y?@—2=0; 21,12). (a) Bat? +y2—22= (b) a? + 4y?@— 2 =16; (1,2, 1). (@) wy? 42042516; (0) + 4+2=11; (1,1). () a + By? 4 22 163. Another form of the equations of the tangent line to a skew curve. If the curve in question be the curve of intersection AB of the two surfaces F(a, y, 2)=0 and G(@, y 2)=0, the tangent line PT at P(&» Yy %) is the intersection of the tangent planes CD and C# at that point, for it is also tangent to both surfaces and hence must lie in both tangent planes. The equations of the two tangent planes at P are, from (71), oF, ox, yh mum) + oe! @-4)= (75) 2. —(- at nae wre Z—Z,)=0. ox, Taken simultaneously, the equations (75) are the equations of the tangent line PT to the skew curve AB. Equations (75) in more com- pact form are ee 2 OF, 0G, oF, 0G, aF, 0G, aF, 0G, aF, aG, Oy, 02, Dz, Oy, 92, OX, Ox, Dz, OH, OY, 7 eth ww “Tor, oF)’ oy, 02, Ox, OY, aG, aG, aG, aG, jay, 22, fax, Oy; using the notation of determinants. APPLICATIONS TO GEOMETRY OF SPACE 269 164. Another form of the equation of the normal plane to a skew curve. The normal plane to a skew curve at a given point has already been defined as the plane passing through that point perpendicular to the tangent line to the curve at that point. Thus, in the above figure, PHT is the normal plane to the curve AB at P. Since this plane is perpendicular to (77), we have at once aF, oF, oF, aF, ox, Oy, 0G, a6, jax, 24, (78) @-4)=0, the equation of the normal plane to a skew curve. EXAMPLES 1. Find the equations of the tangent line and the equation of the normal plane at (r, r, r-V2) to the curve of intersection of the sphere and cylinder whose equations are respectively 2? + y? + 2? = 40%, a? 4 y? =2re. Solution. Let F =a? + y* + 2*— 47 and G = 2var; or, y=r,2+ V2z2=8r, the equations of the tangent PT at P to the curve of intersection. Substituting in (78), we get the equation of the normal plane, — V2 (@—1) 40(¢y— 1) + @—1-V2) =0, or, Vig—-2=0. 2, Find the equations of the tangent line to the cirele + y+ 2 = 26, at2=5, at the point (2, 2-V3, 8). Ans. 20 4 2VBy + 82=25,242=5. 3. Find the equation of the normal plane to the curve +t ear, Bae +y=0, at yyy Me Ans, 2yat— (2a, — Nay — 2 = 0 270 DIFFERENTIAL CALCULUS 4, Find the equations of the tangent line and the normal plane to the curve Qar4 8y242=9, = Barty? at (1, — 1, 2). 5. Find the direction of the curve wyz=1, Y=e at the point (1, 1, 1). 6. What is the direction of the tangent to the curve yan, A=1—y at (0, 0, 1)? 7. The equations of a helix (spiral) are P+yar, =atan 2 y = Show that at the point (x, y,, 2) the equations of the tangent line are e@— x) +m (@—%)=0, ey — 1%) —% (@— 4) = 05 and the equation of the normal plane is ye — ty — e(2— 2%) =0. . , ay 8. A skew curve is formed by the intersection of the cone “+ %— == 0 and @ é the sphere 2? + y? + 2?= 73, Show that at the point (2,, y,, %) the equations of the tangent line to the curve are 2 (a? — B) a, (@ — 2) = e(@—P)y,y—m) and the equation of the normal plane is OCF Ayo — (P+ wary — 8 (a — Pays =0. @ (P+ cz, (2-2), (2+ a2) z(e— 4); CHAPTER XXTI CURVES FOR REFERENCE For the convenience of the student a number of the more common curves employed in the text are collected here. Cunicat ParaBoLa SemicupicaL PaRaBoLa Y| ¥ y=az’, Tue Wircn or AGNest Tur Crssorp or Diocies Y| O™ 1G) vy =4a(Qa—y). ¥Qa-2z)=2*. 271 272 DIFFERENTIAL CALCULUS, Tue Lemniscate or BERNOULLI Lo CANS G+ y= aay). p= a? cos 26. Crctom, Orprvany Case y| Oo x x= aare vers 4 —V2ay — yh. a z=a(0—sin 8), {jac — ort CarenaRy Y| y= hte, Tue Concuorp or Nicomepes ry=ytar ly). p=acscd +b, Cyctom, Vertex av Orrery z= aare vers %4.-V2 ay—y z= a(0+sin 8), {jac oma, PARABOLA 7 F ole x aty ya al, CURVES FOR REFERENCE 278 Hypocycior or Four Cusrs Evouure or Exiipse y| id QW : VV x at ytaal, (az)t + byt = ca? Bt x=acos’ 6, {ya eainte Carpio Forrum or Dxscarres ¥| x e+yparaavat iy a+ y'—8ary=0. p=a(1— cos 6). Sive Curve Cosine Curve Y| y=sing, y=cosa, 274 DIFFERENTIAL CALCULUS Limagon Srroruor ¥ y ~) and freer we tan 1+2- ] Let us now emphasize what is apparent from the preceding expla- nations, namely, that Differentiation and integration are inverse operations. Differentiating (C) gives () d fi S'\(@) dz =f'(@) de. Substituting the value of f/(2) de[= df(2)] from (BY in (C), we get @) [v@=re. Thorefore, considered as symbols of operation, Z a inverse t0 cach others ot if we ave using differentials, d and fare inverse to each other. * Called anti-differential by some writers. + Historically this sign is a distorted S, the initial letter of the word sum. Instead of defining integration as the inverse of differentiation, we may define it as a process of sum. mation, a very important notion which we will consider in Chapter XXVIII. { Some authors write this D;1/’(x) when they wish to emphasize the fact that it is an inverse operation. INTEGRATION 281 When d is followed by Uf they annul each other, as in (D), but when f is followed by d, as in (#2), that will not in general be the case unless we ignore the constant of integration. The reason for this will appear at once from the definition of the constant of integration given in the next section. 166. Constant of integration. Indefinite integral. From the pre- ceding section it follows that since d(a*) = 8.a°dr, we have [8 a%dz=2"; since d(x’ +2) =3.%dz, we have ['3 Pde =P +25 since d(2?—7) = 8 2°dz, we have f 8 atde = 2—7. In fact, since d@+0)=824de where ( is any arbitrary constant, we have f Batde= a +O. A constant @ arising in this way is called a constant of integration. Since we can give C as many values as we please, it follows that if a given differential expression has one integral, it has infinitely many differing only by constants. Hence [roe=sores and since C is unknown and indefinite, the expression f@te is called the indefinite integral of f'(x) dx. It is evident that if $ (x) is a function the derivative of which is F(@), then $(z)+, where C is any constant whatever, is likewise a function the derivative of which is f(#). Hence the Theorem. If two functions differ by a constant, they have the same derivative. It is, however, not obvious that if $(x) is a function the derivative of which is f(x), then all functions having the same derivative f(x) are of the form b@) +0, where C is any constant. In other words, there remains to be proved the * Constant here means that it is independent of the variable of integration. 282 INTEGRAL CALCULUS Converse theorem. If two functions have the same derivative, their difference is a constant. Proof. Let $(2) and y(z) be two functions having the common derivative f(2). Place F@)=$@)—¥(@); then ad . A "@=GZlr@-¥@]=-f@-S@ =. By hypothesis But from the Theorem of Mean Value (46), p. 166, we have F(a+Ar)—F (2) = Ark'(a+6-Ar). 0<0<1 o. F@+A2)—F(@)=0, [Since by (4) the derivative of F(z) is zero for all values of 2] and F@+Az) = F(2). This means that the function F@)=$@)-¥@ does not change in value at all when x takes on the increment Az, ie. $(2) and (2) differ only by a constant. In any given case the value of C can be found when we know the value of the integral for some value of the variable, and this will be illustrated by numerous examples in the next chapter. For the pres- ent we shall content ourselves with first learning how to find the indefinite integrals of given differential expressions. In what fol- lows we shall assume that every continous function has an indefinite integral, a statement the rigorous proof of which is beyond the scope of this book. For all elementary functions, however, the truth of the statement will appear in the chapters which follow. In all cases of indefinite integration the test to be applied in veri- fying the results is that the differential of the integral must be equal to the given differential expression. 167. Rules for integrating standard elementary forms. The Dif- ferential Caleulus furnished us with a General Rule for differentiation (p. 29). The Integral Calculus gives us no corresponding general rule that can be readily applied in practice for performing the inverse operation of integration.* Each case requires special treatment and we arrive at the integral of a given differential expression through * Byen though the integral of a given differential expression may he known to exist, yet it may not be possible for us actually to find it in terms of known functions, beeause there ave functions other than the elementary functions whose derivatives are elementary functions. INTEGRATION 283 our previous knowledge of the known results of differentiation. ‘That is, we must be able to answer the question, What function, when dif ferentiated, will yield the yiven differential expression 2 Integration then is essentially tentative process, and to expedite the work, tables of known integrals are formed called standard forms. To effect any integration we compare the given differential expression with these forms, and if it is found to be identical with one of them, the integral is known. If it is not identical with one of them, we strive to reduce it to one of the standard forms by various methods, many of which employ artifices which can be suggested by practice only. Accordingly a large portion of our treatise on the Integral Cal- culus will be devoted to the explanation of methods for integrating those functions which frequently appear in the process of solving practical problems. From any result of differentiation may always be derived a formula for integration. ‘The following two rules are useful in reducing differential expres- sions to standard forms: (a) The integral of any algebraic sum of differential expressions equals the same algebraic sum of the integrals of these expressions taken separately. Proof. Differentiating the expression fret fae fan w, v, w being functions of a single variable, we get du + dv — dw. By My, p. 34 ® vf Gu ao — a) fons fao— fav. (b) A constant factor may be written either before or after the integral sign. Proof. Differentiating the expression fa gives adv. By 1, p. 34 (2) oe fot= fa. On account of their importance we shall write the above two rules as formulas at the head of the following list of 284 @) 8) (4) 6) (6) 2) 8) @) (10) (4) (12) (13) (4) (15) (16) an INTEGRAL CALCULUS StanpArp ELEMENTARY Forms faut do— ay fare [ao fae, = log v + loge = log cv. (Placing C=10g0: -f etna S46. loga few=e+ Cc _fenvde=—cosv4 6. feosvdr=sinn+ 6. fetode=tanv +6. festvdn=—catv4 6. _fosevtanvdo=seov+C. _feseventedy=—cscvt 6 fran 0 dy= tog seoo 4 6. _fevtvdo= tog sinv + €. _f secv do= og (seev + tand) +6. foe dv = log (csc v — cot v) + C. INTEGRATION 285 a (18) Brg = gaetan2 +6. dv 1 v—a (19) Fo@= Be'Sata +¢. av v 20 =arcsin- +C. (20) aan arc sin? + €. ely JS Teer OHFEA 40. . (22) Ser erenite. 2 av— dv 1 (23) Searing te Proof of (8). Since g@p4.0)— dey Tr, p. 34 we get fonere Proof of (4). Since _/ yas a(S +6) Sade VI, p. 34 yp we we get fowasate This holds true for all values of m except n=—1. For, when n=—1, (4) gives ae aa 1 f Oates ee Gites gee 0 which has no meaning. The case when n=—1 comes under (5). Proof of (5). Since i ddogy+0)=2, Villa, p. 35 we get P logue, The results we get from (5) may be put in more compact form if we denote the constant of integration by loge. Thus [Par 9+ og e= loge Formula (5) states that if the expression under the integral sign is a Fraction whose numerator is the differential of the denominator, then the integral is the natural logarithm of the denominator. 286 INTEGRAL CALCULUS EXAMPLES* For formulas (1)-(6). Verify the following integrations: ast at tao 2 yout oo 1, fatas= Fo +0 7 +6, by (A), where v= 2 and n A 2. fVedo= frbao=P+0= by (4) where v= 2 and n=}, Mo fa-sae= 2S + o=— she, by (4) where v=2 and n=—8, 6 4. faside =a farde == +0. By (2) and (4) o ta=% . 5. frde=Z +0. 12. f Year=*F 40, 8at 6. fabar =" +0. 13, fsrtass2ve4c. 7. feta 24 46, 14. [ 8ap2aa = ub? + 0. c2 5 mez? a 1 15. [ Sm2ztde = +6. (je eo. 8 [anon at Is ne D : 16, f Me = BHP" sg, _ (2a _ Sat og, . Lara azy 2a den 1 10. fout = 58 40. 17, f(t) de = (nz) + C. aa —m=ady =— 11. { V8pede = 50 VOpe +C. 18. firmay ap te 19. f @2) — at 824 4)dx = fede — f5a%de (rede + fade by (1) =2fcar—5 forde—8 fade +4 fae by (2) at BaS 8a? 22 38 sarse. yy ttste. Nore. Although each separate integration requires an arbitrary constant, we write down only a single constant denoting their algebraic sum. by (1) =2afzbar—v fe-rdet Sofas by (2) wrat 1 tent ye by (4) stave 24 2ect ec. ‘* When learning to integrate, the student showld have oral drill in integrating simple funetions. INTEGRATION, 287 2 gat : 9 3a8 s =i A — 8. 1. 21. fee Ba! +120 — 8)de =" — “4 Bat— Bn +0. - f (ak — chan = ae + Pabst — 2 af 23. fw a8)de = ae + cabal — oa Hive. First expand. 24, [ev Vay = 298 (82 25. [ve- Vijsat = alt —2ath + sale ad @. 10 ga8 . (a? — 2)88de = 2 BE 4 ogo 7 26. fe 2yaddn = TF — E+ Bat — Bat + 0. 24 Patybeds = tbe 21. f (@ + Pa) beae = oe Te Hivz. This may be brought to form (4). For let v= 2+ 0%? and n= 3 then dv= 202d, If we now insert the constant factor 202 before zdz, and its reciprocal a before the integral sign (so as not to change the value of the expression), the expression may be integrated, using @, namely, ‘Thus, i (a2-+ B02) dda fe (a2+ veep havieden if (a2-+ 2x24 d (a2 + b222) 202 Aes be9)8 (2+ DF, my +n He Nore. The student is warned against transferring any function of the variable from one side of the integral sign to the other, since that would change the value of the integral. 28, [Vd — wade = f(a — aryhede =— yar — a1 + 6. 29. f Bae + 4bo)$ (20x + 4b2%) dz= 4 (Bac? + 402%) 40, Hixt. Use (4), making v= 3 az? +4 bx3, dv = (6 ax +1222) dx and n=4- 30. fb (Gaz? + 8b2!)f (ax + Abz%) de = bast +8d2)8 4.0, ode a1. f=" Sgt Hi. Write this f' (a2 + 28)" 4a2de and apply (4). (@ +2 40. ae 32. =-2VI=24+0. res 98. f2my(4 + 1)bay =F 2 +998 +0. 34. [teem =z0+eji+e. 35. foe cos rdz = f inayPcosedz Hivr. Use (4), making v=sin, dv=cos edz, and n=2 288 INTEGRAL CALCULUS 36. fcoste sin eds = ¢. 31. fsin?ax cos azde = = sin'az + C. 38, f cost 3 sin Bade = — 2 costBz +0. de = 29. [FS Ve—#+C. 5adt a 410. 7 =H Boyt 41. [Viv Pade = 504 et ec, sds 8 + 2. f Sar gar ates. witdu (a fouryi—m Ethos inGom to 2asds 4. [Gram Gos ~ aay Fe Sardz _ 3a By at = pale t ext) +0. 8 avd ade Solution. Veron = 80 faa By (2) ‘This resembles (5). For let v = 0? + 22; then dv =2@edx. If we introduce the factor 2¢? after the integral sign, and 54, before it, we have not changed the value é of the expression, but the numerator is now seen to be the differential of the denom- inator, Therefore SH tog Qe eat) 40. By () 2ende _ 8a cd? + ee?) 8a f= 34 (2 Cee) “Saran gel By ee zed B+ ext Flog (@2—1) +0. = a) de 8 + 47. pie SE = log — 8 aent4c. Batde + 48, Saenk Fig = 0g 102" + 15)8 +. 5 — — 662%) 7p los Ba — 6t2*) +0. — ty eeryre. ‘Hixr. First divide the numerator by the denominator. Qala, : 51. 7 Ppptt se loge +340. INTEGRATION 289 Prog er— na) +0. — 22 58. fet 5-540 levee. ¥ te ldt 54, = 1”) Sire Flog (a + 80) +6. 55. f (og at jlogayt +c. 2 Phare E+ rt Qlog(r—1) 40. br. Sa aloe ey +e. sin edz, 58. ms ~ flog (a+ b eosz) +0. sect a0 59. = . Scsmmo=3 glo + Stand) +0 60. fers Stet bogie 1) 40. 61. Far = log (e+1)@— 1 +. 62. Integrate the following and verify your results by differentiation: 2 wf (4e- 2)ae. 2 dz _ Aas i 22) an = 4 fete — =) . satution, (42 2)ds = 4 fatde— 2 f= aloga 4c. ee ie Y 2 Verification, ae 2loge +0) = Saat, ae = (102 2)a, mtn 222 ) fi atde. (h) f ‘smtnds. — (n) i = () i sint 2 e952, po : ayy 9 a7 (©) fovea. — @) fastag. (0) S< n : @ fea. w fe. © feode, ©) f Va= tear. © fiiav. a) f t-2at, _ eis . fone. 2245 acotg y 1)de ee © free. 0) fovBac. oo fee () Grea. se tat fe 5 (OS Fame MSoswt Proofs of (6) and ek ‘These follow at once from the corresponding formulas for differentiation, IX and IXq, p. 35. 290 INTEGRAL CALCULUS EXAMPLES For formulas (6) and (7). Verify the following integrations: bare Qloga Solution. fred = » farar. By (2) L f bat de = +0. ‘This resembles (6). Let v=22r; then dv=2dz. If we then insert the factor 2 before dz and the factor } before the integral sign, we have b b ed sade =? [arenas = fat ao ofa de =5 fo’ de pf er @a) 2 iogat By (6) 2. [aede = Ser. 1. fortes e400, 2 z eG 3. fetdz = ner +0. 8, fewae = +0. : oe 4. foimrcosade = eine + 0, 9. fa de = eg te i eat 5. fetonssin ade 10. four = Tate cetde 0 nL fo ee = gg +e. a) +0 log a 13. (forree et Ade = per rset8 40, wz ome 4. fw mde = aig 18. fre )dr= ae) +0. 16. Sor e-dy = $(@v— e-20) + 2y 40. (a=) arb a0 a1, fC ae = Tosglogh 22+ az a= 8-22 4 a8# + 8b-24) de = . Oe 4 * Bloga 2logd Lest e-tet 14 e-asae = 1fE% 4 goa gear c 19. flerse Bat aa Be- z|+ 20. Integrate the following and verify your results by differentiation : @) Si ‘ards. (O} Sf e- 8d, @ f ‘bettdz, (an) fora. ae « () fo-sae, ( frat, @ fewae. (ay fer as, (of ewae. () fseas. (%) fac-meae, © f@rrae. @ Pe. w fe. of o fe. INTEGRATION 291 (a) f ‘a2sind cos pido. © f ‘cos sin 640. (uy fi ‘etentsec? tt, 546% : tt © fe@re dz. () fe Szdr, ) fox : Proofs of (8)-(18). These follow at once from the corresponding formulas for differentiation, M1, ete., p. 35. Proof of (14). fe ci -(2 vde cose —sin vdv cos v d (cos v) ao =— log cos 0 +0 by (5) = log sec v +C. [since—ogeoren— tog low-loss] Proof of (15). f cot vd = [ss costed = (o~ ain») sino sino =heg sin +. By (5) sec v-+tanv sec v+ tan v _ sec v tan v+sec’v ~ secv-+tanv oo sec v tan v + sec’v dv “secu tano = fiesta v) see v + tan o =log(secv+tane)+0. By (6) Proof of (16). Since see v=see v 7 ese u—coty Proof of (17). Since csc v =cse vx. esc v—cotv _ = ese » cot v+ esc?v ese v — cot — ese v cot v + cse’v dL esc v — we v—cotv v . fie v—cot v) ese ¥— cote = log (ose v — cot v) +C. By (5) 292 INTEGRAL CALCULUS EXAMPLES For formulas (8)-(17). Verify the following integrations: 1. sin 2 aede =— wos2 a 0, Solution, This resembles (8). For let » = 2ax; then dv = 2adz. If we now insert the factor 2.@ before dx and the factor s before the integral sign, we get a fen 2azde = fsin 202-202 1 . z ao -d@azy)=1.— . pf sin 202 (2 a2) = 5 — cos 2ax + C. By (8) cos 2 ax, o. 2a . 2. feos made = si c. 7 b ayety = c. (feos made = 7 sin me + feseay cot ayay =— 7 ese ay + 1 3. Uf tam bade: = Flog secon + C. 8, fcse*Bude =— footdz +C. 4, fcc arde = Hog(secar-+ tanaz)+C. 9. foot dx = BlogsinS +0. Fae = ee cate a8 Pde = 2 5. fese% de = alog (eso cot?) +0, 10. [soot aide = } tana? $C. dz. 6. [see 8ttan Sidi = 450036 +0. nf wz. fe stans+e, cos? cote +C. 13. Uf (tan + cotayae = tan d — cot +0. 14, f (seoa— tan a)¥dar = 2(tan a — 00 a) — @ + C. 15, f (tan 28— 1)%ds = f tan 28 + log e088 +0. a a 16. f (cos§— sin 84) a9 = Bain + 5 con 8040. z 11, f (sinac-+ sin2) de 2 cos az — acos? +0. 18. ficosta+ bu)dy = Esin(a + by) +0. 19. feosecta® -atdz =— } cota? +0. de . loga) = = si @ 20, Seostogay S sin (log2) + dz a. (——— SrFeoz Hiv. Multiply both numerator and denominator by 1~ cos z and reduce before inte- grating. dx : =tang— @. 22. Uieaac tang — seex + cota + esow +O = tane +0, INTEGRATION 293 83. Integrate the following and verify the results by differentiation : (@ fain Bar. (ny Sa (0) J (mds— cot 3) as. : i) ftan® (p) f (cote — nae. (b) fore edz. (i) fo ge ii t—1)?dt. () f seo tan $ a9, 6) Ss ‘ese? (a — be) dx. 2 J ) wo fowttonttin, 0 fil © feosb+anac. — (y fH «) i (1— ese y)2ay. a 0 Siam cot Ty” of =. a fscc?2 azar, (on) f (s0020 ~ exe 8) a0, oy 2a, of Para (a) fang + seo g)tdp, — (¥) oa Proof of (18). Since 1 : ia G are tan + ¢) by XII, p. 85 4) we get if ee : +08 +a a =) uta, : 1 Proof of (19). Since x; f dw 1 ( Td Jas oa) =a aoa Flog (o—a)—log(r+ay}+o by qlee S40. sao a(2 are cot? ) and fH = —Harecot + O%. Hence vee ve Farotan’ + C——2 are cot? + 0", ara Since are tan? + are cot =, we see that one result may be easily transformed into the other. a ‘The same kind of discussion may be given for (20) involving are sin 2 and areeos®, and for (23) involving are sec? and are ese 2. + By breaking the fraction up into partial fractions (see Case I, p. 325). Bl 204 INTEGRAL CALCULUS Proof of (20). Since as d (we sin? +0) a by . 85 ey eS y XVILL, p. we get Jogrmets +0. Proof of (21). Assume v=atanz, where z is a new variable; differentiating, dv = a sec'zdz. Hence, by substitution, a asectade _ (*-_sectedz Vey Vetme+a J Vtan?2+1 sec edz = log (secz+tanz)+C by (16) = log(tan z+Vtan?z+1)+e By 28, p. 2 But tanz="; hence, v a = log(v+V0? + a#)—loga +e. Placing C =— log a+, we get Fee an ale vera @)+C, In the same manner, by assuming v= aseez, dv =asecz tan zdz we get = log (secz + tanz)+¢ by (16) = log(sec z +-Vsec?z—1)+¢ by 28, p. 2 S tog (+ 1) be= log v4 VF @)+C. Proofs of (22) and (28). These follow at once from the corre- sponding formulas for differentiation, XXII and XXIV, p. 36. INTEGRATION 295 ‘A lange number of the fractional forms to be integrated have a single term in the numerator, while the denominator is a quadratic expression with or without a square.root sign over it. The following outline will assist the student in choosing the right formula. NuMERATOR OF Frnst NuMERATOR oF ZERO DEGREE, DEGREE v ae Sl are tan 7 + 0, or, in denominator o aresin = +0, or, =log(v+ V8 a) +0 ot Radical foo= in denominator (=-9) Students should be drilled in integrating the simple forms orally and to tell by inspection what formulas may be applied in inte- grating examples chosen at random. EXAMPLES For formulas (18)-(23). Verify the following integrations : a1 Qe a Saar sy +6. Solution. ‘This resembles (18). For, let 0? = 422 and a? = 9; then v = 2, dv = 2dz, and a= 8. Hence if we multiply the numerator by 2 and divide in front of the integral sign by 2, we get —— 2dr _1_a@a) 42249 2) @z)?+ (3)? 2 a)? + (> = jane tan 22 +0. By (18) 3. Soxer pre iee garetanZ +0. a esr ante. aaa pyle sive. 5. [— = log (e+ VP— 9) +0. de vVe—9 | INTEGRAL 'CALCULUS) Bare sing? +0, = are vers ; +e dz ee er 1 Qe edt © gg Ht paresec Se +0. 4. Sz - etdt : . tan $0. 15. =aresine +C. : = game + Sa Tds 16. aresin 428+ C. SF a 3 WW. — log (Vaw + Vavt— 0) + 0. Sas = vi cosada _1 sin a 18, f SESE =F arotan (HE +0. 19. fae = wesin (loge) 4-6. T= logtz = (ut oF adz 23. — te Cn ae = jarewm dx 1 e+] a t c. lapses Hiwr, By completing the square in the denominator, this expregsion may be brought to a form similar to that of Ex. 17. Thus, By (18) Here v=2+1and a=2. de 25. =aresin SwwS Hinr. Bring this to the form of Ex. 16 by completing the square. Thus, a ad dx Saea Sl cen lea Here »=2—} and a4. 2 2etl 26. Sais vane te ae 1 7 Siz zy ae aeed INTEGRATION 297 & 1 ae, 28. = Sve SF = Tare sin8@ +3 4 ¢, ‘41 29, —— are sin (22 — 8) +0. ‘82 —a*-2 do 7 v—5 a C. OY iecters to 31, Sea 2y4+8-VB io P+8ytl Vo 2y+8+V5 a 1 32. (—# _ ato (434+ VErer) +0 ST AVE & 33. Sapcgegi = tone DFC. as 34, ( —S— = log(s+- a+ V2as +8) 40. wa (+ ex)de _b nie 37. [LEON = are tan= + Slog (a? + 2%) +C. ce a at gets) + Hint. A fraction with more than one term in the numerator may be broken up into the sum of two or more fractions having the several terms of the original numerator as numer tors, all the denominators being the same as the denominator of the original fraction. Thus, the last example may be written (b+ ex) de f bdx f i ax if Oren ae + of + ; att atta? atta? . e+e each term being integrated separately. = hog (at +9) 1 2 Saretan® +¢. garetan’ 5 av3— V2 = Ho (3.23 — 2) ave ——— + 6. BOR a eva4 va 40. Goneeeeae a. flee dx = Vt 4 4 + B8log(e+ V4) 40. Viv (Stat _5 pay ag, (EL=N4 _ 8 garg Vse—9 3 log (tV3 +-V38—9) 46. 298 INTEGRAL CALCULUS 43. Integrate the following expressions and verify your results by differentiation : fs VSS fA of Of © fits Sn © os SS mea Vase (Sates 0 fa Se oO fas foe. oy fae. o fGa08. wf vane” Oe OS GRS Onum Tas Saar 168, Trigonometric differentials. We shall now consider some trigo- nometric differentials of frequent occurrence which may be readily integrated by being transformed into standard forms by means of simple trigonometric reductions. Example I. Zo find i sin" cos" edn, When either m or n.is a positive odd integer, no matter what the other may be, this integration may be performed by means of formula (4), a f vdeo. For the integral is reducible to the form f (terms involving only cox) sin xd, when sin has the odd exponent, and to the form f (terme involving only sinz) coe nla, when cos has the odd exponent. We shall illustrate this by means of examples. INTEGRATION 299 Invesrravive Exampre 1, Find f sin? cos* nde, Solution. f'sin®z costade = f sinte cost coszdz = fointad. = sin?a)? coseda by 28, p. 2 = f (inte — 2sinta + sin’z) cosada = f (in 2)? cosade — 2 f (ain 2)* cosnde + fin 2)° coszda. sin? _ 2sin’z sin’x 3 5 ti ing, dv = coszdz, and n = 2, 4, and 6 respectively. +6. By (4) Here v Tsosreamive Examvin 2. Find fcostedz. Solution. fcostzde = f costs cosade =fa- sin®z) cosnde. = fcosade — f'sin?e cosade sinter =sing “740, sing ~ = + EXAMPLES rh fointede = feostz—cosz+C. 5. for 66 cos 6048 = o sintx — 2 faint cosede 7 te 6. f cost 26 sin2 600 in? $ 8. ffsinz cosede = SF 4.0. 7. SOP BEE = cso — hoscte +0 in? 4. foosasinada =— 8 4.0, 8, (made _ seca + cosa + C. 3 costae 9. foosta sin?ade = Foos'a + }oosta +C. s 10. fsintede = coss + Feast — = +0. fe ante U1. f costede = in — Fsinte +2 40. 12. [oink cost ode = rpsin® o — asin’ 9 +0. # This was integrated by the power formula taking n=1, v=sinz, dv=cos dz. To illus- trate how an answer may take on different forms when more than one method of integration is possible, let us take n=1, v= cos z, dv=—sin zdz, and again integrate by the power formula. Then Si sin coszdz= a result which differs from the first one in the arbitrary constant only. For, 2. ~ sin? int, in? = 88 c7 FMB yr 1 PE ore BPEL, Hence, comparing the two answers, ('=-4+C". 300 INTEGRAL CALCULUS 13. faint a coss 6d0 = sink 6 — yy sin + a, sin’? + C. ay == 2G (1 —Femty + omy) 4.6. = paint (a — Joints + Zoints) +0. 16. Integrate the following expressions and prove your results by differentiation: (@) f sin?20d0. ® Si ‘cost az sin azde. (ke) S ‘sin mt cos midt. (vy f cost Sao, ) f sint 2 cos22 ap, () f sins neat () (fein 22 cos 20dz. (hy feost3a2sin3 2de, (mn) (fointe coszae. @ f ‘sin ¢ cos? edt, w i ‘sin’bs cos dsds, (a) S ‘costy sin ydy, of cos” sin ae. of cost faint Say, (0) ff cost (a + bia Example I. 7b find if tan*ede, or f cottede. ‘These forms can be readily integrated, when n is an integer, on somewhat the same plan as the previous examples. Iuustmative Examrin 1, Find f tantedz, Solution. if tantade = if tan®2 (see?a — 1) dx = 7 ‘tan? sectnde — 7 tantade = i (tan 2)2d (tan 2) — f' (seo? — 1) dz 3 = St _tanete4c. by 28, p. 2 Example III. 7 find if sectade, or f ese" nid, These can be easily integrated when 7 is a positive even integer, as follows: Inwsrnative Examrre 2, Find f'sectedz, Solution. f ‘sec’ 2dz = if (tanta + 1)?secade = f(ana)'secteds + 2 f (tana)tsectade + fsecteds ‘ 2 = Oey ME tone $C. 6 3 ‘When n is an odd positive integer greater than unity, the best plan is to reduce to sine or cosine and then use reduction formulas on p. 808, by 28, p.2 INTEGRATION 301 Example IV. 7 find [ tan"x secede, or if cot™x esctede, When n is a positive even integer we proceed as in Example IIT. Inusrranive Examrie 8, Find ftan'esectede, Solution. fi tan®z sectadz = if tanS2(tantz + 1) sectade by 28, p. 2 = f (anaysecteds + ftan’zseoteds tan®x | tan’x c. (4) tot By (4) Here v = tanz, dv = sec*dr, ete, When m is odd we may proceed as in the following example. Tnusreanive Examrir 4. Find f tan'seotadz, Solution, ——_ftan?zsootedr = tanta sects seo tanadz = f (seote — 1)? sec®e seca tan ade by 28, p. 2 = f (scot — 2secte + sec?) seca tan zde sect _ 2socks | secka bi 5 3 Here v = ‘secs, dv = sec tanzdz, ete. +0. By (4) EXAMPLES tants cotte Peds = Sede =— 22 _ tog si 1. ftanteds = Ft log cosa + 0. 3. foot ade = — O° — log sine +0 2, frant2ede = 02% _ 240, 4. froottade =—cotn—2 +0. 42 az =— cots® = 5. foot 5 ax cot? § + Scot + a4 C. 6. foot ada =— foot! a+ 4oot?a + logsina + C. 6Y ay = tant Y — Ye a y 7. fitan Gay = tant {—2 tant T + dog sect + C. tan?x | Stand 7 5 9. fesetadz =— cotx — j cots —~ pootha + C. + tants + tanz +6, 8. fseotadz = tant 7 10. f' ‘tant sect gdp = + i. fan @sect ag = }secl @ — Lseck + C. cote cottr Sz esctade = _ . 12, f cotta eset ade = — te 5 # esectnds = 2tante , 2tanke 13, ftankesoctede 3 tty te 302 INTEGRAL CALCULUS “ 2 14. ftansysech yay = aac y (SEY eect te Dec, it 7 sect ada cot? 1b, f SE tana — cba, STGAS = tan 2e0t a F 40, 16. fanz + tantz)dz = ptantz + 0. ag if (tant + cot t)8dt = 4(tan% — cot) + log tant + 0. 48, Integrate the following expressions and prove your results Ly differentiation. (@) firantzeat, @ i sec? tan?6d8. (m) SS ian a t : cate (by f cot? 5 at. (ny Sf ‘esotp cot2p dg. (n) SS ae (0) fant araz. of, “a (0) fseoteds. @ fi cot? de. G) ftantesect eat, () fesot edz, of, oe (0) ff cottyesetvay. (a) ftans sect ede. 340 ofa, ofl fase Example V. To find if sine cos ade by means of multiple angles. When either m or n is a positive odd integer, the shortest method is that shown in Example I, p. 298. When mand n are both positive even integers, the given differential expression may be transformed by suitable trigonometric substitutions into an expression involving sines and cosines of multiple angles, and then integrated. For this purpose we employ the following formulas: sin wcos u = }sin 24, 36, p. 2 sin’u = $— feos 2u, 38, p. 2 cos'u = $+ feos 2m 39, p. 2 Inusreative: Examvin 1, Find f costedz. Solution. feostads =fa + $008 22) dz 38, p.2 1 1 geil. =e ! feos 2edz = 4 )sin 2240. gf et 5 f cos 2ada = 5 + 75in 22 +0, Innusrnanive Examrin 2. Find f'sints cos? dz, Solution. f'sin®e cost ade = + f'sin? 22de 36, p. 2 =f (— 4008 42) dz 38, p.2 at) sinteec. a 32° INTEGRATION 808 Innusrmanive Exawete 8. Find sina costade, Solution. fi sintz costade = f (sinz cosa)*sin* edz = [isin 22 — $c08 22) de 86, p.2; 38, p.2 =4 [sin 2edn— 4 fsin? 22 008 22de =4fG- 40s 42) dz — + sin? 2a cos edz a sind sin? 2a == _ Sete _ tt ag, “st Example VI. 0 find if sin mee cos neds, f ae nde oe f oe cos nada, when m + n. By 41, p. 2, sin mz cos na = } ain (m+n) 2+ ain (m—n) 2 wf sin ma cos node = 5 [sin (m + nade + [sin (me — n)ede = Les(mtn)z _ cos(m—njx , o 2(m +n) 2(m—n) Similarly, we find : . __sin(m+n)e , sin(m—n)x Lf sit mesin nade = ian) + B(m—ny °° — sin(mtn)z , sin(m—n)e _ffcosme cos med 2m +n) tae +c. 4, fsintoas = 3 (60 —4sin20 4 S522 4 Saindz)+ 0 1 nog _sint2e 8. 5. feostaas = 2 (52-4 4sinae— SE 2 4 Fsindz) +0, ° 6. Uf sat cost ada = — S02 g-ste +0. 1 7. fsintt costedt = ag (9¢— sine + ee. 28 - 128 (62+ Bairae— sine 2882) 40, 8. f costa sin? edz 304 INTEGRAL CALCULUS ‘ __cos8y _ cos2y 9. fi cos By sin Bydy =— SOU SREY 40, 10. f'sin 52sin 62dz =— oe ec 22 2 sinl1s , sin3s 11. f'cos 48 cos 7 sds c. feos 4s 0087 5 = ot 169. Integration of expressions containing Va" — x* or Vx" a? by a trigonometric substitution. In many cases the shortest method of integrating such expressions is to change the variable as follows: When Vai — x occurs, let «=a sinz. When Va? + x* oceurs, let x =a tanz. When Vz? — @ occurs, let x= a secz.* ae . a? — 22h Solution. Let 2 = asinz; then dz = acoszdz, and de a cos zdz a cosziz a Seeawal asin? z)h ~Sseove laa hme e+e Inwusrrative Examrce 1. Find f- +0. ~ 2 Since sins =, draw a right triangle with 2 as the opposite . leg to the acute angle 2, and a as the Doypotenuse,, ‘Then Le the adjacent leg will be Va?—z? and tanz . aat Inusrrative Examroe 2. Find ; ave+1 Solution, Let x = tanzt; then de = sec? 2dz, and f dz, ic seo? ede sect edz aVe+1 % tang Vian’ ae eee = feta =f = fesezds wane” = J sing Veei-1 = log(esez — cot 2) = og SA +0. ao y sincotansasciteetandeicre YEH, | 6S fe z = # We may also use the substitutions x= a cosz, z= acotz, and z= aese 2 respectively. 7 In this example a= 1. a. fee 2. [Vea Bae = 3. sey dz = log («£ + Va? + a?) — INTEGRATION EXAMPLES de = VP=@— a are sec? + 0. 2 few + Saresine +0. VEre z <(2 Laresing —2YU “J Vin=g 2 (22-1) VF rT 6 fs waar. evita ae Ve—@ (e—a28 . = C, 1 [ae et az. 7 Sam 2, feetnae, +2 (ax + dae 3. | ———. Same 4. fom Sa9, (40 —1)dx 5. SVS dz OV aece. Bt "Sea dt. 8 fre 9. fies. 10, (fans2 —1)%ae. 11, f tants seotaas. aa 12. fsint2 ae. sints —_ cost @ MISCELLANEOUS EXAMPLES 4, f——"_. Saperse 8.cos 6d 16. Soa taing® fe [eee ds ae Sas 3e— 18. Sos ae. de 19. f= _. Saga CO al. i sin 26" dt ada Saar 305 de Soap 1+ sec?@ T+ tand*” dt 29, f—e_. Sams 30. f (a— 82%)" 22dz, 2a%de 31. . Sani (@— 28 a2. f° Cee 33. f tested 34, fere=tatae, as, (22=2 | 37. f costa sin azde. 38. S cot! aydy. 39. fsin?6zde, 306 INTEGRAL CALCULUS. 40. The following functions have been obtained by differentiating certain func- tions, Find the functions and verify your results by differentiation. (a) 62 + singe. Solution, In this example (52° + sin 2) dz is the differential expression to be inte- grated. ‘Thus J (60? + sin 22) de Set teoste +0. Ans. 4 Verification. Z(H jeonze + 0) = 52° + sin2a, (b) 52° — 62. (©) 227-824, @ cos?az + sin”. a Va+ia | () Va + br. : ae A ( ° Tea e bata 5-68 pe 2S ae »~ ©) S422 () (@—e ay. ! 8420 ae ow) 242d. Oa +1" (0) sin mz cos mz. 1 (p) cos?4 pr. @) 1-82 . wve—1 i © fT re (y) 2@viee CHAPTER XXIII CONSTANT OF INTEGRATION 170. Determination of the constant of integration by means of initial conditions. As was pointed out on p. 281, the constant of integration may be found in any given case when we know the value of the integral for some value of the variable. In fact, it is necessary, in order to be able to determine the constant of integration, to have some data given in addition to the differential expression to be integrated. Let us illustrate this by means of an example. Intusprative Exampre 1, Find a function whose first derivative is 32?— 2a + 5, and which shall have the value 12 when a = 1. Solution, (8x2— 22 4 5)dz is the differential expression to be integrated. ‘Thus (j[@e—224 \de=#— 2+ 5240, where C is the constant of integration. From the conditions of our problem this result must equal 12 when « = 1; that is, 1251-14546, or C=7. Hence 2*— 2° + 52 + 7 is the required function. 171. Geometrical signification of the constant of integration. We shall illustrate this by means of examples. Inuusrrarive Exampre 1. Determine the equation of the curve at every point of which the tangent has the slope 22. Solution, Since the slope of the tangent to a curve at any point is #, we have, by hypothesis, W Los, or, dy = 2202. Integrating, y=2 fda, or, (A) yaar, where C is the constant of integration, Now if we give to C a series of values, say 6, 0, — 8, (A) yields the equations y=O46, yoa, yoat—3, whose loci are parabolas with axes coinciding with the axis of y and having 6, 0, ~ 8 respectively as intercepts on the axis of ¥. 807 308 INTEGRAL CALCULUS All of the parabolas (A) (there are an infinite number of them) have the same value of $4; that is, they have the same direction (or slope) for the same value of 2. It will also be noticed that the difference in the lengths of their ordinates remains the same for all values of «. Hence all the parabolas can be obtained by moving any one of them vertically up or down, the value of C in this case not affecting the slope of the curve, Ié in the above example we impose the additional condition that the curve shall pass through the point (1, 4), then the codrdinates of this point must satisfy (A), giving 4=140, or 0=3, Hence the particular curve required is the parabola y = P43, Inwusrrative Exampre 2. Determine the equation of a curve such that the slope of the tangent to the curve at any point is the negative ratio of the abscissa to the ordinate. y| Solution, The condition of the problem is expressed by the equation ay de or, separating the variables, ” Integrating, or, e+ y=20, ‘This we see represents a series of concentric circles with their centers at the origin, If, in addition, we impose the condition that the curve must pass through the point (8, 4), then 94+16=20, Hence the particular curve required is the circle 2? + y? = 25, ‘The orthogonal trajectories of a system of curves are another sys- tem of curves each of which cuts all the curves of the first system at right angles Hence the slope of the tangent to a curve of the new system at a point will be the negative reciprocal of the slope of the tangent to that curve of the given system which passes through that point. Let us illustrate by an example. Inwwsrrative Exampie 3. Find the equation of the orthogonal trajectories of the system of circles in Illustrative Example 2. Solution, For the orthogonal system we will then have dy_y aa’ or, separating the variables, dy _ dz ye Integrating, logy = logs + loge = logex, or, yacn. Hence the orthogonal trajectories of the system of circles 27+ y= C is the system of straight lines which pass through the origin, as shown in the figure. CONSTANT OF INTEGRATION 309 172. Physical signification of the constant of integration. The fol- lowing examples will illustrate what is meant. Intustrative Exanrie 1. Find the laws governing the motion of a point which moves in a straight line with constant acceleration. Solution. Since the acceleration [-¢ from (14), p. 02 is constant, say J, we have a an or, dp = filt. Integrating, (4) f+. To determine C, suppose that the initial velocity be-v,; that is, let + v=) when t ‘These values substituted in (A) give %=04C, or, C= 0. Hence (A) becomes (B) vafltr, Since v= s [(®), p. 90], we get from (B) ds Fatt ty or, ds = flut + vylt, Integrating, (©) = TE + ot + C. ‘To determine C, suppose that the initial space (= distance) be s, ; that is, let s=8) when t=0. ‘These values substituted in (C) give %=0F04C, or, C=%, Hence (C) becomes (D) ASP 9b + 8. By substituting the values f=, t= 0, s,=0, s=h in (B) and (D), we get the laws of motion of a body falling from rest in a vacuum, namely, (Ba) v=gl, and (Da) tot Eliminating t between (Ba) and (Da) gives v= V2gh. Ivuusrrative Exampre 2, Discuss the motion of a projectile having an initial velocity v) inclined at an angle a with the horizontal, the resistance of the air being neglected. Solution, Assume the XY-plane as the plane of mo- tion, OX as horizontal, and O¥ as vertical, and let the projectile be thrown from the origin, Suppose the projectile to be acted upon by gravity alone. Then the acceleration in the horizontal diree- tion will be zero and in the vertical direction — g. Hence from (15), p. 93, deg at =0, ana 310 INTEGRAL CALCULUS Integrating, %=Cy and %=—gtt Cy, But ¥, 008 @ = initial velocity in the horizontal direction, and vpsin a = initial velocity in the vertical direction. Hence OC, = »,cos a, and C, = vysin a, giving () By = vy C0S a, and vy =— gt + vysina. But from (10) and (11), p. 92, v = e, and v= % ; therefore (E) gives a = pcos, and Y — gt + msina, dt at or, dz = vy cos adt, and dy =— gidt + vpsin adt, Integrating, we get (F) THnHcosa-t+Cy, and y=— jy + vysina-tt Cy ‘To determine C, and C,, we observe that when t=0, e=0 and y=0, Substituting these values in (F) gives O,=0, and O, . Hence (@) 2 =v, cosa-t, and (H) y=— tot + vysin at, Eliminating ¢ between (@) and (HZ), we obtain = ga? w Y= aan a a! which is the equation of the trajectory, and shows that the projectile will move in a parabola, EXAMPLES 1. ‘The following expressions have been obtained by differentiating certain functions Find the function in each case for the given values of the variable and the function: Derivative of Value of Corresponding function variable value of function ' Answers (a) 2-8. a 9. abet 8. 2 bat ()) 84a—5a%. 6 20. ao4 92452. A Ye (©) e—hy. 2 0. EE you. (a) sina + cosa. e 2. sina — cosa +1. 1 0. log (2¢— 2). (f) sec? @ + tand. 0. tand + log sec 8 + 5. 1 £ 7 ® sty a aretand +. (h) ba? + az + 4, b 1 i) Vi+ — a @) ai : {j) cots — csc? g. a (&) Beet. 0o. CONSTANT OF INTEGRATION 311 2. Find the equation of the system of curves such that the slope of the tangent at any point is: : fa) a. Ans, Parabolas, y (b) 22-2. Parabolas, y = 2?— 24 4. 2 @i. Parabolas, “= 240. y 2 a @) = Semicubical parabolas, e = = +0. v : a ge © - Semicubical parabolas, [ = [ +0. () 82%, Cubical parabolas, y = 2° + C. (@) a +52. Cubical parabolas, y ea + bay ©. i i ee Oe Cubical parabolas, = 2+ 0. @ 2. Equilateral hyperbolas, y?— 2? =C. y w- _ Equilateral hyperbolas, zy = C. oo Hyperbolas, ay? — 6%? = C. ay @ - ee Ellipses, iy? + az? = C. (m) ay. (a) y. (0) m. Straight lines, y = ma + C. oy pe. Circles, 2? + 724+ 22—2y40=0. —- . 3. Find the equations of those curves of the systems found in Ex, 2 (a), (c), (a), (i), (), (m), which pass through the point (2, — 1). a Ans. (a) a?—2y—6=0; (m) y= 4, Find the equations of those curves of the systems found in Ex. 2 (b), (¢), (g), (b), (0), (p), Which pass through the origin. Ans. (b) y= a%— 22; (0) y= mea; ete. e® 5 etc. 5. Find the equations of the orthogonal trajectories of the following systems of curves found in Ex. 2: (a) y="4 C, Ex. 2 (a). Ans. y =— logz + C. (0) F =a + 0, Bx. 2 (0). logy =— 2 +0. woe at ©) =F +0, Bx. 2 (a). logy = 7+ 0. @) Y—2?=C, Ex. 2 (i). ay=C. (©) ty =C, Ex. 2 (j). . Pr-@=d, (f) y= cer, Ex. 2 (n). ¥a-ese. (g) y= ma + C, Bx. 2 (0). my +2=C. (h) 24+ y+ 20-27 +0 =0, Ex.2(p). y-1scet)). i | 812 INTEGRAL CALCULUS 6. Find the equation of the curve whose subnormal is constant and equal to 2a, 4 Ans. y?=4az + C, ; Hur. From (4), p.77, subnormal = y : Mies aad 7. Find the curve whose subtangent is constant and equal to a (see (3), p. 77). ‘Ans, alogy =2+C. 8, Find the citrve whose subnormal equals the abscissa of the point of contact. Ans, y?— 2? = 20, an equilateral hyperbola. 9. Find the curve whose normal is constant (= R), assuming that y = R when 2=0. Ans. 2? + y? = RB, a circle, ‘dy\2 = Hinr. From (6), p. 77, length of normal= 1 1+(/ WY, ot dz =4(R2— y)~bydy, 10. Find the curve whose subtangent equals three times the abscissa of the point of contact. Ans, 2 = cy*. 11. Show that the curve whose polar subtangent (see (7), p. 86) is constant is the reciprocal spiral. 12. Show that the curve whose polar subnormal (see (8), p. 86) is constant is the spiral of Archimedes. 13. Find the curve in which the polar subnormal is proportional to the length of the radius vector, Ans. p = ce, 14, Find the curve in which the polar subnormal is proportional to the sine of the vectorial angle. Ans. p=¢—acosd. 15, Find the curve in which the polar subtangent is proportional to the length of the radius veetor. Ans. p = ce, 16. Determine the eurve in which the polar subtangent and the polar subnormal are in a constant ratio, Ans. p = ces, 17. Find the equation of the curve in which the angle between the radius vector and the tangent is one half the vectorial angle. Ans. p = ¢(1— cos). 18, Determine the curves in which the subtangent is n times the subnormal ; and find the particular curve which passes through (2, 8). . Ans. Vay =2 +0; Valy—8)= 2-2 19, Determine the curves in which the length of the subnormal is proportional to the square of the ordinate, ‘Ans. y = cel, 20. Find the curves in which the angle between the radius vector and the tangent at any point is m times the vectorial angle. Ans. p* =csinnd. Assuming that » acceleration is : v when ¢ = 0, find the relation between v and ¢, knowing that the 21, Zero. Ans. 0 =v, 22. Constant vant lt. 2 23. at bt. vant at > Assuming that s = 0 when t = 0, find the relation between s and ¢, knowing that the velocity is: 24. Constant (= »,). 25, m+ nt. 26.34 2t—82 CONSTANT OF INTEGRATION 313 21. The velocity of a body starting from rest is 5? feet per second after ¢ seconds. (a) How far will it be from the point of starting in 8 seconds? (b) In what time will it pass over a distance of 860 feet measured from the starting point ? Ans. (a) 45 ft.; (b) 6 seconds, 28, Assuming that s = 2 when ¢ = 1, find the relation between s and ¢, knowing that the velocity is: (a) & Ans. $= 81-1. (b) 2t—38. sat*—8tt4. 2 - Be 2 5 (:) #4 2t— sal 4e—t4e. i 3 8 (@) >. s=logt +2. t ©) 40-4. sat 404s. oF sa pede, - =-t i 29. Assuming that »=3 when ¢= 2, find the relation between v and t, knowing that the acceleration is: (a) 2. Ans. 2t-1 (b) 8841. v=O 41-7. (c) &— 24. f-848. (@ tae Doig ea t oe 30. A train starting from a station has, after ¢ hours, a speed of &— 21 + 804 miles per hour. Find (a) its distance from the station; (b) during what interval the train was moving backwards; (c) when the train repassed the station; (d) the dis- tance the train had traveled when it passed the station the last time. Ans. (a) $'— 1 + 402 miles; (b) from 5th to 16th hour; (0) in 8 and 20 hours; (@) 4658} miles. 31. A body starts from the origin and in ¢ seconds its velocity in the X direction is 12¢ and in the Y direction 4t?—9, Find (a) the distances traversed parallel to each axis; (b) the equation of the path. / & Ans. (a) =60, y= 58-91; (Hy =(fe- 0) i 32, The equation giving the strength of the current i for the time ¢ after the source of E.M.E. is removed is (It and L being constants) ria—1, a cn Find i, assuming that I= current when ¢ = 0. Ans, izle E 33, Find the current of dischange é from a condenser of capacity C in a circuit of resistance R, assuming the initial current to be I), having given the relation (C and R being constants) aa a Ans. i= Te0R, 34. If a particle moves so that its velocities parallel to the axes of X and ¥ are ky and kz respectively, prove that its path is an equilateral hyperbola. 35. A body starts from the origin of codrdinates, and in ¢ seconds its velocity parallel to the axis of X is 64, and its velocity parallel to the axis of ¥ is 3—8. Find (a) the distance traversed parallel to each axis in ¢ seconds; (b) the equation of the path. Ans, (a) C=88, y=O—8t; (b) ZY? =2(e— 9) CHAPTER XXIV THE DEFINITE INTEGRAL 173, Differential of an area. Consider the continuous function $(2), and let. GaGa) be the equation of the curve AB. Let CD be a fixed and MP a variable ordinate, and let w be the measure of the area CMPD.* When 2 takes on a sufficiently small increment Az, u takes on an increment Aw (=area MN@QP). Completing the rectangles MNRP and MNQS, we see that area MNRP < area MNQP < area MNQS, or, MP-Ar < Aw< NQ- Ac; and, dividing by Az, up <3 < not Now let Az approach zero as a limit; then since MP remains fixed and VQ approaches MP as a limit (since y is a continuous function of z), we get au a UHM), ox, using differentials, gy, yp. Theorem. The differential of the area bounded by any curve, the axis of X, and two ordinates is equal to the product of the ordinate ter- minating the area and the differential of the corresponding abscissa. 174. The definite integral. It follows from the theorem in the last section that if AB is the locus of y=O@) then du = ydz, or (4) du= (x) de, * We may suppose this area to be generated by a variable ordinate starting out from CD. and snoving to tl hence u will be a function of x w shes when x= a. {In this figuro IP is less than NQ; if MP happeus to be greater than VQ, simply reverse the inequality signs. 314 THE DEFINITE INTEGRAL 315 where du is the differential of the area between the curve, the axis of 2, and any two ordinates. Integrating (4), we get wu f (a) de. Since {(z) dz exists (it is here repre: “' sented geometrically as an area), denote it by F@) +e (By Hwaf@te. We may determine ©, as in Chapter XXIT, if we know the value of w for some value of x. If we agree to reckon the area from the axis of y, ie. when (oy g=a, w=area OCDG, and when v=b, u=area OFFG, etc., it follows that if ) 2=0, thenu=0. Substituting (D) in (B), we get u=fO)+C, or C=—f(0). Hence from (B) we obtain ) u=f(@)-fO), giving the area from the axis of y to any ordinate (as MP). To find the area between the ordinates CD and EF, substitute the values (C) in (22), giving (P) area OCDG =f(a) —F (0), (@) area OLFG =f (b) —f (0). Subtracting (7) from (@), (® area CEFD = f(b)—f(a).* Theorem. The difference of the values of [ ydx for x=a and c=b gives the area bounded by the curve whose ordinate is y, the axis of X, and the ordinates corresponding to x=a and x=b. ‘This difference is represented by the symbol t (ea) fore on foerae * The student should observe that under the present hypothesis f(z) will be a single. valued function which changes continously from f(a) to f(b) as x changes from a to b. + This notation is due to Joseph Fourier (1768-1830). 316 INTEGRAL CALCULUS and is read “the integral from a to 6 of ydz.” The operation is called integration between limits, a being the lower and b the upper limit.* Since (Z) always has a definite value, it is called a definite integral. For, if fe@u=seo+e, then f (a) de = [ ors ef. =O +E] -LFO+e} on [e@e-rO-T, the constant of integration having disappeared. We may accordingly define the symbol ewe or fore as the numerical measure of the area bounded by the curve y=$(a),! the avis of X, and the ordinates of the curve at r=a, x This definition presupposes that these lines bound an area, i.e. the curve does not rise or fall to infinity, and both a and b are finite. ‘We have shown that the numerical value of the definite integral is always f(b)—f(a), but we shall see in Illustrative Example 2, p. 824, that f(6)—f(a) may be a number when the definite integral has no meaning. 175. Calculation of a definite integral. The process may be sum- marized as follows: Finsr Srev. Find the indefinite integral of the given differential ex- pression. Srconp Srer. Substitute in this indefinite integral first the upper limit and then the lower limit for the variable, and subtract the last result from the first. It is not necessary to bring in the constant of integration, since it always disappears in subtracting. The word Zimit in this connection means merely the value of the variable at one end of its range (end value), and should not be confused with the meaning of the word in the Theory of Limits. + @) ¢ is continuous and single-valued throughout the interval [a, 0]. THE DEFINITE INTEGRAL 317 4 Innusrearive Examrce 1. Find f'2%de, 1 4 a]! 64 1 i edz = |™]*= 84-1 oar. ans. Solution. fee : 21, Aas. Inusrrative Exampre 2. Find Solution. fisinzae = [- cosa |= [ Tnwsreative Bxanrue 8, Find f° 0 2. Ans. & a+ at Solution. f = li are tan =]i= J ave tant — are tano eye oa a a a 7 7 =— —- Ans. 7 a EXAMPLES “ 1, 6atde = 38. 7 A 13. sect @d0 = 4. 2. fi @e-2)ae= ua [ae =a 0 ve 15. f GVi- seers 2vi—5 = vrde 16. f’ @_= ef, Vaoe 2 1 5. f@— 22+ 2)(@—Ndo=— 4 : igen So Feagn 1 _ . v8-1. “Lz on ee 8 Ge = Se agtde 8 = = 5 — logs. 7 frei . * tap = 19. 2af- (2 + 2cos8)hd9=8a. 20, fPsintarcostada = 2. 0 ai. fitanada=0. + + | 10. : sin gdp = 23, [2 SSE 7. 12. fsin gdp i. Traini = 2 318 INTEGRAL CALCULUS 176. Calculation of areas. On p. 816 it was shown that the area between a curve, the axis of X, and the ordinates «=a and «= is given by the formula Area i yd, a where the value of y in terms of z is substituted from the equation of the given curve. Ittusrrative Examrre 1. Find the area bounded by the parabola y = 2%, the axis of X, and the ordinates 2 = 2 and 2=4, Solution, Substituting in the formula 7 sya Aven ABDC = {adr = [FI 2 EXAMPLES 1, Find the area bounded by the parabola y = 2%, the axis of X, and the ordinate a=8. Ans, 9. 2, Find the area above the axis of X, under the parabola y? = 42, and included between the ordinates z= 4 and ¢ =9. Ans, 25}. 3, Find the area bounded by the equilateral hyperbola zy = a2, the axis of X, and the ordinates ¢ = a and 2 = 2a. Ans. a? log 2. 4, Find the area between the parabola y = 4 — a and the axis of X. Ans. 10}. 5. Find the area intercepted between the eotrdinate axes and the parabola 1 2 aby yt=al. Ans. 5. 6, Find the area by integration of the triangle bounded by the line y = 52, the axis of X, and the ordinate « = 2. Verity your result by finding the area as one half the product of the base and altitude. 7. Find the area by integration of the triangle bounded by the line y = 2« + 6, the axis of X, and the ordinate x= 4, Verify your result as in the last example. 8, Find the area by integration of the trapezoid bounded by the line — y + 4 = 0, the axis of X, and the ordinates s =— 2 anda = 4, Verify your result by finding the area as one half the produet of the sum of the parallel sides and the altitude. 9. Find the area by integration of the trapezoid bounded by the line + 2y — 6 =0, the axis of X,and the ordinates ¢ = 0 and 2=8, Verify your result as in the last example. 10. Find the area by integration of the rectangle bounded by the line y = 5, the axis of X, and the ordinates x = 2 and «= 6, Verify your result geometrically. 11. Find by integration the area bounded by the lines ¢ = 0,¢=9,y=0,y =7. Verify your result geometrically. 12, Find the area bounded by the semicubical parabola y# = 2%, the axis of X, and the line « = 4. Ans. §-Y1024. THE DEFINITE INTEGRAL 319 18. Find the area bounded by the cubical parabola y = 2%, the axis of X, and the ordinate ¢ = 4. Ans. 64. 14, Find in each of the following cases the area bounded by the given curve, the of X, and the given ordinates : @) y=9-2 Ans. 36, = = b) ‘ log V65. O)y=z +a log (©) y=sinz. a @ y=x + 82% 422. 54, P+e+1. 8. at + 403 + 2a? + 3, 2813. ae Blog”. a CGS 4 (m) zy =12. sland. 15, Find the area included between the parabolasy®=4z and a2=4y. Ans. 5}. 16. Find the total area included between the cubical parabola y = 2 and the line y=2z. Ans, 2. 17. Prove that the area bounded by a parabola and one of its double ordinates equals two thirds of the circumscribing rectangle having the double ordinate as one side. 18. Find the area included between the parabolas y? = 4 4 « and y? = 19. Find the area between the curve y = a and the line y = Ans. log4—}. 20. Find by integration the area of the triangle bounded by the lines a+3y—8=0, 52-y-16=0, x—y+1=0. Ans. 8. 177. Geometrical representation of an integral. In the last section we represented the definite integral as an area. This does not neces- sarily mean that every integral ¢s an area, for the physical interpre- tation of the result depends on the nature of the quantities represented by the abscissa and the ordinate. Thus, if 2 and y are considered as simply the coérdinates of a point and nothing more, then the integral is indeed an area. But suppose the ordinate represents the speed of a moving point, and the corresponding abscissa the time at which the point has that speed; then the graph is the speed curve of the motion, and the area under it and between any two ordinates will represent the distance passed through in the corresponding interval of time. That is, the waber which denotes the area equals the nwmber which denotes the distance (or value of the integral). 320 INTEGRAL CALCULUS Similarly, a definite integral standing for volume, surface, mass, force, ete., may be represented geometrically by an area, On p. 366 the algebraic sign of an area is interpreted. 178. Mean value of ¢(x). This is defined as follows: ; [sea b—a Mean value of go) from x=atox=b Since from the figure [e@u = area APQB, this definition means that if we construct on the base AB(=—a) a rectangle (as AZAfB) whose area equals the area of APQB, then area ALMB AB.CR : ———— = —— = altitude CR. boa AB iocee 179. Interchange of limits. sine [ 6@ae=l-~SO and fs@ae-F@-10=-FO-FO} . we have i eyar=— ["pcapar. ; , Theorem. Interchanging the limits is equivalent to changing the sign of the definite integral. 180. Decomposition of the interval of integration of the definite integral. . Since f $(a) dx =f(@,) —f(a), : and ff $@ae=FO-F@), we get, by addition, = : [s@aet [ @ae-KO-10. Bf 6@ar=fO-S: therefore, by comparing the last two expressions, we obtain [sea [Pr@at [sear mean value = THE DEFINITE INTEGRAL 321 Interpreting this theorem geometrically, as in $174, p. 815, we see that the integral on the left-hand side represents the whole area CEFD, the first integral on the right- hand side the area CMPD, and the second integral on the right-hand side the area MEFP. The truth of the theorem is there- fore obvious. Even if z, does not lie in the interval between a and 2, the truth of the theorem is apparent when the sign as well as the magnitude of the areas is taken into account. Evidently the definite integral may be decom- posed into any number of separate definite integrals in this way. 181. The definite integral a function of its limits. : From if $(2) de =f () f(a) we see that the definite integral is a function of its limits, Thus : if $(2)dz has precisely the same value as f b(a) de. Theorem. A definite integral is a function of its limits. 182, Infinite limits. So far the limits of the integral have been assumed as finite. Even in elementary work, however, it is some- times desirable to remove this restriction and to consider integrals with infinite limits. This is possible in certain cases by making use of the following definitions. When the upper limit is infinite, t limit (° [soe fo @ar, and when the lower limit is infinite, fe@ar= ae fo@ax provided the limits exist. Intusrrarive Exampre 1, Find . de limit ede limit mp sition, (OFS Sot e[~ zh = limit [1 JL = [-ge |= Ans. 822 INTEGRAL CALCULUS + Bat Tuvvsteanive Exames 2, Find ('** 2048 0 w+ da! +2 Bakde limit 7? 8a%dx Solution. a —— i P+ ie todo + 4a it [gatare tan] = lint Paataretan 2] = 40 Let us interpret this result geometrically, ‘The graph of our function is the witch, the locus of /// 2 =4u@aretan 2 Now as the ordinate Qb moves indefinitely to the right, 4@aretan 2 2a is always finite, and ag [s a@are tan a] = 2a", © 2a. which is also finite. In such cases we call the result the area bounded by the curve, the ordinate OP, and OX, although strictly speaking this area is not completely bounded. fede Tuusrnative Examvee 3, Find f°. : tedz limit dz _ Solution. f Fave tod, gba 4 olB))- ‘The limit of log as b increases without limit does not exist ; hence the integral has in this case no meaning. 183. When y = (x) is discontinuous. Let us now consider cases when the function to be integrated is discontinuous for isolated values of the variable lying within the limits of integration. Consider first the case where the function to be integrated is con. tinuous for all values of z between the limits a and b except e=a. If a sin (: . | are sin = c= gy tide Solution, Here 2 becomes infinite for = 0. ‘Therefore, by (A), Td limit ¢ _ 1) In this case there is no limit and therefore the integral does not exist. If elies between a and &, and $ (2) is continuous except at «=e, then, cand ’ being positive numbers, the integral between a and b is defined by CO) fe@aras (sears ls (peas provided each separate limit is a definite quantity. Qadr ° eae Solution. Here the function to be integrated becomes infinite for z = a, i.e, for a value of & between the limits of integration 0 and 8a, Hence the above definition {C) must be employed. ‘Thus i Qudz _ limit 2ede limit cade ° Gant §= 0 Geant = eter ant — limit 4] oo s, limit ae ioe enl referer], 48at] + ims vee _sVayep—a] Intusrrative Exampte 1, Find = Sat + Gat = oak. Ans. To interpret this geometrically, let us plot the graph, i.e. the locus, of Qe at and note that # = a is an asymptote. area OPE = f" oa | emai =8VG@—o)— @ + 8at. Now as PE moves to the right toward the asymptote, i.e, as approaches zero, 3Va— oP — a + Bat is always finite, and, Py Jinj[s Va@=F— @ + sat] = sal, 824 INTEGRAL CALCULUS which is also finite, As in Illustrative Example 1, p. 828, 8a is called the area bounded by OP, the asymptote, and OX. Similarly, area E’QRG =f" 20 _ 3 ¥eS_3Vapepoe ate Ga ait is always finite as QE’ moves to the left toward the asymptote, and as ¢” approaches zero, the result 6aé is also finite. Hence 6a* is called the area between QR, the asymptote, the ordinate z= 8a, and OX. Adding these results,Swe get 9at, which is then called the area to the right of OY between the curve, the ordinate = 84, and OX, az, (e— a)? Solution, This function also becomes infinite between the limits of integration, Hence, by (C), 20 20 Inwusrrarive Exawrin 2, Find aa de + Tinie ¢- ye om (- te ‘): In this case the limits do not exist and the inte- gral has no meaning. If we plot the graph of this function and note the limits, the condition of things appears very much the same as in the last example. It turns out, however, that the shaded portion cannot be properly spoken of as an area, and the integral sign has no meaning in this case. ‘That it is important to note whether or not the given function becomes infinite within the limits of integration will appear at once if we apply our integration formula without any investigation, ‘Thus 2a de [- 1 fe _2 o @—ap z—alo a’ a result which is absurd in view of the above discussions. <7] Vy py ‘ _ » EXAMPLES +2 1 1 er arde = —. 0 a fo 1 8. f Gey Gear a. fi pieme to de 20. f PHeQete “20-V@i a ade = 7. ee oF Pas = «ade _ 8 V2—4 ° at +a%)h : 12, CHAPTER XXV INTEGRATION OF RATIONAL FRACTIONS 184. Introduction. A rational fraction is a fraction the numerator and denominator of which are integral rational functions.* If the degree of the numerator is equal to or greater’ than that of the denominator, the fraction may be reduced to a mixed quantity by dividing the numerator by the denominator. For example, +32 e+ 2e4+1 The last term is a fraction reduced to its lowest terms, having the degree of the numerator less than that of the denominator. It readily appears that the other terms are at once integrable, and hence we need consider only the fraction. In order to integrate a differential expression involving such a fraction, it is often necessary to resolve it into simpler partial frac- tions, ie. to replace it by the algebraic sum of fractions of forms such that we can complete the integration. That this is always possible when the denominator can be broken up into its real prime factors is shown in Algebra.t 185. Case I. When the factors of the denominators are all of the first degree and none repeated. To each nonrepeated linear factor, such as 2—a, there corre: sponds a partial fraction of the form A za Such a partial fraction may be integrated at once as follows: fZ -4f, dx =Alog(z@—a) +0. * That is, the variable is not affected with fractional or negative exponents. +See Chap. XIX in Hawkes’s "Advanced Algebra,” Ginn and Company, Boston. 825 826 INTEGRAL CALCULUS 2+ ae Invosrrarive Examppe 1, Find dee z =a" Solution. The factors of the denominator being 2, x — 1, « + 2, we assume* Sete Ae 8 2 z@-)@+ @ tai where A, B,C are constants to be determined. Clearing (A) of fractions, we get (B) Qu 48 =Ale—1) (42) + Bet 2)2+C(@—1)2, 2Q24+8=(A+B+C)a2+(44+2B—C)z—-BA. Since this equation is an identity, we equate the coefficients of the like powers of © in the two members according to the method of Undetermined Coefiicients, and obtain three simultaneous equations A+BLC=0, A+2B-0=2, -2A=3 (@) Solving equations (C), we get A=-}, B=, C=-}. Substituting these values in (A), Qe48 3 5 1 z@—N@+2) ae ha 2048 a ze-h@rH” oJ aye a oo 1) = flog (x + 2) + loge -1t = log $2—=D" . Ans det ot ‘A shorter method of finding the values of A, B, and C from (B) is the following Let —_factora =0; then 3=— 2.4, or A=-}. Let factorz—1=0,or2=1; then 6=3B,or B= §. Let factor 2 +2=0, ors =—2; then —1=60, or C= te A useful exercise is to integrate without determining the constants A, B, C, ete. For instance, in the above example, fettO*s, (Qe+3)dx Bdx , (Cae z (@—1) (+2) ItJaes =A sage ingte ty cotag(o-t2), ‘In the process of decomposing the fractional part of the given differential neither the integral sign nor dz enters. INTEGRATION OF RATIONAL FRACTIONS 3827 EXAMPLES Q@r-1de _\@—2) @-Ne-% eras tee ade a. (+3) — or = = le — EFNet ETH 8 S45 HH @-1a _, e@+ 4 3 [ayia ss Ga at 4. Or ae — Log fe(e + 8)2(¢—2)). 8. [EAN A hog Vee= HEF +0. eo #624 log 242" +0. at =245 jae + log TERM 4c. 8 foeay- = 2a Flog Zoi + loele +2) +0. (a—dbydy ig Ya" Sarton a= gos t® (@+pajdt_ _ 1, tv) t+) 10. (erp te i poeeoe i z-v2, 1 z—v3 — a log = + log oe Va et va BV8 2 4vB 5 a Ae ertar a'S3" +0. a fe 0 14804 22? 6 4(22 + 62—8)dz _ | 200 = 10g 2%, ere 8 Sr 186. Case II. When the factors of the denominator are all of the first degree and some repeated. To every n-fold linear factor, such as (e— a)", there corresponds the » partial fractions A B @-ay t@aat The last one is integrated as in Case I. ‘The rest are all integrated by means of the he power formula, Thus za =4f@-a ee oe dno 3828 INTEGRAL CALCULUS s Invosreative Exaweie 1, Find f= 41 z@-15™ Solution, Since ¢ — 1 occurs three times as a factor, we assume Clearing of fractions, 2 41=A(e—1)? + Be + Co(e—1)+ De(e—1). a 41=(4+4 Da? 4+(—844C—-2D)224+(8A4B—-C+D)e—A. Equating the coefficients of like powers of 2, we get the simultaneous equations A+D=1, —8A40-2D=0, 844B-C4+D=0, -AslL Solving, A=—1, B=2, C=1, D=2, and e+ =@—1) — de EXAMPLES 1 z—-2 log= == +6. + log + ae, : va ode —— I +e. @+27@4+1) apt ete co he. @ + log(e—1) +0. Sa 1)de a? 1 ot = a oap aStat ti ttetetitie. 22° x (Bet2de 4248) at Be + Nae Fe48 og S _ (e+ 1) Be+IF* Seip ade 5a+12 (x + 4) __ ede BH ig (244 Se Feti @4+62+8"” GH if dy 8. + logy +1) +0. Spayraae a co) INTEGRATION OF RATIONAL FRACTIONS 829 Co t 1 tava ° farm aeayt on iva astds a erat = alog(s +a) + 2% a aprat © nz ne of (A) & = loge + my a" +0. 1 te ae =) log (e + myn +n)" — =1—log2. =a call e142) = dt 1 f° ta loge}. 1s. f tat+p 8" 4 (1482) dx 8 3 14, = logs +. hapamea 8545 187, Case II. When the denominator contains factors of the second degree but none repeated. To every nonrepeated quadratic factor, such as 27+ pa+q, there corresponds a partial fraction of the form Ar+B w+ prety This may be integrated as follows: Ap _ Ap ) Av +P P+ Bde (Ar+B) de _ ( eer oad [FES a+ pet@ [Adding and subtracting 4? in the numerator.] fe Dar {Ge “J Frpetg tJ) Papeete -4 6 (Qa + p)de + (2252) de: SABES) (te 2 (Completing the square in the denominator of the second integral.] _ : 2e+p = Slog tet a) + Fae retin FEE Since 2?+ px +q=0 has imaginary roots, we know from 8, p. 1, that 4 g—p*> 0. +6. 330 INTEGRAL CALCULUS Tuwsrnarive Examrie 1, Find ‘s Bic P+ sa Solution, Assume ee s@+h 2° wed Clearing of fractions, 4 = A (a? + 4) + 2(Be+ 0) =(4 + Bye? + Cr+44, Equating the coefficients of like powers of «, we get A+B=0, C=0, 44=4, 4 2 This gives A =1, B= eee pete a@+h « #44 very SE See = log 2 — Flog (2? + 4) + loge = log 1, © =0, so that af adr _ Ay, +d tHE) 0° e+ o+ot a Fare tan +0. Qa 82 . 3. Si epee 7 + faretan +0. ca —Haretana +6. 1-2 4 . ce 1 + fq = Bog —_* ave ta . ’ faaners a Gaiety genet 2 (= — Ode B+4 8 isotan® —— + aro ton-*_ 40. = log “Sat Gat + ae 2 2 Ve V2 = 1 de ___ jog 22 548 aretan2—4 — > arets zm rae — © @43@— 2245) et 77 yee A . af de 1, +1)" Bei 6 'P—a4l ede ‘2-4 V2 (et +3 are tan 2 ® fae 5 ioe (£4) _ ae an Ft On 4dt_ 1 @+tvasl ig tv : CHEN ETS + Vi are tan «. Sani alent wot at Lp wtyti yt aytl ett” 4 —aret Cc. nL S4o- seo tye et ms Sareea” eal 1 Qa tae43 0 @+H@+h a. +} INTEGRATION OF RATIONAL FRACTIONS 331 188. Case IV. When the denominator contains factors of the second degree some of which are repeated. To every n-fold quadratic factor, such as (2*+ px +9)", there cor- respond the » partial fractions a Ac+B Cx+D op atm (@+pe+g* (+ pzet+y"” a+ petg To derive a formula for integrating the first one we proceed as follows: (4042-4 n)ate ace 2 2 @tpato tpt po [Adding and subtracting 4? in the numerator | - fer (Gee @rpet gy) Stpry =F fort met a Carpet : Sway ‘The first one of these may be integrated by (4) p. 284; hence Ar+B | © loin” aoe +2250 2B— 4) J etm +O” 242) Let us now differentiate the function ——~——~__. Thus @ + peta 2 2 vy al 23 A 20 (2+) 7 ee Se ae\(+ pet gy | (+ pe+ gy * G+ pe+gy" p n—1(y-F oy ath | -(@n—3) 76 (4 Da (ee ee @+pet+ ge | \@tpet gr @+pet+—)” [since wt+peta=(e42)'s(«-2,ana (e+2)'=crrve40-(0-%)] 332 INTEGRAL CALCULUS Integrating both sides of (C), -(Qn- dx >| wiper oo +2@—-1) (4) faery or, solving for the last integral, bo. ate 2(n—1) (1 Fem 7 2n—3 f dx 2(n—1) (-#) (+ pet+qy* Substituting this result in the second member of (B), we get * @) fees (Ax+ Bde _ A(p’—49)+(@B— Ap) (2x40) G@ Forty" 21) 4q— P+ erty (2B -Ap)(2n— 3) @=1)G@q-~) deren or It is seen that our integral has been made to depend on the inte- gration of a rational fraction of the same type in which, however, the quadratic factor occurs only x—1 times. By applying the formula (BE) n—1 times successively it is evident that our integral may be made ultimately to depend on dx Leeper 7 and this may be integrated by completing the square, as shown on p. 296. In the same manner all but the last fraction of (4) may be inte- grated. But this last fraction, namely, Le+M P+petq may be integrated by the method already given under the previous case (p. 829). *4q—p?>0, since 2? + px + g=0 has imaginary roots. INTEGRATION OF RATIONAL FRACTIONS 833 (a3 + 22+ 2)dx ery Solution. Since 2? + 2 occurs twice as a factor, we assume B+ ae+2_ Act B, Oo+D @+2> @+2e” #42 Iniusrrative Exampre 1. Find Clearing of fractions, we get 4a 4+2= Ax + B+ (Cx + D) (x? + 2). a at+2= Cx + Det + (A+ 2C)e4+B42D. Equating the coefticients of like powers of 2, C=1, D=1, 442C=0, B42D= This gives A=-2 B=0, C=1, D Se Sern Sls Hgrpyt grown pt jis tate. Insrrative BXxampne 2, rot Solution. Since 2? 4 1 occurs twice as a factor, we assume Qe+e4+3_ Ac+B, Oxr+D_ @+i @+y as Clearing of fractions, 2a 4+a4+8=Axt B+ (Cx + D) (+1). Equating the coefficients of like powers of x and solving, we get A=-1, B=3, C=2, D=0. 284+24+38, c—2+8 2adx ery @ Seay’ Seni 8 sos? +0) + f Gat oa Now apply formula (Z), p. 882, to the remaining integral. Here A=-1, B=8, p=0, q=1, n= Substituting, we get —24+3 1482 8 a 1482 ene tee ee t @+i” -2@ 4 Jeu ae@ED ta are tang, ‘Therefore 284043 14382 dz = log (2? +1) + oT ti C @in weenta Sore tana + 834 INTEGRAL CALCULUS EXAMPLES & eur +ip 2@4) +5 Faretane +0. log (2? + 2y¢ - 1. + log (2? + 2) as a f @+2? 4@?42) Qed att oe Tryarep awe a@aH +f Epa) ex 2 aye e an (404 8)de 409452 Qn +5) eae eon aee Sede + Hog 24" 4. VBare tan @+) a 2 Seat atop arte Ose 10 @ + 2° @ + 8) + Plog (+ 2) — 910g (a? +8) +0. 242) P43 (4at—8aydo _Sat@—2 | @ 1) @oi@ +h" @-@ aH +t log apy taretne $C. (@z4+2)de __ W2-a | 20 eo ten +6. Since a rational function may always be reduced to the quotient of two integral rational functions, ie. to a rational fraction, it follows from the preceding sections in this chapter that any rational function whose denominator can be broken up into real quadratic and linear factors may be expressed as the algebraic sum of integral rational functions and partial fractions. The terms of this sum have forms all of which we have shown how to integrate. Hence the Theorem. The integral of every rational function whose denominator can be broken up into real quadratic and linear factors may be found, and is expressible in terms of algebraic, logarithmic, and inverse-trigono- metric functions ; that is, in terms of the elementary functions, CHAPTER XXVI INTEGRATION BY SUBSTITUTION OF A NEW VARIABLE, RATIONALIZATION 189. Introduction. In the last chapter it was shown that all rational functions whose denominators can be broken up into real quadratic and linear factors may be integrated. Of algebraic functions which are not rational, that is, such as contain radicals, only a small number, relatively speaking, can be integrated in terms of elementary functions. By substituting a new variable, however, these functions can in some cases be transformed into equivalent functions that are either in the list of standard forms (pp. 284, 285) or else are rational. The method of integrating a function that is not rational by substituting for the old variable such a function of a new variable that the result is a rational function is sometimes called integration by rationalization. ‘This is a very important artifice in integration and we will now take up some of the more important cases coming under this head. 190. Differentials containing fractional powers of x only. Such an expression can be transformed into a rational form by means of the substitution a= where n is the least common denominator of the fractional exponents of x. For 2, dz, and each radical can then be expressed rationally in terms of z. +t Inwsreamive Bxamrue 1. Find f=" ae, a Solution, Since 12 is the L.C.M, of the denominators of the fractional exponents, we assume oo Here de=122Mdz, ata, ataed, atazt, $ gt 8 8 ata Anz de = fF ara = 12 fe — 2)az 7 3 Sere =$24— 4224 0=5eb-gebyo. [substituting back the value of z in terms of 2, namely, 2%.) ‘The general form of the irrational expression here treated is then 1 RQ" de, | where # denotes a rational function of 2”. 835 336 INTEGRAL CALCULUS 191. Differentials containing fractional powers of a + bx only. ‘Such an expression can be transformed into a rational form by means of the substitution atbe=z, where n is the least common denominator of the fractional exponents of the expression a + bx. For 2, dz, and each radical can then be expressed rationally in terms of 2 ee . G+atea4at Solution, Assume l¢a=2; then dz =22dz, (1+ayi=2, and (14 oe = of de, Qede aft GQealecsel sete J P41 =2aretang + 0 =2aretan(1 42)? +, ‘when we substitute back the value of z in terms of z. Tutusrrariye Exampce 1. Find f The general integral treated here has then the form 1 B[a, (a+ bx)"] de, where & denotes a rational function. 192. Change in limits corresponding to change in variable. When in- tegrating by the substitution of a new variable it is sometimes rather troublesome to translate the result back into the original variable. ‘When integrating between limits, however, we may avoid the process of restoring the original variable by changing the limits to correspond with the new variable.* This process will now be illustrated by an example. Tnwustmatve Exaurrn 1 Caleulate f” ae Solution. Assume z = 24. "Then da = 428dz, ct = .. Also to change the limits we observe that when 2=0, and when 2=16, 2=2. watde pre-4ehde 2 Ai : 4 J 14at 142 Ji (# 1+ a)® 2 de [4d 2 ade — = if eae af? de + ‘favee [$ 424darotane]? =$+ daretan2, Ans. * The relation between the old and the new variable should be such that to each value of one within the limits of integration there is always one, and only one, finite value of the other. When one is given as a many-valued function of the other, care must be taken to choose the right values. INTEGRATION BY RATIONALIZATION 337: EXAMPLES ahr 435 4 2 tala 4 tae ae —glos@et +1) +0. 3 f ipetace 6 a dz 7 = at Fall) 4c. Oe a 1B Tare 2 wba. Be dea 8 4? yg toge— 24 log (arts +1 +0. eee a a at 6. = fat + atop =? 1 daretanat +c. Saag ate, 8 Verde al at dat } . [NES = 18 2 2 4 tat 4 1604 (ab — | Woe [E+ q tog tat + 102d + 92 log (xt — 2) 146 8. f= 4-2 10g8, Ol+Vve ees * @-2t+3 2 vw, post dy= uf ove vt ta vit ytt as 3 ‘ 18. f dz = log @ £0) - oo retin e+niai. 14, (—2d2__ 224+ be) Viet Va + be i ade _ Oa +6241, (dat nF 12(42 + 1)2 16. fy Var vay = yedy — 8a) (a+ y+. Vet I+) ay —F 17. dezat14+4Ve41+ 4log(Ve+1—1) 40. “ Veqi—1 eC ) = é + Va 18. Swe se+y —8(@ +184 Slog (14+ Ve+1) +0. 19, (AE ae = 2. Ve—3 + VEaretan aVo—2 td 20. [2 — = 5.81. 21. =~ 8.986. £ ated : 22, S—"— 5 = tet nt 42+ nyt + 2logie + type. @+pi-@+nh 338 INTEGRAL CALCULUS 193. Differentials containing no radical except Va+ bx +23." ‘Such an expression can be transformed into a rational form by means of the substitution Vatlate For, squaring and solving for 2, 2 oo _2Gt beta) de. og) ee ace : and Veni.) ee b+22 Hence 2, dx, and Va + bx +2” are rational when expressed in terms of 2 Inisrmanive Exawrie 1. Find oo Vi¢ete Solution, Assume Vitate= ‘Squaring and solving for 2, #-1 Qt etd a 3 the =i, atT ss i Tes $241 Vipeee(=2—2) Pte), and T+ x + 22(: 2) R241 2@2te+ Ide (22 +1)? Qdz oe = logi@e +1 Ss e+eti epi 7 le + Ne] 2241 | when we substitute back the value of z in terms of «. 194. Differentials containing no radical except Va + bx— x°.t Such an expression can be transformed into a rational form by means of the substitution Va +be— 2 [=V@—a)(B —2)]=(@~«)z[or =(B—2)2], wherex —a and B—x are real factors of a+ bx —2*. ‘*If the radical is of the form Vn+pz + qx, 9>0)1 it may be written V@. +22, and therefore comes under the above head, where a= 7 * oak . t If the radical is of the form Vn+p2—qz, q>0, it say be written ao + Banat and therefore comes under the above head, where a=”, b a } If the factors of a+ bx 2? are imaginary, Va + bz — 2 is imaginary for all values of 2. For if one of the factors is z—m-+ in, the other must be — (z— min), and therefore b+ ax — atm — (2 m+ in) (@-m—in)=—[(e-m)2 +72], which is negative for all values of z. We shall consider only those cases where the factors are real. INTEGRATION BY RATIONALIZATION 339 For if Va+bc—2®*=V(@—a)(B—2)=(e—a)z, by squaring, cancelling out (e—a), and solving for 2, we get +B. then dea 2Gm Meds, e+1? — @+1F and Vat be — 24 [= @-a-E=92 Z. Hence 2, dz, and Va+62z—2* are rational when expressed in terms of z Junusrrative Examrie 1. Find SS Vi-n— a? Solution. Since 24+ae—-2= (412-2), we assume V@FNA—H=(e4+1)z. 2-2 ing f =25*, Squaring and solving for 2, 2 = 5— = — 62dz 3 Hence de= Oy and VEE EH =@ +N] = dz, ee yen =— 2are te Cc f = -2f3- retang + CO =~ 2aretan / at fz when we substitute back the value of z in terms of z. : EXAMPLES Lf _ 4 og SSETA te tVa—aed Ve—a+a¢z4va 2. (—— vee ce avar+2e—1 af a - re LrrC— avete—@ V24224+V2—2 4 [ogi nome are etEe | aVe+4a—4 - dx= P called the reciprocal substitution. Let us use this substitution in the next example. Iuiusrrarive Exampne 1. F na f YER as, Solution. Making the substitution «=, ax z Sete = Swe — het = Ba® ~ Saat 346 INTEGRAL CALCULUS EXAMPLES ae 1 o 1 Soares A 3a Sapat ‘Assume a? = 2. 2. {7 pede = log (e — 2) — be apt’: Assume « a 2 1 or Tpit tse te. ‘Assume z +1=2. 2 4. =—_——— + 0 Assume « Vas ai a@veta de, 1 ce 5. = = 5s —— Assume Saas oat Veta dz, on 1 6. (—— = log —__F___. Assumes ==. laa Q+ot2vitatae z 1, f Ae ag =F (1 + toga) +0. Assume 1+ loge = 2. 22 8. f— “aq Ge-gerniee. Assume e* +1 (@ +1) 9. faetget ge it jeter a +0. Assume & [ *2_ site. @ +n 3 Assume a Assume z= =e, x oo B+ sin 20 4 ssume sin — cos6 1, pene cos6) 8 _log3 5 13. are tane—F. Assume 6 =z. 44. Assume « = a sin®z, 16. ‘Assume e*—1= 2, = 16. V2t-+4 Bdt = V8 — } log (2 +-V3). Assume t+ 1=2. 5 Assume e —1 Ww. [Oo varias eae (2 4 1)de ava + Ta +1 = log 3. Assume 2 — CHAPTER XXVII INTEGRATION BY PARTS. REDUCTION FORMULAS 199. Formula for integration by parts. If w and v are functions of a single independent variable, we have, from the formula for the dif- ferentiation of a product (V, p. 84), d(w) = udv + vdu, or, transposing, udv = d(w) — vdu. Integrating this, we get the inverse formula, (A) frdoawo— f oa, called the formula for integration by parts. This formula makes the inte- gration of udv, which we may not be able to integrate directly, depend on the integration of dv and vdu, which may be in such form as to be readily integrable. This method of integration by parts is one of the most useful in the Integral Calculus. To apply this formula in any given case the given differential must be separated into two factors, namely, wand dv. No general directions can be given for choosing these factors, except that (a) de is always a part of dv; (b) it must Be possible to integrate dv; and (©) when the expression to be integrated ts the product of two func- tions, it is usually best to choose the most complicated looking one that it 4s possible to integrate as part of dv. ‘The following examples will show in detail how the formula is applied : Tnwusrearive Examriz 1, Find fe cosedz. Solution. Let and dv = cosada ; then au = de and v= feosade = sinz, Substituting in(4), gy _ py Jf 2 cose = Sing dz =asing + cost + 0, 3847 348 INTEGRAL CALCULUS Intusrrative Exaurze 2. Find fz logedr. Solution, Let u=loge and dv=2dz; a then dus" and v= fedtr=™. @ 2 Substituting in (4), Tnwwsreative Examecn 8, Find fzeedz, Solution. Let use and dv=ade; then du =e. ade and v= fade Substituting in (4), a Sein = ene 5 evra _ tet a Pg =F -4 f weeds, But 2%eedz is not as simple to integrate as zedz, which fact indicates that we did not choose our factors suitably, Instead, Jet usa and dv= edn; then du=dz and v= feeds = a Substituting in (A), oF pew Semteme.S— f Fae men ear = Stes It may be necessary to apply the formula for integration by parts more than once, as in the following example: Tnwusrnative Exameze 4, Find f2tede, Solution. Let u=a? and dv= edz; then du=2ade and v= femds =<. ‘Substituting in (4), food =o. — f &.aede a @ ter 2 () —-5 frerae. INTEGRATION BY PARTS 349 ‘The integral in the last term may be found by applying formula (A) again, which Ce S: nemmdee Substituting this result in (B), we get ten | gee ferent 22 22(,2) 40 2 ta)te Among the most important applications of the method of integration by parts is the integration of (a) differentials involving products, (b) differentials involving logarithms, (c) differentials involving inverse circular functions. EXAMPLES r 3) +e. 2 2 ae - f? log 2d = = (loge 2. fasinada =— aoosa + sina +C, 3. faresin edz = aaresine + VI-@ +0. Hawn. Let ware sin and do=dz, ete. 4 froeede =a(loge—1)+C. 5. (fore tanedz = care tane — log (1+ 2)#+0. antl 6. ferlog ade = (lege 4) +0. ce n+l e+ : re a= 1. fare tanzde aretanz — 340. 8. fue cot ydy = yarccoty + Flog (1+ y2)+C. |+ C. a>Oand 41. a, ogi : ep 10. [Pata =a lig mat wal ©. a>Oand 41. ih _f 280 log sin 849 = sin 9 (log sin @ — 1) + 0, 12. (feeds =e 204+2)40. 13, fasinzcosade = gsin 22 —decos22+C. 14. peers e#@- 22-240. 15, fare tan Vade = zare tan Va — Va + arctan Vz +0, 850 INTEGRAL CALCULUS - r 16. fowte =F [e-h +0. 20. J# logzae = (logs — 3) +0. 7 11, ftlogeds = 81. fsresinzde = 2 18. fvevdy L 22. {ffovctan 600 = — tog V5. 0 19. Sf 2@tsin ada = 23, ffstiogsas =— 7. 0 _e@/9 3a, 6x 6 24. f atearda = = (ot e+ G-S)+e 25. [o?sin gdp =2cosg + 2¢sing — pcos + 0. 26, (J (oga)tde = a logs — Blog a + 2] +0. 2 27. ae = atana—~ 5 + log cos a + C. log ade @+1) Let u=logz and dv=—@_, Hiner, Let we log # and do= - 2) ete 29. fat arcsinade =% aresine += 4? Viw + 0, pyle? — lowe + +e. 80. seo? @log tan 849 = tand (log tan 8 — 1) + 0. 31, 198 og 2) = tog. tog og) — log + 0. log (« + 1) dx 32, [SEE+DO _ 9 Ve + i flog(e +1)-2] 40. SS {log @ + 1) — 2] 33, [23 (a—2%)hde =— pa2(a— 2) — A (a—a)2 4. Hrvr, Let w= 2? and dv=(a-22)tade, ete. 34. [Vara a =i Ve= a + Saresin? +0, a wide lee —2yt 35. ial ge +ad—ayt+e. 36. [Vers ae = EVR rat Slog (2 VE H) + 0. i pee eae = ar. f- ag EV O— 4 + Zain + 0. (log we)2de 38. [OR 2 [ 4 8 at logta + Floga +f] +0. Bat 8 8. 200. Reduction formulas for binomial differentials. It was shown in § 195, p. 840, that any binomial differential may be reduced to the form "(a+ ba")? de, where p is a rational number, m and n are integers, and n is positive. Also in § 196, p. 841, we learned how to integrate such a differential expression in certain cases. REDUCTION FORMULAS 851 In general we can integrate such an expression by parts, using (A), p- 347, if it can be integrated at all. To apply the method of inteyra- tion by parts to every example, however, is rather a long and tedious process. When the binomial differential cannot be integrated readily by any of the methods shown so far, it is customary to employ reduc- tion formulas deduced by the method of integration by parts. By means of these reduction formulas the given differential is expressed as the sum of two terms, one of which is not affected by the sign of integration, and the other is an integral of the same form as the origi- nal expression, but one which is easier to integrate. The following are the four principal reduction formulas : x” (q+ byMyPtt (p+ m+tyb - Soe on . Gpimeiys) Cat ear, x" (a+ bx") mp+m+1 anp npt+m+ (4) frat bx)? de = (By i X"(at bx")? dx = [cat bx")P-4 de, Ca 4 beet Gra — PRR E DD [erence betta. ema bene mbt ta mptntm+t pe | Met ey tae. (cy [ere ar= ) i 2(a+ bx")Pdx = — While it is not desirable for the student to memorize these formulas, he should know what each one will do and when each one fails. Thus: Formula (A) diminishes m by n. (A) fails when np -+m+1=0. Formula (B) diminishes p by 1. (B) fails when np-+m+1=0. Formula (C) increases m by n. — (C) fails when m+1=0. Formula (D) increases p by 1. (D) fails when p-+1=0. 352 INTEGRAL CALCULUS I. 7o derive formula (A). The formula for integration by parts is (4) dk ud = ww — f du. (A), p. 847 We may apply this formula in the integration of feo bat) de by placing am" and dv=(atbe")Pattdes es mon = at beet then dum (m—n ttarnde and y= GET, Substituting in (4), feetinyaen Earby nb(p +1) Cr + 5 fom + be")? de. Substituting this in (2), we get staf bate "(a + ba" Pde =e Cat batt frereye ey _(m=nt)a fn, 1 DOE | eet meyae _m=nt1 n(p +1) ‘Transposing the last term to the first member, combining, and solv ing for f 2*(a-+be")Pdx, we obtain Cap beet b(np +m+1) _ a(@m—a4t) b(np +m+1) a"(a+ be" )rde. (4) if 2"(a-+ bx" Pde = wna betta. In order to integrate dv by (4) it is necessary that # outside the parenthesis shall have the exponent n—1. Subtracting n—1 from m leaves m~7n-+ 1 for the exponent of in us REDUCTION FORMULAS 3538 It is seen by formula (A) that the integration of 2"(a-+ be")"dz is made to depend upon the integration of another differential of the same form in which m is replaced by m—mn. By repeated applications of formula (A), m may be diminished by any multiple of x. When np +m +1 = 0, formula (A) evidently fails (the denominator vanishing). But in that case mi n hence we can apply the method of § 196, p. 341, and the formula is not needed. +p=0; Il. Zo derive formula (B). Separating the factors, we may write (ce) f 2™(a+ be")? da = ie 2™(a-+ be")?1(a-+ ba") de (at bare dx +0 forme + ba" Pt da. Now let us apply formula (A) to the last term of (C) by substi- tuting in the formula m+n for m, and p—1 for p. This gives a farce dba ptdy Cattery? _a(m+t) _ _. fee tape) ee Substituting this in (C), and combining like terms, we get x(a 4 bx")? YP dx = —_~___*_ (8) fare t= a anp ~ mp+m+1 Each application of formula (B) diminishes p by unity. Formula (B) fails for the same case as (A). IIL. Zo derive formula (C). Solving formula (A) for fort weyras, and substituting m+ for m, we get x™**(at bx)? tt a(m+1) _ op +n+m+1) a(m-+1) ™ (a+ bx")P—dx, (C) f x"(a+ bx)Pde= x" (a4 bx")Pdx, 354 INTEGRAL CALCULUS Therefore each time we apply (C), m is replaced by m+n. When m-+1=0, formula (C) fails, but then the differential expression can be integrated by the method of § 196, p. 841, and the formula is not needed. IV. To derive formula (D). Solving formula (B) for feo + ba")? de, and substituting p +1 for p, we get x™+3 (a byte? 7 ody = — Ca A (Dy f* (at bx)Pdx aan mintm+t mots an fxrcer oxy dx, Each application of (D) increases p by unity. Evidently (D) fails when p-+1= 0, but then p=—1 and the expression is rational. EXAMPLES wie 1 at uf age + IA-aHt +O. Solution, Here m=8,n=2,p=—4,a=1,b=—1. ‘We apply reduction formula (A) in this case because the integration of the differen- tial would then depend on the integration of f'(1— a*)~+dz, which comes under (4), p- 284, Hence, substituting in (4), we obtain matey ttt 1-241) —1(-1484+1) 0 -1(-14+84+1) =~ ga%(1— 2th y afc — 2) tae =—4er—ayt—ga—aytte =-4@ 420-27 +0. 2 S*,--(je+ gets) Va 4 Eat aresin 2 +0. @-at a *§ 8 a fea-2ytae= —2(1— at) Fae Hin. Apply (4) twice. 2 8. [@ + arybae =F VEEE + Flog (et VT e) +0. Hint. Here m=0, n=2, p=4, a=a?, b=1. Apply (B) once. de wat it 4. Imm Fat pareweon +0. Hint. Apply (C)) once. ade 2 a 2 8, (2M _ = 2 va 4S oresin® + 6. S- ag EVO Ft Poresin + REDUCTION FORMULAS 355 atde = fet 20 VEE LC. Vepe aide, 42 8) aoe -(— Tot " i 2 & . 15) +H) a. ‘ 8. fe Vea Bas = Feat — a) Vw + F aresin® +0, a Hint. Apply (4) and then (B). ae. 2 1 2 cen tc. Uae aaa te rong tO Huwr. Apply (D) onee. ae aoe 1 2 10. vee | © Ie=s oat 12a ovate aoa ua f_#H tae 4g (a+ i (@+ at 2 223 fe (ai 2 Lo, —2)8 Bat(at—2' ‘ ss ees apten tte 4 405 Fa 4 A toe +VE4F@) 40. 14. few . ay an = Fea +) Vat a rose +Vi +e) +0. ade eae ty b. (== (Qax — 22)? + PE arovers? tte. SJma77 F wld Hint. (> =—= [xh @a-z)-4dz. Apply (A) twice. Irae 2 M@=# og oy = 2A BOE Ya 4 Oot wove +6, ; = @at—eyty are vers’ +0. s 3s (@ + 8)8 42 (@ + 8%)?" Bak (a + #) 8a" 20, S- JOA +44 evinw ee. ai. wr 25. [ovary Pat. ae, Se 22. aot 28. ([ 23. f. me 2. (_e_ Pde — ast 856 INTEGRAL CALCULUS 201. Reduction formulas for trigonometric differentials. The method of the last section, which makes the given integral depend on another integral of the same form, is called successive reduction. We shall now apply the same method to trigonometric differentials by deriving and illustrating the use of the following trigonometric reduction formulas: — . __ sin™** x cos*-*x (EE) fom ‘x cos’ an ne n-1 a + fa ‘x cos"? xdx. sin™~*x cos"tx (FY fo xcostxde = — “SS m-1 inm-2, +E fo ‘x cos"xdx, in +1 x cost +3 (6) f sin x cos" xde = — SUNT 00s" x n+t mae f sin" x cos"** xdx, nti A . sin™*+*xcos"** x () sintx cost ade = “P—*E a m+1 Here the student should note that Formula (E) diminishes n by 2. (E) fails when m+n=0. Formula (F) diminishes m by 2. (F) fails when m+n=0. Formula (G) increases n by 2. (G) fails when n+1=0. Formula (H) inereases m by 2. (B) fails when m+1=0. To derive these we apply, as before, the formula for integration by parts, namely, (4 f do = tev — f ode (A), p. 347 Let uscos'-'2, and dv=sin" 2 cos drs Se oe asin, then duse—(n—Teost“*r sin dey and 9 OT REDUCTION FORMULAS 857 Substituting in (4), we get sin"*1z cos""1x @) f sin"2 cost ade = 4 =O 2a (ium ey costtade. m+ In the same way, if we let wesin""2, and dv=costz sin xd, we obtain Per : sn" cost adn = _ SMT cost tte (©) fom costedy = — Seat mo1 + sin"~*z cos" **2da, n+1 But if sin cos'~tade = f sin" (1 — cos*2) cos'~2adz = ints cost-arde— sine costed Substituting this in (B), combining like terms, and solving for f sin"2 costadz, we get sin™+* x cos"-*x m+n n= (By f sin” x cos" xde = sf sint x ost? ad, m+n Making a similar substitution in (C), we get See (FY f sin x cos" xde = — Sin” * x costtt x m+n m—1 myn f sin”"~? x cos" xdx. Solving formula (£) for the integral on the right-hand side, and increasing n by 2, we get sin™** x cos"**# x n+1 m+n+2 n+i (G) S sin”x cos" xdx = — + sin™ x cos"*? xdx, 858 INTEGRAL CALCULUS In the same way we get, from formula (F), _[feiercast xt = Sa m+1 + REESE (starts xenst ade. m+1 Formulas (£) and (F) fail when m+n=0, formula (6) when n+1=0, and formula (Z) when m+1=0. But in such cases we may integrate by methods which have been previously explained. It is clear that when m and n are integers, the integral faint costae may be made to depend, by using one of the above reduction formulas, upon one of ths following integrals: fe J sinads, feos ads Ji sinzeosats, [22 = [eons sing =f see ade, Soxtams : f tan ada, f cob ada, cos.2 cosesing all of which we have learned how to integrate. EXAMPLES ‘sing cos’ + sin costa 6 24 Solution. First applying formula (F'), we get ‘ (4) _fointe costade =— snzoote +4 5 J costed. [Here m=2, n= 4.) 1 in? — S : 1. fsinta costade = +g bine cosa + 2) + 6. Applying formula (£) to the integral in the second member of (A), we get dean Sint Coe | Bg (B) fcostaae = EE +5 feostade. [Here m=0,n=43 Applying formula (Z) to the second ntember of (B) gives sine cosa Feild (cy foostade = 2 +5 Now substitute the result (0) in (B), and then this result in (A). ‘This gives the answer as above. ; cosa /sinte sinte _ sing’ 2. fsints costade = 22 ( ee °. 2 3 12 +i ot Saat oe +0. Sint cosa 3 REDUCTION FORMULAS 359 costeds cote Be 4. — SMF 3 — cost ©. Jig gO cota) — Sr + 5. fecctede =F seo tang + Flog (see + tana) +0. ie fesctede = Jose cote + Z log (sex — cota) +0. costada ___ cosa 8 « a =- — cosa Flog tan +6. Site Sane ee ae 8. fsintada ed i a) +540. 1 8) 3 6 (a [ aad) *3 a o sing 1 5 5 Zeose C cose t 12cosp * ati 7g 108 Seed + tang) +0. 10, fi ‘sect gdp = 11. f costae = SEE (coste + Zeostl + SF caste + Fost) + 2 4 0. dy 1 ( 1. (4 = Jay cory ae 3 ) + Prog secu + tany) +6. i = 3, _ 850 13, Sf sinSz costadz = = 16. Pcos* ada = = 35m T _8e 14, fi sintada =F 17. fsintzas =. a Zeosteat = 37. 15. _ sin aa =< 18. 7 cost idt = 5, 202. To find f e*sin nxdx ana f “* cos nxdx. Integrate e** sin nadx by parts, letting use, and dv=sinnedz; then du =ac*dx, and v=—°S™, n Substituting in formula (A), p. 347, namely, i Sie f edu, we get (4) fersinnede = Sen S fercosnads. Integrate e* sin nadx again by parts, letting ussinnz, and dv=e*de; then du=ncosnedz, ad v=—. a 360 INTEGRAL CALCULUS Substituting in (A), p. 847, we get @) f et sin nde = SM _® f e cos nada. a Eliminating f &* cos nad between (4) and (B), we have (+n) fesin nate = (asin nz —n cos ne), : (asin ne — n cos ne or e sin nada = ee. e+H Similarly, we may obtain [e cos nedx =" (nsin na + acosna) . ¢ y e+ In working out the examples which follow, the student is advised not to use the above results as formulas, but to follow the method by which they were obtained. EXAMPLES 1. fersinads =F (sins — cos2) +0. 2 (feeosade =F (ine + cose) +0. 3. {fer cossde = 5 (8 sin 82 + 200822) +0. sin xe sing + cost | (ane analon2 | 0. a lee tet cos2adz 1g Be Tyas sin 2x — 8 cos2z) +0, 4 e 2sin 2a + cos2a\ f Pade = & (1 28in2e + cos2z) | o 6. fersintede 5( 7 eo, 1. fe costaaa =F (1 + canteens), c. 8. feeosgar = a (sing + cos) + c. ema sin aa @ 9. fon (inaa+ cosaa) da = +0. 10, fe=(sin22— cos 22) de = (sin 22 — 5 cos22) +0. 1. foessinade =}. CHAPTER XXVIII INTEGRATION A PROCESS OF SUMMATION 203. Introduction. Thus far we have defined integration as the inverse of differentiation. In a great many of the applications of the Integral Calculus, however, it is preferable to define integration as a process of summation. In fact, the Integral Calculus was invented in the attempt to calculate the area bounded by curves, by supposing the given area to be divided into an “infinite number of infinitesimal parts called elements, the sum of all these elements being the area required.” Historically, the integral sign is merely the long S, used by early writers to indicate “sum.” This new definition, as amplified in the next section, is of fun- damental importance, and it is essential that the student should thoroughly understand what is meant in order to be able to apply the Integral Calculus to practical problems. 204. The fundamental theorem of Integral Calculus. If $(2) is the derivative of f(x), then it has been shown in § 174, p. 815, that the value of the definite integral Yh : fe@e=-FOSO gives the area bounded by the curve y=$(@), the X-axis, and the ordinates L erected at 2=a and z= 6. Now let us make the following con. —okaxt 4» & 1 struction in connection with this area, Divide the interval from «=a to ¢=6 into any number n of equal subintervals, erect ordinates at these points of division, and complete rectangles by drawing horizontal lines through the extremities of the ordinates, as in the figure. It is clear that the sum of the areas of these m rectangles (the shaded area) is an approximate value for the area in question. It is further evident that the limit of the sum of the areas of these rectangles when their number n is indefinitely increased, will equal the area under the curve. 361 362 INTEGRAL CALCULUS Let us now carry through the following more general construction. Divide the interval into » subintervals, not necessarily equal, and erect ordinates at the points of division. Choose a point within each sub- division in any manner,* erect ordinates at these points, and through their ex- tremities draw horizontal lines to form rectangles, as in the figure. Then, as before, the sum of the areas of these n rectangles (the shaded area) equals approximately the area under the curve; and the Limit of this sum as n increases without limit, and each subinterval ap- proaches zero as a limit, is precisely the area under the curve. These considerations show that the definite integral (4) may be regarded as the limit of a sum. Let us now formulate this result. (1) Denote the lengths of the suecessive subintervals by Az, Az, Ary +, Az, (2) Denote the abscissas of the points chosen in the subintervals by a, yyy Wey 4 Bye Then the ordinates of the curve at these points are SG)» $C) PO)» 1 OC)» (3) The areas of the successive rec- tangles are obviously P@)Ary, $@,)Aty $(%,)Amy + $(%,) Ary (4) The area under the curve is therefore equal to limit, [s (2,) Ae, +6 (a,) Av, +6 (2) Ae, +o +6) Aa, |. n=0 Ct te A But from (A) the area under the curve = ii (a) de. Therefore our discussion gives : (By) f “b@de= Ime] ba ran tb Gant +65) AM]. * This construction includes the previous one as a special case, namely, when the point is, ehoson at one extremity of a subinterval. INTEGRATION A PROCESS OF SUMMATION 363 This equation has been derived by making use of the notion of area. Intuition has aided us in establishing the result. Let us now regard (B) simply as a theorem in analysis, which may then be stated as follows: > FUNDAMENTAL THEOREM OF THE INTEGRAL CALCULUS Let $ (a) be continuous for the interval x=atox=b. Let this interval be divided into n subintervals whose lengths are Ax, Ax, +++, Ax,, and points be chosen, one in each subinterval, their abscissas being ty 2, ++" 2, respectively. Consider the sum ©) $@ Az, +6(@,) Ar, + +9@) A2,=> 6) Az,. Then the limiting value of this sum when n increases without limit, and each subinterval approaches zero as a limit, equals the value of the definite integral b oe ii p(«) da. Equation (B) may be abbreviated as follows: : eo) fe@enne md) Ae The importance of this theorem ae from the fact that we are able to calculate by integration a magnitude which is the limit of a sum of the form (C). It may be remarked that each term in the sum (C) is a differen- tial expression, since the lengths Az, Ar, ---, Az, approach zero as a limit. Each term is also called an element of the magnitude to be calculated. The following rule will be of service in applying this theorem to practical problems. FunpAMENTAL THEOREM. RULE Frast Ser. Divide the required magnitude into similar parts such that it is clear that the desired result will be found by taking the limit of a sum of such parts. Sxconp Srer. Find expressions for the magnitudes of these parts such that their sum will be of the form (C). Tummy Srer. Having chosen the proper limits x=a and x=), we apply the Fundamental Theorem Int So@an= [pede and integrate, 364 INTEGRAL CALCULUS 205. Analytical proof of the Fundamental Theorem. As in the last section, divide the interval from 2 =a to 2 =D into any number ‘n of subintervals, not necessarily equal, and de- note the abscissas of these points of division by dj, Dg, «++, Ona, and the lengths of the subinter- vals by At;, At, +++, At. Now, however, we let af, 24, +++, 4 denote abscissas, one in each inter- val, determined by the Theorem of Mean Value (44), p. 165, erect ordinates at these points, and through their extremities draw horizontal lines to form rectangles, as in the figure, Note that here @(z) takes the place of g/(z). Applying (44) to the first’ interval (a =a, b=b,, and aj lies between a and },), we have = oh), on, since a= An, Fy) —F(@) = 6(@8) An. Also S(bs) —F (b,) = (£8) Aty, for the second interval, Fs) —F (bs) = . = . . + ete., FO) —SOn-1) = (2h) At, for the nth interval. Adding these, wé get (2) SQ) —F@) = (ef) Avy + (@f) Atty + +++ + G(R) Arn. But (#1) - At, = area of the first rectangle, (26) - At, = area of the second rectangle, ete. (v4) Az,, for the third interval, Hence the sum on the right-hand side of (Z) equals the sum of the areas of the rectangles. But from (A), p. 361, the left-hand side of (Z) equals the area between the curve y = (2), the axis of X, and the ordinates at z= a@andz=b, Then the sum ®) Yee an a equals this area. And while the corresponding sum (@) Dear a [Where x; is any abscissa of the subinterval Ay] (formed as in last section) does not also give the area, nevertheless we may show that the two sums (F) and (@) approach equality when n increases without limit and each subinterval approaches zero as a limit. For the difference (z/) — (zi) does not ex- ceed in numerical value the difference of the greatest and smallest ordinates in Ars. And furthermore it is always possible* to make all these differences less in numerical value than any assignable positive number e, however small, by continuing the process of subdivision far enough, i.e. by choosing n sufficiently large. Hence for such a choice of n the difference of the sums (F) and (@) is less in numerical value than ¢(b— a), * That such is the case is shown in advanced works on the Caleulus. INTEGRATION A PROCESS OF SUMMATION 365 i.e. less than any assignable positive quantity, however small. Accordingly as n in- creases without limit, the sums (J) and (@) approach equality, and since (F) is always equal to the area, the fundamental result follows that i * o(e) de = limit Y oe aes, in which the interval [a, b] is subdivided in any manner whatever, and 2; is any abscissa in the corresponding subinterval. 206. Areas of plane curves. Rectangular codrdinates. As already explained, the area between a curve, the axis of X, and the ordinates x=aand 2=b is given by the formula (A) area = f "yd, the value of y in terms of x being substi- tuted from the equation of the curve. Equation (A) is readily memorized by observing that ydx represents the area of a rectangle (as CR) of base dz and altitude y. It is convenient to think of the required area ABQP as the limit of the sum of all such rectangles (strips) between the ordinates AP and BQ. Let us now apply the Fundamental Theorem, p. 363, to the caleu- lation of the area bounded by the curve «= $(y), (AB in figure), the axis of ¥, and the horizontal lines y = ¢ and yaa. yf Frrsr Srer. Construct the n rectangles yed! asin the figure. ‘The required area is clearly the limit of the sum of the areas of these rectangles as their number increases with- Yfyyy, out limit and the altitude of each one ap- _<« AWWW proaches zero as a limit. Seconp Srer. Denote the altitudes by % | > Ay, Ayy etc. Take the point in each inter- val at the upper extremity and denote their ordinates by y,, y,, ete. Then the bases are $(y,), (y,)s ete. and the sum of the areas of the rectangles is SODADA EOD AIH FOWIAM= DOU) Ate Tarp Srer. Applying the Fundamental Theorem gives a gay. WY». Yi» YY) \ limit 366 INTEGRAL CALCULUS Hence the area between a curve, the axis of Y, and the horizontal lines y=c and y = dis given by the formula ; @) area = if xdy, the value of 2 in terms of y being sub- stituted from the equation of the curve. Formula (B) is remembered as indicating = the limit of the sum of all horizontal strips (rectangles) within the required area, x and dy being the base and altitude of any strip. VEE” me Intusrrative Exampre 1. Find the area included between the semicubieal parab- ola y? =a and the line # = 4, Solution, Let us first find the area OMP, half of the required area OPP’. For the upper branch of the curve y =" ‘2, and summing up all the strips between the limits = Oand = 4, we get, by substituting in (A), area OMP = if “yde = S, “hae = 94 = 124. 0 ° y Hence area OPP’ = 2.14 = 253. If the unit of length is one inch, the area of OPP” is 253 square inches. Nors. For the lower branch area OMP’ =fic ah) dx =— 124, —a; hence | This area lies below the axis of « and has a negative sign because the ordinates are negative. In finding the area OMP above, the result was positive because the ordinates were positive, the area lying above the axis of x. | ‘The above result, 25, was the total area regardless of sign. As we shall illus- | trate in the next example, it is important to note the sign of the area when the curve crosses the axis of X within the limits of integration. Inwsrrative Exanrre 2. Find the area of one yj arch of the sine curve y = sin. Solution, Placing y = 0 and solving for 2, we find a=0, m 2m, ete. Substituting in (A), p. 865, area OAB » a0 Also area Bob = f vd = f sinedz =— 2, > ae and area OABOD = f yde = [-“sinede lo ° This last result takes into account the signs of the two separate areas composing the whole, The total area regardless of these signs equals 4, INTEGRATION A PROCESS OF SUMMATION 367 Iuiusrrative Exampre 3, Find the area included between the parabola a? = 4ay and the witch 8a? rr Te Solution. To determine the limits of integra tion we solve the equations simultaneously to find where the curves intersect. ‘The codrdinates of A are found to be (— 2a, a), and of C (2a, a). It is seen from the figure that area AOCB = area DECBA — area DECOA. * 8abde 2 But area DECBA = 2 x area OECB = 2 f. 2ma?, o +4 tag! 4a and area DECOA = 2 x area ORC =2 f° = de ==", o 4a 8 4a Hence aren AOCB = 2ma?—"F —2a*(w—§). Ans. Another method is to consider the strip PS as an element of the area. If y/ is the ordinate corresponding to the witch, and y” to the parabol2, the differential expression for the area of the strip PS equals (y— y”)dz. Substituting the values of y’ and y” in terms of ¢ from the given equations, we get area AOOB = 2 x area OOB aa ia fw va aay gaia af, (eta h)” =20(r—§). 2 Iniusrrarive Exanrre 4, Find the area of the ellipse = + ¢ =1 Solution. 'To find the area of the quadrant OAB, the mits are ¢=0,@ =a; and Hence, substituting in (A), p. 865, eee area OAB= fe aban a, = [Ee ee- ant + Baan aa 4 ‘Therefore the entire area of the ellipse equals rab, 368 INTEGRAL CALCULUS 207. Area when equation of the curve is given in parametric form. Let the equation of the curve be given in the parametric form z=fO, y=bO- We then have y=$(t), and de=f'(dt, which substituted * in (A), p. 365, gives (4) area = [“pCes'(t)dt, where , when e=a, and t=, when e=5. ‘We may employ this formula (A) when finding the area under a curve given in parametric form. Or we may find y and dx from the parametric equations of the curve in terms of ¢ and dt and then substitute the results directly in (A), p. 865. ‘Thus in finding the aren of ne ellipse in Illustrative Example 4, p. 867, it would have been simpler to use the parametric equations of the ellipse z=acsg, y=dsing, where the eccentric angle ¢ is the parameter (§ 66, p. 81). Here y=bsing, and de=—asingdg. When z=0, o=55 and whenz=a, $=0. Substituting these in (4), above, we get « ° mab = — in? gdp = 7, aren OAB = fyde = [asin ody = 7 n a Hence the entire area equals wab. Ans. ) 4 2 EXAMPLES 1, Find the area bounded by the line y= 52, the axis of X, and the ordinate 2=2, Ans. 10, 2, Find the area bounded by the parabola y2= 42, the axis of Y, and the lines y=4andy=6. Ans. 123. 3, Find the area of the circle 22 + 9? =r". Ans, 7, 4, Find the area bounded by y= 92 and y = 82, Ans. } 5. Find the area bounded by the codrdinate axis and the curve y=e. Ans. 1. 6. Find the area bounded by the curve y = loga, the axis of y, and the lines y=Oandy=2, Ans. @—1. 7. Find the entire area of the curve 2# 4 y# = at. Ans. $ 7a! 8, Find the area between the catenary y : [e+ 4], the axis of Y, the axis of X, and the line ¢ =a, @ é@—1). ze 1] "For a rigorous proof of this substitution the student is referred to more advanced treatises on the Calculus. Ans. INTEGRATION A PROCESS OF SUMMATION 369 9, Find the area between the curve y = loge, the axis of X, and the ordinates landz=a, Ans. a(loga—1) +1. 10. Find the entire area of the curve fa\2 [y\t 8xab ry (Ya. Ans, 372, (}) a (i) 4 11. Find the entire area of the curve a®y? = «(2a —2). Ans. a?, 12. Find the area bounded by the curves z(y—e)=sinz, and 2zy =2sinz + 2%, 1 the axis of Y, and the ordinate « ‘Ans. if (@—40%)de=e—J=1.554- 0 * 13, Find the area between the witch y = —©" and the axis of , its asymptote. eye a . zt ns. 4°, 2 14, Find the area between the cissoid y®=—*"— and its asymptote, the line 2a. oe Ans. 8ra*, 15. Find the area bounded by y = 2%, y= 8, and! + axis of Y. ‘Ans, 12, 16. Find the area included between the two parabolas y? = 2px and a? = 2py. ne te 3 17. Find the area included between the parabola y® = 22 and the circle y® = 4a— 22, and lying outside of the parabola. Ans, 0.475. 18. Find the area bounded by y = 2%, y =a, y = 22. Ans. }. 19. Find an expression for the area bounded by the equilateral hyperbola a? — y° = a°, the axis of X, and a line drawn from the origin to any point (e, y). at+y Ans. —\ . _. 20, Find by integration the area of the triangle bounded by the axis of Y and the lines 2¢ + y + 8 =O and y =— 4, Ans, 4, 21. Find the area of the circle = 7 c0s8, 6 being the parameter, Ans. mr? 22. Find the area of the ellipse cos 4, y=Ddsing, where the eccentric angle ¢ is the parameter. Ans. mab, 28, Find the area of the cardioid =a (2cost— cos24, y =a Qsint— sin 20), Ans. }ra®, 24, Find the area of one arch of the cycloid 2 =a(0—sind), y (1— cos 6), @ being the parameter. ‘Hive. Since varies from 0 to 27a, 8 varies from 0 to 27. Ans. 3-702; that is, three times the area of the generating circle. 3870 INTEGRAL CALCULUS 25, The locus of A in the figure, p. 82, is called the “companion to the cycloid.” Its equations are 0, a(1— cos 6). y Find the area of one arch, Ans. 2ma?, 26. Find the area of the hypoeyeloid i / [2 = acos*6, a ! y =asin®@, Bra : 9 being the parameter. Ans. } that fs, three eighths of the area of the ciroumseribing circle. 27, Find the area of the loop of the folium of Descartes m4 y9=Sary. Bat Hunn, Let y= tz; then 2=2He, 7 ve Batt 1-28 ¥ Bae | and de Badt. +8 are ‘The limits for ¢ are 0 and 2. 28, Find by integration the areas bounded by the following loci : @ @-apPaey Ans. Ys 0) @-7P =H, are (©) @y=2(e— a2), y da, @«d+v) ™ @) y=20—2"), t @ z=" u-)), ve (g) Y=at(2%+41). Areaofloop. — y$5. (b) #2 =22(2241). Areaofloop. % — (p) #= 100, 208. Areas of plane curves. Polar codrdinates. Let it be required to find the area bounded by a curve and two of its radii vectors. For this purpose we employ polar codrdinates. Assume the equation of the curve tobe p74, and let OP and OD be the two radii. Denote by « and 8 the angles which the radii make with the polar axis. Apply 1 the Fundamental ‘Theorem, p. 363. First Step. The required area is clearly the limit of the sum of circular sectors constructed as in the figure. Sxconp Srv. Let the angles of the successive sectors be 40, A8, | etc., and their radii p, py ete. Then the sum of the areas of the sec- tors * is Sh pid0, fos ' * Tho aren of a cireular sector= }radius x are. Hence the area of first sector= 4; “PA, = boZA0,. ete. 4070, +4 p00, +--+ 3 p20, INTEGRATION A PROCESS OF SUMMATION 871 Timp Srer. Applying the Fundamental Theorem, fae imi 5 p28, = -{" p'd8. Hence the area swept over by the radius vector of the curve in mov- ing from the position OR to the position OD is given by the formula 8 (A) area =} i pd, a the value of p in terms of @ being substituted from the equation of the curve. Iniusrrative Examrte 1, Find the entire area of the lemniscate p? = a? cos2 6. Solution, Since the figure is symmetrical with respect to both OX and OY, the whole area = 4 times the area of OAB. Since p = 0 when 9 =~, we see that if 0 varies 4 from 0 to 3 the radius vector OP sweeps over the IK LLL area OAB, Hence, substituting in (A), 1 8 entire area = ‘a B= 4-— alt entire area = 4 x area OAB = 4 ae 8 =20 f Urat is, the area of both loops equals the area of a square constructed on OA as one side, ‘cos 2646 = a? ; EXAMPLES 1, Find the area swept over in one revolution by the radius vector of the spiral of Archimedes, p = a8, starting with @ = 0, How much additional area is swept over in the second revolution ? an, 222, 2, Find the area of one loop of the curve p= a cos26. > 3. Show that the entire area of the curve p= asin 26 equals one half the area of the cireumscribed circle. 4, Find the entire area of the cardioid p = a(1— cos). One ; that is, six times the area of the generating circle, 5. Find the area of the circle p = a cos. Ans. 6. Prove that the area of the three loops of p area of the circumscribed circle, asin8@ equals one fourth of the 7. Prove that the area generated by the radius vector of the spiral p = e? equals one fourth of the area of the square described on the radius vector, 8. Find the aren of that part of the parabola p = asce?® which is intercepted between the curve and the latus rectum, : Bat Ans, 52, 9, Show that the aren bounded by any two radii vectors of the hyperbolic spiral 04 = ais proportional to the difference between the lengths of these radii. 372 INTEGRAL CALCULUS att 10. Find the area of the ellipse p? = ay ap Ans. ab, 11. Find the entire area of the curve p = a(sin2 + cos 26). Ans, 7a. 2 12, Find the area of one loop of the curve p? cos 0 = a? sin38, Ans. i — Floss. . 2 13, Find the area below OX within the curve p = asin® é. Ans. (104+ 27 Oa 14, Find the area bounded by p? = asin 48. Ans. a. 15. Find the area bounded by the following curves and the given radii vectors : (a) p=tan8, 6=0, o=%. (@) p=sec + tand, = 0, o=%. 6. 8 7 (b) p= et, @=7, g=7. =sino + cos, @=0, 6=7. @ pHa ea] 7 () p=sing + cos5, 0=0, 8 a (0 p= asec’, 3 (© p=asind + be0s8, = 0, 9= 5. 16. Find the area inclosed by each of the following curves : 4sin20. (@) p=14 2cos0. © 2(1— cos 6). cos8 6. (©) p=8 + 008 0. () p=a(1+sind). sin 40, () p=2—sing, (i) p= ac0s54. 209. Length of a curve. By the length of a straight line we com- monly mean the number of times we can superpose upon it another straight line employed as a unit of length, : as when the carpenter measures the length of a board by making end-to-end applica- ¢ tions of his foot rule. Since it is impossible to make a straight <' line coincide with an are of a curve, we cannot measure curves in the same manner as we measure straight lines. We proceed then as follows: Divide the curve (as AB) into any number of parts in any manner whatever (as at C, D, #) and connect the adjacent points of division, forming chords (as AC, CD, DE, EB). The length of the curve is defined as the limit of the sum of the chords as the number of points of division increases without limit in such a way that at the same time each chord separately approaches zero as a limit. Since this limit will also be the measure of the length of some straight line, the finding of the length of a curve is also called “ the rectification of the curve.” The student has already made use of this definition for the length of a curve in his Geometry. Thus the circumference of a circle is defined as the limit of the perimeter of the inscribed (or circumscribed) regular polygon when the number of sides increases without limit. INTEGRATION A PROCESS OF SUMMATION 873 The method of ‘the next section for finding the length of a plane curve is based on the above definition, and the student should note very carefully how it is applied. 210. Lengths of plane curves. Rectangular codrdinates. We shall now proceed to express, in analytical form, the definition of the last sec- tion, making use of the Fundamental Theorem. Given the ourve y _ 6¢1), and the points P'(a, ¢), Q(b, @) on its to find the length of the are P'Q. Finsr Spe. ‘Take any number n of points on the curve between P' and Qand draw the chords joining the adjacent points, as in the figure. The required length of are P’Q is evidently the limit of the sum of the lengths of such chords. Secon Srxr. Consider any one of these chords, P'P” for example, and let the codrdinates of P’ and P" be PI@', y!) and P"(a! + Ad’, y+ Ay'). Then, as in § 90, p. 134, PiplaVGay ey : | or, per=[1+ (At) [ae : Ad! (Dividing inside the radical by (32) and multiplying outside by a2") But from the Theorem of Mean Value, (44), p. 165 (if Ay’ is denoted by £6) —f(a) and Aa! by b— a), we get Ay! x =f" (a), al (15 — 16 log 2). 10, Find the volume generated by revolving about OX the areas bounded by the following loci : : e (a) The hypocycloid at + y = ai. a. = : 3 (b) The parabola 2t + yt = at,2=0,y =0. _ (0) One arch of y = sinz. ° (@) The parabola y? = 4, 2m. () y=se,2=1,y=0. qe-D () Y= 92, y=82. az. : 8a3 (g) The witch y = eer 4a’, (bh) PA+2)=1y=0c=00250, (l) y(l+e)=2,y=0,0=0,2=8, (m) y@— 2)? =1,y=0,e=38,0=4. (n) #=(@+2)',y=0,2=-1e=0 (©) @-l)y=2,y=0,2=2,0=5. Oyaay @) 6-2) () 4yt 2,05 380 INTEGRAL CALCULUS 11. Find the volume generated by revolving the areas bounded by the following loc! : About OX About OF @) y=e,2=0,y=0 Qa. (b) y=a*,2=2,y=0. oe (©) ay? = 28 $a, et re @ Bt pat G4. ‘ay, (y\t Saat © (2) +(%) a7 () Yr =02,y=0,2 () 2 =16-y,y (g) yw =4-2,2=0, (k) 22 + 9y? = 36. (ht) Y=e4+9,2=0. Oy 0, @ @=ltyy=0. (n) y =0,2 12. Find the volume generated by revolving one arch of the cycloid a=rareversY— Vary ye about OX, its base. r fe Huvr, Substitute d= 40% + and limits y=0, y=27, in (A), p. 378. Ang, 5 ate, Vary P 13. Find the volume generated by revolving the catenary y = gia+ e #) about the axis of X from ¢ = 0 toz =b. 14, Find the volume of the solid generated by revolving the cissoid y? about its asymptote « = 2a. a 2 ee 15. Given the slope of tangent to the tractrix @% + find the solid generated by revolving it about OX. de Vay Ane. $a, 16. Show that the volume of a conical cap of height a cut from the solid generated by revolving the rectangular hyperbola 2 — y? = a? about OX equals the volume of a sphere of radius a. 17. Using the parametric equations of the hypocycloid if = aos", y=asin®@, o find the volume of the solid generated by revolving it about OX. Ans, BEE 18. Find the volume generated by revolving one arch of the cycloid £ =a(— sind), about its base OX. nee Ans. br%at, Show that if the arch be revolved abont OY, the volume generated is 6 ma, 19, Show that the volume of the egg generated by revolving the curve xy? + (¢ — a) (c —b) = 0, (a (i+. IG: Tae, = lateral area of second frustum, 2 re QU+I@, Tbe, = lateral area of last frustum. Hence D27C-)A+F~@)*}Aa,= sum of lateral areas of frustums, a This may be written Dy Deny. (1+F'@)Tae,— Yell +l @)* aa a mA Tauep Step. Applying the Fundamental Theorem to the first sum (using the limits 04 = a and OB =6), we get fat Somat e sr Pana [aay ltr @y Tae ‘The limit of the second sum of (D) form + 00 is zero." Hence the area of the surface of revolution generated by revolving the are CD about OX is given by the formula (4) S,=27 ff “yt + (2) Fas, where y and & in terms of # must be substituted from the equation of the revolved curve, and S denotes the required area. Or we may write the formula in the form . S=2mf[ yds, ds = (da? + dyyt= [i . (yf ~ This formula is easily remembered if we consider a narrow band of the surface included between two planes perpendicular to the axis of revolution as the element of area, and regard it as the convex surface remembering that (20), p. 135 * This is enslly soen as follows. Denote the second sum by Sw If « equals the largest of the positive numbers |«|, |¢,|, +**, |€xl, then Ss > Ley @aptan. a The sum on the right is, by (B), p. 381, equal to the sum of the chords CE, EF, etc. Let this sum be Jy, Then S, ely. Since limite=0, Sp is an infinitesimal, and therefore 0.

You might also like